Você está na página 1de 177

Pharmacology 1. Which is correct?

Tranexamic acid is used to reduced bleeding time (YUP) -this is a synthetic antifibrinolytic agent, for simple extraction with patients that are heamophiliac or with von willebrands disease 2. metronidazole , what is true intolerance with alcohol (YUP) - synthetic antimicrobial - highly effective against trichomoniasis ( common vaginal inf) - (+)bacteria and protozoans who are obligate anaerobes - narrow spectrum, low risk of suprerinfection - good for ANUG, Syphilis, pericoronitis - excretion by urine, saliva - metallic taste, transient rashes - reaction with alcohol, causing flushing, sweating, palpitations and nausea 3. Disinfectants = cleansing of any items and implies removal of bacterial contamination, particulary by antiseptics, - they are non selective in their action on cells and need to be used in relatively high concentrations - they require considerable time to destroy significant amounts of bacteria - they are usually inhibited by organic materials such as pus or blood - they are usually ineffective against bacterial spores and will not necessarily destroy viruses 4. Antiseptics: a) coagulation and precipitation of cell proteins phenols b) damage to cell membrane, detergents c) oxidation, halogens 5. Which of the following are used as an antiviral agent? a) Glutaraldehyde*, cold sterilization, bactericidal (30 mins) + many spores (7-10 hrs), weak against TB bacillus, viricidal, 2% for impression materials b) sodium hydrocloride (1% NaOH - miltons solution)*-viricidal, bactericidal c) iodine*-some spores and viricidal, bactericidal, fungi, 7.5% (scrub) 10% skin antiseptic d) 70% alcohol- bacteriocidal e) chlorhexidine, bacteriocidal f) quarternary amonium, bacteriostatic, good detergent but poor antiseptic 6. When a patient is on steroid (anti-inflammatory) therapy, you send him to the physician because:

a) *undermined adrenal cortex, inability to withstand stress and can precipitate acute hypotension and circulatory failure b) difficulty in healing c) to use as an anesthetics with vasoconstrictor 7. Antibiotics should be used routinely to prevent infection arising from oral surgery in patients suffering from? (prophylactic) a) Agranulocytosis* b) severe uncontrolled diabetes* c) leukemia* d) aplastic anaemia* e) mumps f) renal dialysis patients* g) under radiation* 8. Endogenous morphine-like substance which control central pain (brain)are known as? a) bradykinin b) peptides c) prostaglandin d) endorphins* e) enkephalins* 9. Denture Stomatitis treatment? Due to fungal infection a) Metronidazole tx of choice for acute ulcerative gingivitis b) Tetracycline c) systemic penicillin d) amphotericin B* e) mycostatin* nystatin 3 x a day for 14 days, denture must be soaked in nystatin solution overnight 10. Tetracycline is used in 10 years old growth of candida albican - Broad spectrum bacteriostatic but not effective for viruses and protozoans 11. If patient is sensistive to penicillin? Erytromycin 12. For patients with bacterial endocarditis? Amoxycillin 2 grams 1 hr prior to surgery, 600 mg clindamycin or 500 azithro 13. Which is least likely to produce post surgical bleeding? a) antibiotic therapy b) poor surgical c) aspirin, prevent platelet aggregation thus prolong bleeding time d) codein* analgesic 14. Hydrocortison (secreted from the adrenal cortex), which is correct? Produces its anti-inflammatory action through inhibition of prostaglandine, nope, Produces its

anti-inflammatory effect by inhibiting phospholipase and thus reducing the synthesis of prostaglandins and leukotrienes. The prostoglandins causes trhe pain, edema and vasodilation of acute inflammation and the leukotrienes mediate cellular infiltration, mucosal secretion and bronchoconstriction in more prolonged inflammation. 15. Which of the following drugs can cause bleeding during operation a) Aspirin* b) codein c) antibiotic d) vasoconstrictor 16. barbiturates are associated with the following, except a) analgesia* b) general aneasthesia c) hypnosis d) sedation 17. What drug has common depressive side effect, respiratory a) antihistamine b) Tranquilizers* c) Non-barbiturate sedative drugs d) Anti-depressant e) Narcotics* 18. Least possible cause of post operative bleeding a) aspirin b) antibioitic c) codeine* d) poor technique 19. A 37 years old female complains of gingival enlargement on anterior teeth. On examination both marginal and attached gingival, on bilateral sides: dilantin gingival hyperplasia 20. A patient age 13 year-old using dilantin tabs oral examination reveals gingival hyperplasia the treatment is: a) oral prophylaxis* b) oral hygiene measure, plaque removal* c) gingivoplasty* d) stop medication

21. Acute ulcerative gingivitis, treatment of choice is:

a) penicillin b) metronidazole* 22. Which is correct? a) diazepam has hangover effect b) barbiturates (ex Nembutal) has a hangover effect* note: barbiturates problem are the following - ready tendency to widespread addiction - respiratory depressant effect - frequent poisoning - interaction with other drugs 23. Which of the following is incorrect about Benzodiazepines? a) No side effects and high amnesic effect* b) Powerful anxiolytic agent c) Muscle relaxant d) Anticonvulsant e) Amnesic effect 24. Which of the following is correct about Diazepam (Valium)? a) The active metabolites can cause, post-op headache true (hang-over like effect) b) Active metabolites remain for 8 hours, even more, cannot drive after 12 hours 25. Which of the following is correct about intra-oral infections a) Amox and Metronidazole have equal penetrating properties b) Amox can effectively lower most intra-oral infections* 26. A patient taking a MONOamine Oxidase inhibitor represents for dental treatment which is contraindicated antidepressant, persist even after 2 weeks a) barbiturate b) penicillin c) a local anesthesia with felypresine d) acetylsalicylic acid e) pethidine or morphine* - prolonged umconsciousness and deepen respiration or depressed 27. What is true about benzodiazepines? a) It can be given safely to children paradoxical stir b) It can cause after effect (hang-over) c) It is a tranquilizer and analgesic ( dont possess analgesic properties)

28. Tetracycline antagonist of penicillin ( true because tetracycline is bacteriostatic, while penicillin is bacteriocidal) 29. Tetracycline given with milk ( Calcium or iron -chelates) will inhibit absorption 30. Tetracycline used in juvenile periodontitist because it has the best concentration compared to other antibiotics in the gingival fluid Note: Its binding or etching to hard tissueshas led to its use against severe periodontal disease 31. metronidazole good in treatment of all gingivitis, nope good for ANUG 32. synthetic narcotic cause respiratory depression 33. hydrocortisone has anti-inflammmatory effect 34. Tricloroacetic acid, a strong acid, has been used by dentists for chemical cautery of hypertrophic tissue and of apthous ulcers, Its mechanism of action is: a) Thermodynamic action b) Activation of tissue enzymes c) Osmotic pressure d) Protein precipitate* e) neutralization 35. phenytoin (Dilantin) gingival hyperplasia 36. Which antibiotic is enough for dentistry Amoxycillin, Ampicillin 37. benzodiazepine its action can be antagonized by Flumazenil 38. barbiturates have the following properties sedation, excitement, general anesthesia, hypnosis; except analgesia 39. Commonest type of drug sensitivity is a) anaphylaxis b) serum sickness c) skin reaction* d) syncope e) vertigo 40. Premedication is used in GA a) easy induction* b) analgesia c) prolonged recovery d) prevent unnecessary irritation*

e) reduce basal metabolic rate* 41. What drugs has common depressive side effects a) antihistamine b) tranquilizers*? c) Non-barbituartes d) Anti-depressant e) Narcotics 42. Which drug group may cause respiratory depression: a) barbiturates (cardiac and respiratory depression) * b) tranquilizers c) sedatives hypnotics d) synthetic narcotic 43. Denture stomatitis treatment a) tetracycline b) penicillin c) anphotericin B* anti-fungal d) mycostatin* anti-fungal 44. Which of the following is correct about periapical infections? a) They gradually change from aerobic to anaerobic* b) Can be effectively controlled by amoxycillin? 45. Metronidazole: a) is effective in the management of AUG it interact with ethanol (alcohol) causing flushing palpitation, headache and naused 46. What is not associated with barbiturate a) sedation b) excitement c) analgesia* d) general anesthesia e) hypnosis 47. Dilantin Hyperplasia treatment: oral hygiene assessment, gingivectomy 48. The mode of action of drug may be defined as a) where the drug has its effect b) how it produces its action* (boucher, 192) c) what effect the drug has d) how the drug is distributed

49. Excretion of N2O is through the lungs 50. pharma placebo ( B 278) some participants in a drug study shows gain scores given when no therapeutic agents is administered 51. Chloropromazine ( B 200) belongs to which of the following groups of chemical compounds? a) catecholamines b) tricycline antidrepressant c) narcotic antagonist d) barbiturate e) phenothiazine* a major tranquilizer, is the prototype of the phenothiazine 52. Agents in mouth washes that have been shown to be at least partialy successful in inhibiting dental plaque formation include all of the following except: a) erythromycin b) amine fluoride c) benzethonium chloride d) sorbitol* slowly metabolized by oral bacteria e) urea peroxide note: sorbitol a sweetener that is commonly used in mouthrinse formulation, is only slowly methabolized by oral bacteria and therefore does not contribute to acidogenesis or plaque growth. It does not however inhibit plaque formation 53. What is true about benzodiazepines: a) It can be given safely in children* b) It causes after effects (hangover) c) It is a tranqulizer and gives analgesic 54. Antibiotics in minor surgery: a) if there is evidence that it can reduce infection* b) can prevent swellings after surgery c) amoxyl is satisfactory 55. The action of all barbiturate drugs (B 199) all except: a) sedation b) general anesthesia c) analgesic* d) excitement e) hypnosis note: the barbiturates, which are general CNS depressants, do not have an analgesic effect and may, in fact, sometimes augment and sometimes antagonize the action of concomitant analgesic

56. A patient recent prolonged steroid therapy may develop: a) relative adrenaline insufficiency 57. What drugs are expected to stain teeth while they are used using tooth formation a) tetracycline * b) corticosteriods Periodontitist 1. After prophylaxis, acidogenic oral flora turns into non-acidogenic flora in a. several hours* (3-8 hrs) b. 1 day c. a few days d. a week (note: it is not until 24 hours without cleaning that a clinically demonstrable layer of plaque has formed) 2. Which instrument is used to measure the periodontal pocket? a. probe b. sharp probe c. calibrated probe* 3. The Periodontal Index (Russell) is inappropriate for use with children primarily because: it measures gingivitis and advance periodontitis (YuP) Note: PI measures the prevalence and severity of periodontal disease 4. PMA (Papillary-marginal-attached) a system of epidemiological scoring of periodontal disease devised by Schour and Massler in which the symbols denote the areas involved in the gingival inflammation 5. PI (Plaque Index-Silness and Loe) records the presence of plaque 6. GI (Gingival Index- Silness and Loe) is a system for assessing the quality, severity and location of gingival disease 7. DI (DI-S Debris Index) determines amount of debris on the teeth 8. CI (CI-S Calculus Index) determines the extent of calculus on teeth that is observable visually 9. OHI S (Simplified Oral Hygiene Index Greene and vermillion) combination of debris and calculus index 10. PDI (Periodontal Disease Index- Ramfjord) measures the presence and extent of periodontal disease.

11. GPI (Gingival Periodontal Index OLeary et al.) evaluates the gingival status of the mouth 12. Gracy curette used for subgingival curretage, which of the following is correct? a. used in specific tooth surface* 13. curettes are instruments used for both supra and subgingival scaling and root planning 14. scaling is the procedure which aims at the removal of plaque and calculus from the tooth surface 15. For scaling the most effective angulation of curette is 70 degrees? 16. The most frequent sequela of gingivitis is: a. gingivosis b. periodontosis c. periodontitis* 17. In developing plaque the adhesive polymer produce streptococcus mutans is synthesized from a. glucose b. sucrose* c. fructose d. lactose 18. If periodontal pocket is 7.6 mm deep, what is the best treatment? a. scaling and polishing b. general treatment ( antibiotics, mouth washes etc.) c. Surgical treatment* 19. Which of the following is incorrect about a mucoperiosteal flap? c) carefully separate the mucosa from periosteum note: mucoperiosteal flap is a flap of mucosal tissue, including the periosteum, reflected from the bone. 20. In late teenage group (15-19 years old) the most frequent periodontal disease is: a. periodontitis b. gingivitis c. ANUG d. Periodontosis* juvenile perio?

21. Universal curette

a. one side only b. two sided* 22. Gracy curette used for specific area 23. root planning is the smoothening of the roughened root surfaces by the use of scalers and curettes 24. apical migration of the epithelial attachment with atrophy of marginal gingival result: a. false periodontal pocket b. periodontal recession* c. gingival cleft d. true pocket 25. CPITN Community periodontal Index treatment needs (used to assess periodontal disease in large population) 26. Which is determinine to periapical tissue:? a. H2O2 b. EDTA c. Alcohol d. Chlorhexidine* e. Eugenol 27. Periodontitist gram -, gram+ cocci to gram and spirochete 28. 2 days plaque neutrophils and the number of cocci is reduced 29. Plaque formation: -phase I, (1-24 hours) discrete colonies composed of 80 90% gram positive cocci and short rods appear -phaseII, (2-4 days) rods and filamentous microorganism appears and the number of cocci reduced -phaseIII (6 10 days) vibrios and spirochetes appear and the number of cocci is reduced and increase in size of gram negative anaerobic microognaisms 30. angle of the instrument in root planning 80 degrees 31. Root planning to check the success of calculus removal using cross calculus probe 32. Angular type of bone resorption can be seen more often in a. occusdal trauma b. food particles retention c. periodontosis is a condition in which there is non-inflammatory degeneration of the periodontium characterized by destruction of the

periodontal membrane fibers, alveolar bone resorption with ultimate loosening and possible migration of teeth. d. periodontitis inflammation of the supporting tissue of the teeth. Usually a progressively destructive change leading to loss of alveolar bone and part of the periodontal ligament. Marginal periodontitis develops as a sequela to chronic gingivitis e. all of the above* 33. Alveolar Bone alveolar bone proper and alveolar process 34. Free gingival margin after complete tooth eruption (0.5mm to 2 mm to the CEJ) 35. Epithelial attachment, the normal depth: 0.5 2mm and Junctional epithelium 0.25 1.35 mm 36. Morphology of periodontal fibers (. 25 mm width), shape hour glass, thin at the mid root section) developed from the follicle that surrounds tooth bud plus cementum a. collagen fibers* b. wavy fibers* c. striated fibers d. elastic fibers 37. Periodontium main function attach tooth to bone tissue of the jaw 38. What is the most important function of periodontal ligament: Keep teeth in the socket (protective, sensory, supportive) 39. Periodontal ligament width (normal) 0.2 mm, decreases with age but increase with infection 40. Appearance of periodontal ligament not in function: a. narrow* b. wide 41. Gingivitis or periodontitis in HIV patient: a. is painful from the beginning b. is pathognominic symptom of HIV disease* 42. What radiograph do you use for assessing periodontal condition: a. Periapical b. Bite wing c. Occlusal d. Panoramic* 43. Which of the following statements is correct for periodontal disease a. the finger pressure is enough for mobility diagnosis b. is a communicable disease?

c. x-ray after intra-alveolar surgery is sufficient for diagnosis of healing d. the systemic disease have no effect on it e. ZOE paste will accelerate healing following periodontal surgery 44. verical incision of mucoperiosteal flap whould be: a. always must extend to the alveolar mucoperiosteal* b. bisect the middle of gingivalpapillae c. must be right angle of the tooth 45. Apical migration of the epithelial attachment followed by atrophy of marginal gingival at the same level, result in: a. false periodontal pocket b. periodontal pocket recession* c. gingival cleft d. true pocket 46. periodontal pocket can be best detected by: a. bite wing radiographs b. sharp explorer c. study cast d. calibrated probe* 47. Most prominent feature of Acute Apical Periodontitis is: a. tenderness of tooth to pressure severe continous pain* b. extra-oral swelling chronic c. intermittent pain 48. How can a periodontal pocket be recognized? a. x-ray b. periodontal probe or calibrated probe* c. periodontal marker 49. chronic inflammatory periodontal disease originates in a. the marginal gingival* b. the crestal alveolar bone c. cervical cementum 50. Which is the most important local factor in the etiology of periodontal disease: a. occlusal trauma b. calculus c. brushing habits d. coarse food e. plaque* 51. Calculus attaches to teeth surface: a. acquired pellicle

b. interlocking to the crystals of the teeth* c. by penetration into enamel d. mechanical interlocking 52. Which states best the morphology of periodontal ligament fibers a. elastic b. striated c. non-striated d. tauty e. wavy* 53. Which differentiate an acute periodontal abcess from an acute periapical abcess? a. pigmentation of the gingival b. nature of the swelling c. response to vitality test* 54. the most common cause for gingival irritation is a. calculus b. plaque* 55. Which is the most common local factor in the etiology of periodontal disease: a. occlusal trauma b. calculus* c. brushing habits d. coarse food note: plaque 56. Plaque is considered infection because: a. antibiotic therapy prevents or stop its formation b. indication of bacterial activity* c. it is common to both animal and human 57. Virucidal solution is: a. na-hypochorite (miltons solution)* b. glutaraldehyde* c. guantoial ammonia 58. The most common place for initiation of gingivitis is a. interdental papillae b. the free gingival ridge c. the attached gingival d. marginal gingival* 59. the earliest clinical sign of gingivitis is? a) change in size b) change in contour

c) bleeding when palpated or probed* d) change in color e) changed in stippling Bleeding when Palpated or probed may occur before alteration in color or form, which are the next changes to occur and in that order 60. Healthy gingival few microorganisms, 90% gram + cocci and rods, and remaining gram (-) 61. Lindhe = 1) first phase increased microorganism and a shift to a more gram (-) than gram (+) day 2 2) second phase (day 3-4) fusobacterium and filamentous bacteria 3) third phase (5-9 days) spirilla and spirochetes 4) 7days gram + 50% of microflora 62. the least important in preventing periodontal disease a. brushing b. interdental cleaner c. Vit B8* d. Occlusal equilibration 63. the tissue response to oral hygiene instruction is best assessed by: a. reduced tendency to bleed on probing the gingival margin 64. rapidly progressive periodontitis: usually seen between the ages of 25-35 years. More commonly seen in sufferers of certain systemic disease 65. the end result of oral hygiene program the patient can use the toothbrush and dental floss* 66. periodontitis disease has high incidence in age group (adult) a. 15-25 b. 25-35 c. 35-45* 67. healing after pocket curretage by; a. long epithelial attachent? b. connective tissue 68. What is the characteristic features of gingivitis in AIDS patient? a. a red band on the free gingival with associated petichae. It is the most characteristic for AIDS but to make sure of diagnosis of other features:

b. correlating with other pathogenisis lesiosn of AIDS and does not resolve to periodontal conventional treatment c. severe pain 69. Characteristic feature of gingivitis in HIV ( AIDS) patient: i. erythema of free gingival, attached gingival and alveolar mucosa ii. extensive bleeding on brushing and even gentle probing iii. lack of response to conventional periodontal treatment iv. found in combination with other AIDS manifestation 70. restorative crown located within the gingival margin, produces inflammation: a. rarely b. sometimes* c. always?- because of plaque accumulation d. never 71. The gingival margin is below the cementoenamel junction, the pro measure 6 mm. The diagnosis is: a. True pocket* b. Gingival pocket c. Infrabony pocket Note: If the bottom of the pocket is near the CEJ, there is a pseudopocket, but if the pocket is associated with apical migration and bone loss. This is indicative of a true pocket Pathology 1. bilateral swelling of the mandible a) central giant cell children and young adults - F>M - Mandible 2/3 and maxilla 1/3 - Anterior part of the jaw - Pain not prominent feature - Expansion of cortical plates - Histo; multinucleated giant cells - Tx: curettage or surgical excision - Heals: with new bone and heal with no difficulty - Radiation is contraindicated - Unilateral swelling

b) Cherubism (bilateral) , autosomal dominant disorder Male>Female, 2years to puberty, self limiting disease, mostly

bilateral involving the mandibular angle, ascending ramus, retromolar region, posterior maxilla and coronoid process, painless symmetrical enlargement premature exfoliation of the primary teeth and displacement of the permanent dentition, intelligence not affected, alkaline phosphate elevated, multilocular radiographically c) Infantile Cortical Hyperostosis or Caffes Disease*, self limiting and short lived, 4 weeks of life, onset, , firm, tender swelling with rather deep seated edema, pain, fever and hyperirritability (inflammatory type) (bilateral) d) Fibrous Dysplasia (unilateral) 2. why streptococcus infection occurs because it has different strains 3. Target lesion erythema multiforme/ stevens johnsons syndrome - unknown etiology, precipitating agent (Herpes Simplex inf) - onset 1-3 weeks - young adults mostly - males> females - asymptomatic, vividly erythematous discrete macules, papules or occasionally vesicles and bullae distributed in a rather symmetrical pattern - commonly on hands, arms, feet, legs, face and neck - concentric ring like appearance - target, iris, bulls eye - mucous membrane involvement and oral cavity - Stevens Johnsons disease severe form , mucocutaneousocular 4. Which is not premalignant lesion? a) erythema migrans/ geographic tongue benign, unknown etiology, related to emotional stress b) erythroplakia yup, precancerous lesion 5. Biopsy is least useful in the following condition? a) geographic tongue* b) apthous ulceration? c) keratocyst d) giant cell granuloma e) myxoma 6. Bluish swelling ranula 7. Diagnosis of cyst and granuloma biopsy 8. Organism present in acute ulcerative gingivitis all correct

a) spirochete* ( borrelia vincentii) - more b) fusiform bacteria*- more c) others such as vibrios and cocci* 9. stomatitis nicotina/ pipesmokers palate most often seen in pipe smoker but can be seen in cigar or cigarette smokers as well 10. Incidence of malignancy of leukoplakia/ white patch related to tobacco/alcohol and others a) 0% b) 5-6%* c) 1-4% d) none of the above 11. In fact oral vesicles are most likely to be seen in a) herpes simplex infection* adultHSV1 or HSV2 circulating antibodies that resides dormantly within the regional ganglia, herpetic witlow (fingers), precipitating factors such as stress, some monthly others yearly, lesions are usually lesions are preceded by a tingling sensation or soreness where the lesion appeared., small lesions 1 mm or less in diameter, vesicles that ruptures 7-10 days and no scar, Lipschutz bodies in histo. Ativiral agent but not as a curative drug b) Oral lichenoid reaction c) Recurrent Apthous ulceration/stomatitis canker sores, streptococcus sanguis, F>M, 10-30 eyars old common, no vesicle formation like herpes, ulcers persist for 7-14 days (minor), 6 weeks major, Anitschkow cells,, no specific treatment, tetracycline mouthwash for 5-7 days, 250 mg per 5 ml, steroid ointment, chemical cautery d) Pemphigus vulgaris* - mouth first area of manifestation, serious chronic disease characterized by the appearance of vesicles and bullae, small or large fluid filled blisters, that develop in cycles, etiology unknown, 30 and above years old, males=females, nikolskys sign, tzancks cells 12. Temporo mandibular joint dysfunction a) associated with chronic minor illness* b) more men than women are affected (wrong F>M) c) stress pain d) extraction pain note: masticatory muscle spasm, can be initiated as a result of muscle over-extension ( high dental resto, maxi appliance), muscular overcontraction ( loss of posterior teeth) or muscle fatigue ( chronic oral habits), psycho-physiologic theory

4 cardinal signs and symptoms: 1) pain 2) muscle tenderness 3) clicking and popping noise 4) limitation in jaw movements 5) absence of organic changes in the joint 6) lack of tenderness in the joint Treatment: conservative, relief of emotional stress, correction of obvious faulty restorations and appliances, myotherapeutic exercise physiotherapy and drug therapy. 13. Most serious complication of abscess in canine cavernous sinus thrombosis 14. the most common malignant lesion of the oral cavity is? a) basal cell carcinoma b) squamous cell carcinoma* c) osteoma 15. Painless bluish lump (10mm) just inside then border of the vermillion of the lip is likely a) smoker kerathosis b) squamous cell carcinoma c) mucocelle* d) fibroma e) fibro-epitheliel polyl 16. Hyperplastic tissue formation after long time of immediate denture*all a) change the design b) Surgical interference c) Relieve the flanges d) Ask the patient to leave the denture out to rest the tissue 17. A patient with the diagnosis of pemphigus vulgaris, which one of the following is used to confirm the diagnosis? a) x-ray b) antibodies in the serum c) immmunoflorescence* ( great value in establishing the diagnosis of phemphigus when clinical and microsopic findings are inconclusive, presence of IgG and sometimes in combination with C3, IgA and IgM

18. Histologic picture of lichen planus is? all

Note: Lichen Planus, small angular flat topped papules only a few mm in diameter, In the early course of the disease the lesion appears as red, soon take on a reddish, purple hue, later a dirty brownish color. The center of the papule may be slightly umbilicated, Its surface is covered by characteristic, very fine grayish white lines, called wickham striae, can occur anywhere on the skin surface, bt usually are distributed in a bilaterally symmetrical pattern, unknown etiology. Usually seen in nervous, high strung person is invariably the one in whom the condition develops, Females>males, 40 70 years old, premalignant a) Parakerathosis, hyperorthokerathosis with thickening of the granular layer* b) Acanthosis with intracellular edema of the spinous cells in some instances* c) Saw tooth appearance of the rete pegs* d) Necrosis or liquefaction degenearation of the basal layer cells* e) Colloid bodies or civatte bodies* 19. Psoriasis auspitzs sign and silvery scale 20. patients with sulfur granules - actinomycosis 21. What is the most common congenitally missing tooth a) lower second molar b) upper second molar c) maxillary lateral incisor * d) lower incisors e) lower second premolar Regezzi: third molars> second premolars and maxillary lateral incisors 22. herpes will be transmitted by: direct contact? a) blood and saliva? b) Saliva and lip c) Blood and lip 23. Not a possible symptom of squamous cell carcinoma. a) wart b) non-healing ulcer c) induration d) elevated margins e) fungulation f) pain* (painless dapat) note: arises from epidermal keratinocytes, sunlight damage also, high potential to metatasize to regional lymph nodes, males, 6Th decade of life, asymptomatic in early course, lesions appear as verrucous growths, papules or plaques that can ulcerate later. Tx. Surgery and irridiation 24. Squamous cell carcinoma cannot in a shape of? a) ulcer

b) c) d) e)

raised level hard wart shape leukoplakia* can become cancerous (Squamous Cell C.)

25. Which one of the following is not concerned to have vesicle in the mouth? a) pemphigus vulgaris b) herpes simplex c) apthous stomatitis* 26. A patient following an extraction has an injection at the neck near the angle of the mandible, having sulfur granules. a) Actinomycosis* b) Agranulocytosis 27. Which is not included as a white lesion? a) fordyce granules* - heterotropic collections of sebaceous glands, small yellow spots b) Smoker keratosis, can become malignant c) Leukoedema, asymptomatic, symmetric in distribution and occurs in buccal mucosa, appears as a grey-white, diffuse, filmy or milky surface, can become wrinkled or corrugated. Cannot be remove by a tongue depressor or cloth, no treatment, no malignant potential, no etiology d) Lichen planus 28. What is the characteristic feature of basal cell carcinoma? a. blood metastasis b. does not erode bone c. intensive involvement* d. radioresistant note: arises from the basal cells of the skin, low metastatic potential, sun exposure (UV light) and light complexion, age 40, sun exposed skin, most common cancer of the head and neck , midface most common, Men>Female, slow growing and rarely metastatic, prognosis is good. Tx is surgical and irridiation 29. a patient with diagnosis of candida albican infection, which one of the following used to confirm diagnosis? a) microscopic smear* b) antibodies in the serum c) bacterial culture d) blood count e) serologic examination 30. What is the significance of erosive lichen planus? a. high malignant potential

b. some malignant potential* 31. A 30 years old male complain of painless swelling at the buccal mucosa. It has been present for about 6 months. He admits to playing with it. He is concerned because this might represent cancer. The base is narrow, most likely is : irritational fibroma Note: painless, broad based swelling that is lighter than the surrounding areas, lack of vascular channels 32. The eruptive cyst in an area of second molar in a 2 year old child, what you will do: d. observe* - no treatment e. excise 33. A 10 year old boy present with small grayish white lesion surrounded by a red halo on the soft palate and tonsillar pillars, small vesicles are found. The patient has fever and pain in the ear. Diagnosis is : Herpangia Note: acute viral infection (Coxsackkie type A virus) It is transmitted by contaminated saliva and occationally through contaminated feces. Children most common, malaise, fever dysphagia and sore throat following a short incubation period. Small vesicles on the soft palate, faucial pillars and tonsils, last less than a week, self limiting, tx not required. 34. Keratotic lesion surrounded by cold web like line wickhams striae ( appear as fine lace-like net on the buccal mucosa) Diagnosis is: Lichen Planus 35. A patient is complaining of an open sore on the buccal mucosa. The lesion is painless, ulcerated, has indurated margin, 1.5 cm in diameter, covered by grayishwhite exudates. Enlarged lymph nodes and tender. Negative tuberculin test. Positive serology : Chancre ( primary lesion of Syphilis) Note: Syphilis is a venereal disease. Caused by spirochete Treponema pallidum It is acquired through sexual contact with a partner with active lesions, by transfusion of infected blood or by transplacental inoculation from an infected mother. Primary lesion forms at the site of spirochete entry, with the subsequent development of painless, nonsuppurative regional lymphadenopathy. The chancre heals after several weeks without tratnment, leaving the patient with no apparent symptoms. After a latent period, secondary lesion (mucous patches) develops In this stage there is wide dessimination with fever, flu like symptomsand mucocutaneous lesions appearing with spirochetemia. Lyphadenopathy is typical and other organs may be involved. This resolves also on its own and the patient enters another latent period. Tertiary or late stage syphilis develops. These patients may have central nervous system involvement(Tabes dorsaleslocomotor ataxia), cardiovascular lesion, focal necrotic inflammatory lesion(gummas), vomer ( saddle nose), anterior bone growth(saber shin). Hutchinsons Triad 1) interstitial keratitis, 2) 8th nerve deafness and 3) dental abnormalities, hutchinsons incisors or mulberry molars (enamel organ). Definitive diagnosis 1)darkfield

microscopy 2) special silver stain of biopsy and 3)serological test of T. Pallidum. The treatment of choice in any stage is penicillin 36. An old male presents and complain of having numerous white lesion in the oral cavity within the past few days prior to this family physician prescribed chlortetracycline for an upper respiratory infection, the patient taking this antibiotic for past two weeks. Plaques on the lip mucosa, buccal mucosa and tongue: most likely to be Moniliasis or Oral Thrush or candidiasis 37. Characteristic of squamous cell carcinoma: a. an alcoholic who smokes* b. white skinned people basal cell carcinoma c. It reacts simply to radio therapy 38. How can you differentiate a benign epithelial tumor from a carcinomatous one: soft papillomatous mass, not indurated or not fixed ( freely movable) and pedunculated 39. Fibrous gingival hyperplasia may be associated with: reduced immune response? Cause: bacteria, plaque and calculus which can be potentiated by hormonal changes or drugs such as Dilantin./ Nifedipin , exaggerated response of the gingival fibroblast. Treatment: gingivoplasty or gingivectomy plus prophytlaxis and oral hygiene instructions. 40. Peg shaped teeth associated with anhydrotic ectodermal dysplasia* 41. A cyst at the apexof an upper central incisor measuring 1 cm in diameter is visualized in radiograph. You would expect to remove this lesion a) By extraction of the central incisor, expecting the cyst attached to the apical end b) By exteriorizing the cyst through the buccal bone and mucosa by marsupialization c) By making a mucoperiiosteal flap and removing the cyst through an opening made in the alveolar bone* d) By making a mucoperiosteal flap and making a larger opening through the socket to remove the cyst Note: cysts of this size are not indicated for marsupialization and are too large to be retrieved through the opening in the socket. An attempt for removal in this manner will create excessive loss of alveaolar bone and result in an irregual alveolar ridge

42. the common site of squamous cell carcinoma of the oral cavity a. tongue and floor of the mouth*

note: Tongue (25-40%), Lip (25-30%), Floor of the mouth (15-20%) Buccal mucosa and gingival (10%), and Palate (10-20%) 43. Repeated streptocoocal infection occur because : there are many different serological groups and types 44. Which of the following most common to reccur if not properly treated: Giant cell granuloma Note: bening tumor that present as solitary, radiolucent lesion of the mandible or maxilla, unknow etiology, children and young adults, Female>male, , painless expansion of the jaw, perforation is uncommon, multilocular or unilocular Rx and displaced or resorbed teeth roots, Tx, Surgical with curettage and removal of peripheral bone because of high recurrence rate 45. Hypodontia or Anodontia a. anhydrotic ectodermal dysplasia* b. Downs syndrome 46. Character of malignant lesion is: a. fixed* b. mobile 47. Sudden development of red wheal like distributed lesion on the oral mucosa, rapidly progress, thin walled vesicle : erythema multiforme Note: erythema multiforme unknown etiology but related to hypersensitivity reaction. , acute self-limiting process that affect the mucous membrane and skin, name is such because clinical appearance is varied (macules, papules, bullae and others), symptomatic treatment. 48. A patient worried about small spots on the side of the tongue. The most probabale to be is: a. Foliate papillae* b. Traumatic injury 49. Which of the following cysts cannot be diagnosed radiographically a. Nasolabial or nasoalveolar cyst* b. gobullomaxillary cyst note: nasoalveolar cyst are nonodontogenic cysts, seen only in the maxilla and cannot be seen only in the maxilla and cannot be seen routinely on radiographs

50. ANUG, it is communicable? Not communicable, affect those with low resistance Note: spirochete and fusiform bacteria, , pain , necrosis of the gingival margins, cratering of the interdental papillae and a marked bleeding tendency from the ulcerated

tissue subjacent to the necrosis and covered by a pseudomembrane. Age 15-30 , fever, lymphadenopathy and malaise with the gingival lesion. Difference with herpetic, anug ulcers in the gingival while herpetic is in the mucosa and the fever is prodromal or starts before lesion appears unlike anug together with fever. Tx, local debridement, diluted hydrogen peroxide rinses and cleaning (floss and brush soft). 51. Central Giant Cell granuloma can cause all except? a) multilocular radiographic appearance b) cotton wool radiographic appearance* c) resorption of teeth d) displacement of teeth e) expansion of the jaw bone with or without perforation of the cortical plate 52. Basal Cell carcinoma with cutaneous involvement 53. squamous cell carcinoma metastasis rapidly by blood, yup 54. Granuloma, chronic abscess, cyst, which one is used to confirm diagnosis? a) biopsy* b) x-ray c) vitality test 55. Gingival Cyst: a. does not contain crystals b. contains fluids* note: odontogenic origin, common in infants. Or called as Epsteins Pearl (palate midline not related to tooth forming tissues and Bohns nodule, along the alveolar ridges, related to salivary gland remnants, white or off-white broad based nodule approximately 2 mm in diameter. Tx non for infants will rupture for adults excision overlying epithelium. 56. Branchial Cleft Cyst or lymphoepithelial cyst: a. Is developmental in origin b. Was called a branchial cleft cyst c. Is treated by surgical removal d. Is often located at the anterior margins of the sternocleidomastiod muscle or lateral portion of the neck Note: all statements correct. It is related to incomplete obliteration of the branchial clefts that undergo cystic degeneration. , can enlarge when infected., treatment surgical excsion.

57. Staphylococcus can cause: a. syphilis

b. c. d. e. f.

scarlamia pneumonia* meningitis* septicemia* osteomylitis*

58. the best diagnosis for pemphigus: (Nikolsky sign) a. Tzancs cells b. histological immunoflorecence* 59. Intraoral vesicles are seen in which of the following lesions? a. pemphigoid b. herpetic stomatistis* c. lichen planus 60. Diagnosis in pemphigus- tzank cells 61. A patient complains intermittent swelling on the floor of the mouth. The diagnosis is: a) dermoid cyst b) sialadenitis* 62. Absence of teeth will cause a) prevent the development of alveolar bone* b) prevents the development of the mandible 63. Angular cheilitis in edentulous full denture patient is due to a) decrease vertical dimension* b) vit. B deficiency c) Iron deficiency 64. Recurrent herpes simplex is caused by herpes simplex virus* 65. Ameloblastoma is found most often in the a) hard palate b) soft palate c) anterior maxilla d) maxillary molar region e) mandibular molar region* note: in 81 percent of the cases the ameloblastoma is located in the mandible, while 19 % are found in the maxilla. In the mandible 70 % are located in the molar region 66. The most significant sign of malignant tunor in the mandible? a) Parastesia* b) Bleeding* c) pain*

d) fixed* 67. The most common carcinoma of the oral cavity? a) squamous cell carcinoma* b) basl cell carcinoma c) lymphosarcoma d) ameloblastoma 68. Which one of the following lesions is not included? a) fordyces granules* b) smokes keratosis c) leukoedema d) lichen planus e) chemical burns 69. Dentigerous cyst reduced enamel epithelium 70. What is not the characteristic for cancerous of mouth? a) Elevation* b) fixation c) invasion d) pain 71. Myxoma ( Benign intraosseous neoplasm) characterized by: - numerous small radiolucencies, displaced teeth - hypothyroidism - or hyperthyroidism? - Slow growing, asymptomatic submucosal mass - No gender predilection - Any age - Palate most commonly affected - Looks like a mucoceles, benign, surgical treatment 72. A bluish nodule 10 mm in diameter near the commissure of the lower ip, Diagnosis a) mucocelle* b) fibroma c) lipoma Note: Carcinoma malignant epithelial tumor Sarcomas malignant tumor of the mesencymal origin Oral Medicine 1. A patient has a fainting spells, the signs are ____face, weak pulse, very moist skin, shallow respiration a) 1 ml adrenaline subcutanously b) mouth to mouth respiration

c) nitroglycerine, sublingual d) recumbent position* Note: Syncope following a local anesthesia injection is best managed by placing the patient in the supine position and administering: Oxygen. Syncope is psychogenic in origin and usually controlled by a calm approach and use of supplemental oxygen. This is caused by decreased blood flow to the brain because of drop in blood pressure associated with vasodilation and an increase in the peripheral vascular bed. 2. A patient had kidney stones removal 2 years ago, a radioluscent area is discovered recently, diagnosis a) insufficient renal b) hyperparathyroidism* c) hypoparathyroidiam d) hyperthyroidism Note: Problem with calcium, Bone resorption, x-rays of the jaws in the patients with hyperparathyriodism may have a ground glass appearance like in fibrous dysplasia 3. Viral Hepatitis is known as Hepatitis A, Hepatitis B and Non-A, Non-B Hepatitis, which of the following is related to this statement? a) Hepatitis A and B are communicable and non A non B is not communicable b) Hepatitis A, B and non A nonB are communicable? c) Hepatitis A and B are not communicable, only non-A non-B is communicable 4. A patient with symptoms, kaposi sarcoma, oral candidosis, sore throat and severe recurrent herpes simplex, which one of the following is correct? a) AIDS* b) Hepatitis c) Agranulocytosis 5. Acute-onset diabetes is usually best controlled by: a) glucagons b) insulin* c) diet d) chlorpropamide (Diabinase) e) phenformin note: Almost all patients with acute-onset or juvenile diabetes require therapy with insulin for adequate control 6. Maturity diabetes is probably best controlled by: a) glucagons b) insulin c) diet*

d) chlorpropamide note: Maturity or adult onset diabetes is probably best managed by properly spaced, controlled diet. Oral hypoglycemia are indicated if controlled diet is inadequate; only about 20 per cent of adult-onset diabetes require insulin 7. Extraction of four teeth is required in a 65 year old diabetic female who has had her morning insulin. Preoperative instructions should include: a) do not eat or take any medication by mouth prior to surgery b) maintain normal diet* c) increase medications pre-operatively d) incresase sugar intake prior to surgery note: Patient should have maintain a normal diet to prevent hypoglycemia 8. An odor of acetone on ones breath should direct suspicion to: diabetes mellitus 9. Your patient states that he has recently become excessively thirsty, hungry and arises at night several times to urinate. His most probable diagnosis is: a) chronic renal failure b) diabetes mellitus* c) acute dehydration d) congestive heart failure e) none of the above note: preoperative urinalysis and blood sugar may reveal uncontrolled diabetes mellitus 10. A diabetic patient in the dental chair suddenly becomes agitated and his behavior changed, his skin and mouth are moist and had a full pulse, blood pressure is 150/80 his breath is shallow, which of the following is likely to help him? a) administer 02 b) place him in supine position c) temporary suspend treatment to allow recovery d) give glucose by mouth* e) give injection of insulin 11. Cardiac massage go per minute 12. Cardiac compression 60 times per minute 13. Discoloration of the skin and mucous membrane of the mouth addisons disease Note: characterized by 1) weakness, 2) pigmentation of the skin and mucous membranes 3) vomiting and 4) hypotension. Oral findings: macular areas of brown pigmentation. This disease is due to primary adrenal insufficiency. The pigmentation is supposedly caused by an increased production of the melanocyte-stimulatng hormone

14. An elderly patient with sudden attack of pain by stimulation of a trigger zone. Which one of the following is correct? a) trigeminal neuralgia* b) herpes zoster c) migraine d) migranous neuralgia 15. Parestheisa of the lower lip, we suspect a) metastasis of the malignant tumor in the mandible* b) fracture of the premolar area 16. A patient was blowed at the chin when he opens his mouth, it deviates to the left side. Diagnosis is? a) Fracture of the neck of condyle on the left* b) Fracture of the neck of the condyle on the right c) Fracture of the body of the mandible d) Near the angle of the mandible* e) At the mandibular symphysis f) In the maxillary antrum g) Anywhere in the jaw 17. The treatment of fibrous dysplasia: a) excision with the adjacent teeth involved b) incision, but the lesion is too big, limited excision because of cosmetic reason* c) radiation note: non-neoplastic, following a variable period of growth, fibrous dysplasia frequentl stabilizes or slows considerably after the onset of puberty. Small lesions may require no treatment. Large lesions that have cosmetic or functional deformity may be treated through a process of osseous recontouring. 18. Bleeding tendency usually occurs in a) cirrhosis of the liver* b) hypertension c) cardiac failure 19. The most reliable diagnosis of trigeminal neuralgia a) history* b) clinical examination 20. An 18 year old female had enamel erosion on the lingual surface of most of her teeth, subject to eating binges and sometimes consumed 5000 to 7000 Kcal/day, what might be her problem? Bulimia 21. When the patient is on steroid therapy, you send him to the physician for a clearance because: a) undermined adrenal cortex*

b) difficulty in healing c) to use an anesthesia with vasoconstriction note: steroids may cause suppression of the adrenal cortex. Preoperative steroid administration may be necessary to prevent adrenal crisis. 22. Patient with cancer associated syndrome a) Pierre robin syndrome = micrognathia, glossoptosis, and high arched or cleft palate b) Plummer Vinson syndrome* c) Turners syndrome= nope Turners syndrome, hypoplastic enamel in the permanent teeth due to an infection or trauma to the primary teeth d) Water syndrome = none Note: the only nutritional problem that has been associated with oral cancer is iron deficiency associated with Plummer-Vinson syndrome, which typically affects middleaged female. The syndrome components include a painful red tongue, mucosal atrophy, dysphagia, and a predisposition to the development of oral squamous cell carcinoma. 23. A patient following an extraction has an infection at the neck near the angle of the mandible, having sulfur granules a) Actinomycosis* b) agranulocytosis 24. Which one of the following is not concerned to have vesicles in the mouth a) pemphigus vulgaris b) herpes simplex c) apthous stomatitis* 25. Symptoms of Sjogrens syndrome - Serostomia, dry mouth - Rheumatoid arthritis - conjunctivitis, dry eyes note: dental tx, artificial saliva and artificial tears and scrupulous oral hygiene.

26. A 10 years old child with severe sore throat, swelling of the articulation of the legs: Diagnosis: Recheck a. rheumatoid arthritis b. rhematic fever? 27. Serum Sickness antibody mediated (humoral) immune reaction

28. Calcium and phosphorus in bone: OM (check) a. body supply them from bone whenever needs by which mechanisms* 29. Angioneurotic edema: swollen face, piffiness around the eyes, edema of upper lip with redness and dryness, caused by several deep silicate restoration in anterior teeth, no caries, negative thermal test, negative percussion, negative palapation, negative pain, negative rarefaction. Normal patients temparature. Clean bill of health

Medical Problems

1. Bilateral bone expansion in a child: cherubism* 2. Hutchinsons syndrome: a) enamel hypoplasia of anterior teeth b) mulberry molars 3. High sedimentation rate ? a) indication that there is an infection else where 4. a patient with brown discoloration, (macules) in the vestible: a) addisons disease 5. caf-au-lait spots can be seen in a) von recklinghausens (neurofibromatosis) b) Albrights syndrome ( polyostotic fibrous dysplasia) c) Men III (Mucosal Neuromas of Multiple Endocrine Neoplasia Syndrome, Type III) d) All of the above* 6. Patient complaining of paroxysmal pain behind left eye (watery eyes) , diagnosis is: cluster headache

7. The most common staphylococcus infection is: a) a localized purulent infection of the skin* b) a diffuse purulent infection of the skin c) staphylococcal osteomyelitis d) impetigo

8. staphylococcal infections produce: a) meningitis* b) septicemia* c) pneumonia* d) osteomylitis* 9. leukemic patient in the dental treatment, you are aware of: a) anemia b) infection 70% of px die of infection 10. A 15 year old white male presents with a complaint of swollen and bleeding gingivae for the past several days. On the morning of examination his mother became alarmed when she noted blood on his pillow and his mouth. He gave a history of generalized body pain, malaise, and recent wiegth loss. Examination revealed petechial hemorrhages on the flexor surfaces of both arms, ecchymotic areas on his left cheek and right lower lip, and bilateral cervical lymphadenopathy. Oral examination revealed soft spongy, tumor like proliferation of the gingival, superficial ulcerations of some of the enlarged gingival papillae, hemorrhage around some of the gingival papillae, hemorrhage around some of the gingival sulcus, loose clots in some of the interdental areas of the maxillary and mandibular premolars and dental calculi. The most probable diagnosis is: a) acute myelogenous leukemia* b) Dilantin gingival hyperplasia c) Necrotizing ulcerative gngivostomatitis d) Puberty gingivitis Note: leukemic cells tend to infiltrate into gingival tissue which causes gingival enlargement. The petechial hemorrhages and ecchymosis are due to thrombocytopenia because of suppression in the production of platetlets.A differential white blood count aids in a definitive diagnosis. 11. The primary syphilis lesion in oral cavity - chancres contain T. Pallidum 12. a 12 year old boy has a history of severe sore-throat, followed by migsatery arthralgia and swollen joints of the extremities. The history is suggestive of a) gout b) osteoarthritis c) serum sickness d) rheumatic fever? e) rheumatoid arthritis? 13. Which of the following is most appropriate initial treatment for a patient with HIV associated necrotizing ulcerative gingivitis? a) debridement and antimicrobial rinses* b) definitive root planning and curretage c) administration of antibiotics

d) gingivectomy and gingivoplasty 14. Black hairy tongue; certain structure become enlarged: a) proliferation of filliform papilla* b) proliferation of fungiform papilla 15. A young female patient presented with throbbing pain in theleft lower posterior jaw, with trismus and associated lymphadenopathy. Diagnosis is pericoronitis (with systemic and local signs) 16. A patient presents with a finger like fibrous ___ on buccal mucosa and constantly playing with it: irritational fibroma 17. A ten year old boy presents with small yellowish white lesions surrounded by a red halo, on the soft palate and fauceal pillars. Small vesicles are also found. The patient has fever and pain in the ear. Diagnosis: a) secondary herpetic stomatitis b) herpangina* 18. An ulcer with indurated margins, 1 cm in size covered by a yellowish white membreane. Tuberculin test negative, but with positive serology a) canker sore b) chancre* c) mucous patch d) carcinoma 19. In the cardiopulmonary resuscitation cylinders oxygen is better than mouth to mouth breath ( it increases the cardiac output by 30 ml) ? 20. Triangle of Trigeminal Neuralgia? a) Clinical examination* b) History* c) Loss of Function d) Trigger Zone* Note: In order to make a diagnosis of trigeminal neuralgia, the following must be present: a) a trigger zone, b) intermittent paroxysms of pain along one or more divisions of the trigeminal nerve and c) no abnormal neurologic finding except the tic. 21. In the dental chair the patient became cyanotic: - give oxygen till the arrival of an ambulance team 22. A patient gives a history of breathlessness on exercise, awakening from deep sleep due to shortness of breath. He cannot lie down in the dental chair for more than 1015 minutes before he must sit up to catch his breath. The most likely cause of this condition is:

a) congestive heart failure* 23. Branchial cleft cyst location is: side of neck, anterior border of strenocleidomastoid 24. Desquamative gingivitis: nikolsky sign - can be caused by mucous mebrane pemphigoid 25. Normal values RBC = 5.4+/- 0.8 x 1,000,000 cu mm (adult man ) RBC = 4.8+/- 0.6 x 1,000,000 cu mm (adult female) Hemoglobinometry is 16 +/- 2 g/100 ml (men) Hemoglobinomtry is 14 +/- 2 g/100 ml (female) Hematocrit range 37- 47 % women Hematocrit range 42 52 % men Bleeding time (Dukes method) 1-6 minutes Clotting time ( lee-white method) 4-12 minutes PTT = hemophiliac states, normal is 45 secs or less PT = liver problem, 30-40 secs Circulating leucocytes = 5000, 10,000 cells/ cu mm leucocytosis, infection occurring eosinophilic increase, parasitic or allergic reaction increase monocytes, tuberculosis etc

26. Sedimentation erythrocyte rate SER, is high: this suggest presenc of an infection somewhere in the body. As the activity of a chonic infectious disease increases, the speed of sedimentation also increases. 27. White blood cell count over 100,000 ? - increase eosinophile and neutrophil, most likely the patient is suffering from leukemia 28. Diabetic patient in your clinic suddenly becomes pale, wet moist and pale skin, shallow respiration, BP 150/80 give sugar 29. Epileptic patient with a scar in tongue 30. carcinoma of tongue characterized by asymptomatic at first but can become symptomatic especially when secondarily infected 31. Which lymph nodes are involved in carcinoma of the lower lip a) submental lymph nodes* note: usually affect the ipsilateral lymph node of the submental aor maxillary and if near the center can affect contralateral lymph node 32. Children born with cleft palat, micrognathia and glossoptosis

a) chiticdisease b) Treacher collins syndrome or mandibulofacial dysostosis Note: Bird like or fish like, varying hypoplasia of the mandible, maxilla, zygomatic process of the temporal bone and external and middle ear c) Pierre Robin syndrome*, micrognathia, mandibular hypoplasia, u-shaped cleft palate or high arched, glossoptosis 33. Least causes of xerostomia a) Sjogren Syndrome b) Submandibular sialolith c) Senility d) Emotional stress* 34. Poisoning with fluoride (vomiting) treated by: milk (calcium content) 35. Incidence of malignancy of leukoplakia a) 0% b) 5% c) 1-4%* d) none of the above 36. Which syndrome include: candidiasis, loss of weight, pneumonia and lymphadenitis- AIDS 37. Diabetic patients give glucose by mouth in case of hypoglycemia 38. You suspect cardiac arrest, which of the following statements incorrect: a) positive pressure respirations b) check if patient is really having cardiac arrest, eg pulse, consciousness etc. 39. A patient whose hands feel warm and moist is more likely to be suffering from which of the following consition? a) anxiety b) congenital cardiac failure c) thyrotoxicosis* d) metallic poisoning e) cachescia 40. Osteogenesis Imperfecta or Brittle Bone Disease a) Blue sclera* b) Deafness, conductive hearing loss* c) Osteoporosis* d) Bone fragility* e) Dentinogenesis imperfecta* blue, brown or amber discoloration of the teeth, primary more affected than permanent

41. Herpangina - acute viral infection caused by group A coxsackie virus, fever malaise, headache, vomiting and intraoral lesions 42. Main site of squamous cell carcinoma tongue and floor of the mouth 43. Lichen planus cob web appearance (Keratotic lesion) 44. Mucous pemphygoid affects the eye? 45. ranula cyst in the floor of the mouth 46. acute gingival hypertrophy 47. Hepa B + - sterilize, clean instruments then sterilize again 48. Trigeminal Nueralgia lancating pain, vital teeth, pain in posterior lower jaw 49. Bulimia erosion, tooth loss of upper anterioir due to rampant caries 50. Metastasis from the tongue through blood stream 51. Benign Epi. Neoplasm exopytic, pedunculated, relatively soft mass, freely movable 52. Basal cell carcinoma characterized by infitrative cutaneous involvement, yup I guess 53. chancre complaining of open soars or ulcers on B. Mucosa, painless, ulcerated 54. Down syndrome or Trisomy 21- mental retardation, congenital heart disease 3045% , Respi problems common, leukemia possible, tongue fissured, macroglossia, protruding tongue, periodontal disease common, eruption of primary and permanent delayed, , eruption sequence different, hypodontia, microdontia, macroglossia. 55. Fibrous Gingival Hyperplasia may be associated with depressed immune responses? 56. Anhydrotic Ectodermal Dysplasia Peg shaped teeth 57. In which condition is there dryness of skin, brittle nails and congenitally missing primary and permanent teeth a) erythema multiforme b) ectodermal dysplasia* c) cleidocranial dysostosis d) Pierre-Robin Syndrome

e) Osteogenesis Imperfecta Note:Ectodermal dysplasia affects tissues derived from ectoderm, including teeth, hair, nails and sweat glands. 58. Phemphigus Vulgaris not a subepithelial lesion 59. All of the following include vesicles except Apthous ulcer 60. Pagets disease Increase alkaline phosphatase 61. In the early stage of Pagets disease the patient is very healthy. The laboratory findings in the patients serum is a) normal ca level, normal PH level, normal alkalin PH b) High ca Level, high ph level, high alkaline ph c) Normal ca level, normal ph levels, high alkaline phosphotase* d) High ca level, high phh level, low alkaline ph levels 62. Hyperparathyroidism no brown pigmentation Note: increase level of parathromone and hypercalcemia. Etiology unknow but common in postmenopausal women. Early symptoms include fatigue, weakness, nausea, anorexia, polyuria, thirst, depression and constipation. Frequently bone pain and headaches are present. Stones (renal), resorption (bone), GIT (peptic ulcer) enurologic manifestation if high calcium in blood, Oral well-defined cystic radiolucencies of the jaw, which may be monolocular or multilocular, loosening of teeth, cortical thinning, partial loss of lamina dura, brown tumor nat skin pigmentation. Tx surgical 63. What is the typical features of Lichen planus a) sawtooth, rete pegs* ( is formed but not a typical feature) b) band of lymphocytes c) immunofluorescence of liquefied layer

64. Denture stomatitis is treated with a) amphotericin* b) tetracycline lozenges c) mycostatin* 65. Which drug is specific for trigemjnal neuralgia? a) diazepam b) carbamezepine (tegretol)* c) ergotamine d) phenytoin

66. The radiographic appearance of the initial stage of pagets disease is said to resemble : a) cotton wool in late stage b) ground glass* c) orange peel gingival d) beaten copper wire e) mosaic histology 67. Ameloblastoma in x-ray = soap bubbles 68. Mouth to mouth breathing - > 10-12 times 69. Which of the following is not a white lesion: a) Fordyce granules creamy yellow b) Leukoedema c) Lichen planus d) Chemical burns e) Smokers keratosis 70. Diagnosis of Oral Candidiasis is best confirmed by a) microscopic examination of smear* b) biopsy c) blood count 71. Most common malignant lesion of the oral cavities: a) ameloblastoma (bening odontogenic tumor) b) osteogenic sarcoma c) squamous cell carcinoma* 72. Ameloblastoma occurs most frequently a) near the angle of the mandible* b) in the maxilla c) at the mandibular symmphysis d) anywhere in the jaw 73. Treatment of fibrous dysplasia consists of a) resection of bone involved b) excision or, if large cosmetic reduction by limited surgery* c) irradiation d) curettage e) excision of lesion and removal of adjacent teeth 74. A patient with long standing rheumatoid arthritis and a history of steroid therapy, until a week ago, presents for multiple extractions. The dentist should consult the patients physician because the patient:

a) will be more susceptible to infection b) may have a suppressed adrenal cortex* c) will need haematological evaluation 75. A patient complains of headache, watery eyes, awake him at night cluster headache 76. A patient with Von Willebrands Disease (Platelet adhisiveness problem and factor 8 levels), should be treated with the same regime as for: a) congenital cardiac disease b) haemoplillia*, replacement of missing factors before exo c) rheumatic fever 77. An oral prodromal sign of rubella (german measles, terratogenic) is (petechi forcheimies spots?, rashes) a) Fordyse granules b) Kopliks spots sign of Rubeola c) Geographic tongue d) None of the above* 78. A patient 45 year-old awoke with swollen face, edema around his eyes and the upper lip when he went to bed the swelling disappeared. Examination reveals several deep silicate cement fillings on the anterior teeth. The suspected diagnosis is: a) angioneurotic edema* b) acute apical abscess 79. A patient has a swelling on his upper lip, corner of the nose and under his eye, fluctuant soft tissue under his lip, high temperatures. The first thing to do after taking the patients history is: a) take radiograph and test the related teeth for vitality* b) refer the patient to a physician 80. Normal BP for a patient 35 years old is? 130/80 81. Delayed eruption of teeth, most likely occur a) Rickets, Vit D deficiency* b) Cretinism* c) Cleidocranial dysplasia* 82. An adult with history of bacterial endocarditis requires prophylactic administration of an antibiotic prior to removal of teeth. Indicate the pre-operative regimen you prefer; a) amoxicillin 500 mg 1 hr before treatment( before they used to give gm but now a days 3 grams 1 hr b4 tx)*

b) penicillin V 250 mg orally 6 hours for 1 day before tx c) tatrecycline 250 mg-500 mg orally 2 hours before tx. 83. A 4 years old girl complaining of sore mouth, the patient has painful cervical lymphadenitis and a temperature of 39 degrees. Oral examination shows numerous yellow gray lesion with re margins on the palate, tongue and gingival. What is the most likely to be: a) measles b) erythema multiforme c) herpetic gingivostomatitis* d) stevens-joohnson syndrome 84. The causative microorganism of Herpetic gingivostomatitis is a) herpes simplex bacterium b) herpes simplex virus* c) herpes zoster virus d) borellia vincentii 85. Squamous cell carcinoma of the lower lip, causes swelling a) submental lymph nodes* b) submandibular lymph nodes 86. Carcinoma of tongue is characterized by: pain and pain in swallowing if severe usually asymptomatic 87. A lesion not found in epithelium (intraepithelim) a) erythema multiforme b) lichen planus c) pemphygus vulgaris*, nikolsky sign d) herpetic stomatitis 88. The severe complication in the spread of infection of the maxillary canine: a) cavernous sinus thrombosis* b) cellulites c) damage of the orbital nerve 89. After treatment by radiation, all the following can happen except a) dry mouth b) caries c) increase saliva* d) osteoradionecrosis 90. A boy complains of itching sensation on the upper lip. He says it occurred 3-4 times a year. Which one of the following is correct: a) recurrent herpes simplex* b) recurrent apthous stomatitis

91. Rapid swelling of the parotid gland with pain in the adult patient a) malignancy b) sialoadenitis* c) sialolithiasis 92. Sjorens Syndrome is characterized by all except: a) oral ulceration* b) xerostomia c) dry eyes d) arthritis 93. Intermitent unilateral swelling in the floor of the mouth a) herpes simplex b) chronic abscess periapical* c) sialolith* d) dermoid cyst?- not intermittent, swelling floor of mouth 94. If the following, the finding most constantly present in systemic infection is? a) involvement of regional lymph nodes b) formation of abscess c) cellulites d) fever* 95. Loss of sensation in lower lip may be produced by a) Bells palsy*, muscular paralysis, drooping of corner of mouth, watering of eyes, winking of eyes b) Metastatic malignancy? c) Trigeminal Neuralgia d) Fracture 96. Oral mucosa and _____ pigmentation occurs in patient with: a) Adissons disease b) Multiple myeloma c) Diabetes mellitus 97. In the pathological process called osteoradionecrosis, which of the following factors involved? a) radiation, infection and medication b) radiation, trauma infection* c) radiation, infection and drug side effects d) allergy, bacterial toxins and trauma 98. Treatment of osteoradionecrosis is? a) antibiotic therapy b) conservative teatment, antibiotic and sequestrectomy and resection of Jaw segment* c) conservative therapy

99. A diabetic patient in the dental chair suddenly becomes agitated and her bahavior changed, her skin and mouth are moist and and a full pulse, blood pressure is 150/80. His breathing is shallow, which of the following is likely to help him? a) administer O2 b) place him in supine position c) temporary suspend treatment to allow recovery d) give glucose by mouth* e) give injection of insulin 100. Discoloration of the skin and mucous membrane of mouth Addisons disease 101. Painful salivary gland are most likely to be indicative of a) mucocele b) mumps* c) sjogrens syndrome A patient with acetone odour of the breath should be suspected of suffering heart disease liver damage diabetes*

102. from a) b) c)

103. Approximate incidence of transformation of oral leukoplakia to squamous cell carcinoma is a) 10% b) 2% c) 1-5% approximately 4%* d) 28% e) 100%

104. a) b) c) d) e)

which of the following is the most common oral malignancy ameloblastoma osteogenic sarcoma basal cell carcinoma squamous cell carcinoma* mixed salivary gland tumor

105. patient has weak pulse, cold hands, cyanosis, immediate or emergency management a) trandelenberg position* b) adrenaline

c) call a physiciam d) administer glucose 106. except a) b) c) d) e) All of the following are considered premalignant lesions of oral cavity erythema migrans* smokers keratosis chronic hyperplastic candidosis speckled leukoplakia erythroplakia

107. During routine dental treatment your patient becomes lightheaded, diaphoretic and unconscious, diagnosis is? a) Syncope* b) acute allergic reaction 108. a) b) c) d) 109. a) b) c) d) 110. Which is not a sign of squamous cell carcinoma? ulceration hardness elevation pain* Sjogrens syndrome (more than one) oral ulcers xerostomia* dry eyes* arthritis*

Cheilitis, the most likely cause is: a) vitamin B deficiency* b) trauma* c) allergic or toxic reaction* d) a closed vertical dimension* e) improper balance of occlusion note: Cheilosis- Vitamin B deficiency

111. a 6 year old child who had a history of primary heres simplex got a recurrent infection. What is the likely cause: recurrent herpes because of decrease inmmune response. 112. a) b) c) d) Acute pyogenic bacterial injection may result in busopenia yustopenia beucomytosis*? bimphocytosis*?

113. Prophylactic administration of antibiotic is indicated in patient before oral surgery with a) herpes simplex b) whooping cough c) bacterial endocardities* 114. Premedication is given preceding general anesthesia for the following reason (more than one) ? a) allay apprehension and induce degree of amnesia preceding anesthetic* b) depress reflex irritability c) lessen metabolic activity 115. a) b) c) d) e) 116. 117. a) b) c) d) e) 118. Biopsy is least useful in which of the following lesions geographic tongue* apthous ulcers keratocyst giant cell reparative granuloma myxoma erythema multiforme - target lesion (iris or wheal lesion) squamous cell carcinoma cannot be in a shape of ulcer raised level hard wart shape leukoplakia*

Patient who faints,blands, slow pulse. The treatment should be: a) place the patient inrecumbent position* b) give sugar to patient

119. a patient following an extraction has an injection at the neck near the angle of the mandible, having sulfur granules a) actinomycosis* b) agranulocytosis 120. Which one of the following is not concerned to have vesicle in the mouth a) pemphygus vulgaris b) herpes simplex c) apthous stomatitis* A patient with fracture of maxillary facial, the first consideration is: a) preservation of airway*

121.

b) stopping the flow of cerebral fluid c) stopping the bleeding 122. Recurrent Herpes Simplex is caused by a) virus herpes simplex* b) spirochetes c) fusobacterium

123. An old female diabetic presents to you with white patches on tongue, using tetracycline for the past weeks for strep throat Diagnosis is a) leukoplakia b) moniliasis* 124. a) b) c) d) Characteristic of squamous cell carcinoma: an alcoholic who smokes* white skinned people it react favorably to radio therapy rare in australia

125. How can you differentiate a benign epithelial lesion from a carcinomatous one? soft papillomatous mass, not indurated, freely movable and pedunculated 126. What is the characteristic feature of gingivitis on AIDS patient a) A red band on the free gingival with associated platlet b) Correlating with other pathogenic lesions of Aids

127.

What is the characteristic feature of basal cell carcinoma? a) blood metastasis - rare b) does not erode bone - wrong c) cutaneaous involvement* d) radioresistant - wrong note; most common cancer of head and neck, difference with squamous C.C. is its low metastasis potential 128. A patients kidney stone had been removed 2 years ago. A radiolucent area has been discovered in the jaw recently. Diagnosis a) insufficient renal? b) hyperparathyroidism c) hypoparatthyroidism d) hyperthyroidism 129. 130. Herpangina is caused by:coxackie virus (Type 2, 3, 4, 5, 8, 9, 10) Vesicle in upper lip preceded by itching for the 3rd time is a) impetigo b) herpes simplex* c) herpes zoster

131. a) b) c) d) e) 132. a) b) c) d)

Diagnostic of pemphigus vulgaris tzank cell test dose of corticosteriod test of antibody titre histologic immunofluoresence examination* neurologic test for auto-antibodies Leukoplaquia maliginaw in 0% 2% 5%* 28% Oral Surgery

1. Which of the following are used as an antiviral agents a) glutaraldehyde* b) sodium hydrocloride (Miltons solution)* c) iodine in 70% alcohol* d) chlorhexidine e) iodoform 2. A patient with fracture of maxillo-facial, the first thing to do is? a) preservation of the airway* b) stop the flow of the cerebral fluid c) stop the bleeding 3. What is the mode of action of autoclaving?moist heat sterilization protein denaturation* 4. The autoclave is properly operated to sterilize at a temperature of 121oC (250oF) for 15 minutes 5. Dry heat sterilization kills microorganism by a) oxidation of fatty acids in the membrane b) hydrolysis of DNA c) denaturation of protein* d) hydrolysis of cellwall polymers 6. Ethylene oxides kill microorganisms by inactivating protein by forming stable epoxide bridges that are irreversible

7. Ultraviolet is only effective in surfaces because of poor penetrating ability. 8. Trichloroacetic acid used for chemical cautery of hypertrophic tissue of apthous ulcers. Its mechanism of action is? i. PPT (precipitation) of proteins* ii. Thermodynamic action 9. Which of the following is true about disinfectant? a. It destroys all pathogenic microorganisms b. It reduces the number of microorganisms to a non-infective level* 10. What do you do with instruments after treating a patient with Hepatitis B? a. soak them in hypochlorite solution b. sterilize, scrub and resterilize* c. handle them with two pairs of household rubber gloves d. scrub them with surgical iodine solution 11. Which of the following needs to be sterilized: a. all items touching blood b. all items used intraorally ( diseases can be transmitted via blood and saliva)* 12. autoclaves: denaturation of protein cell membrane 13. Extraction of lower 5s a) rotational* b) buccal, lingual c) direct fraction d) buccal 14. marsupialization is associated with treatment for? a) pericoronities b) cyst* 15. When managing a large cyst of the jaws, marsupialization is the tratement that has occasionally been used. Which of the following statements concerning marsupialization is correct? a) it avoids exposure of the teeth apices adjacent to cyst b) it avoids trauma to the IAN c) it establishes drainage, which eliminate pressure on the cyst wall d) it allows the cyst to become smaller so that its ultimate removal avoids easy perforation into the nose or sinus ( if the maxilla is involved e) all of these are correct*, it is not however the treatment of choice for most cyst.

16. Lymph from lower lip will drain into the: submental lymph node 17. Periapical infection of the maxillary second lateral incisor could point to the; palate 18. Residual cyst found: in the socket after extraction 19. The initial movement to extract a tooth by a forceps: a. Apically* b. buccally c. lingually 20. Which of the following accounts for the main cause of failure of replanted tooth a) ankylosis b) infection c) pulppal pathosis d) external root resorption* e) internal resorption 21. A dentist surgically removes a maxillary 1st molar, fracturing a large portion of the palatal root. The root is forced into the antrum and cannot be removed. Adjacent teeth and bone are normal. The appropriate approach to recover this root is through the a) hand palate in the canine area b) maxillary incisive fossa medial to the canine c) maxillary canine fossa above the level of the premolar roots* d) nasoantral wall above the middle concha or turbinates e) first molar alveolar by enlarging the antral opening

22. A patient complaining of chronic oro-antral fistula for long time ago, because of 6 extraction. What are the treatment? a. surgical flap? b. give antibiotics and nasal drops* c. wash the antrum 23. After 12 weeks, an oroantral fistula doesnt close, the treatment is: a. excision of fistula and surgery b. decongestant and antibiotics* c. intraoral autrostomy and surgery note: the first aim of treatment is to eliminate any existing infection. Washing by saline, well filling acrylic base plate

24. During extraction of maxillary 3rd molar, the tuberosity is fractured. It remains in place and attached to mucoperosteum, what should be employed? a. leave the tuberosity in place and stabilize if necessary 25. Complications of canine space infections include; a. respiratory paralysis resulting froma cute edema of the pharynx b. thrombosis of the internal jugular vein c. erosion of the internal carotid artery d. erosion of the ascending pharyngeal artery e. cavernous sinus thrombosis* 26. When down-fracture Leforte osteotomy is performed i. necrosis is prevented by blood supply to maxilla from palatine arteries ii. a patient with cleft palate is more likely to develop necrosis 27. A patient presentswith a radiolucensy, 1 cm in size at the apex of non-vital maxillary right central incisor. Which of the following would you do? a. Marsupialization b. Extraction and removal of cyst then the socket c. Mucoperiiosteal flap over labial bone, remove ther cyst, foloowing endodontic of 11 28. Marsupialization is used in the treatment of: a) cyst* b) abscess c) cellulites d) impacted molar 29. Cavernous sinus thrombosis thru the ophthalmic vein

30. What are the features of autotransplantation a. healthy PDL forming on root surface b. external resorption c. haphazardly arranged collagen fibers 31. Ankyloglossia is cause by a. edentulous ridge b. short lingual frenum* c. short labial frenum 32. Complication of thrombocytopenic purpura (suppression in the production of the platelests) during surgery is a. edema

b. haemorrhage* c. acute infection 33. Granuloma, abscess and cyst can be most accurately differentiated by means of a. x-ray b. electrical pulp test c. biopsy* d. thermal test e. percussion 34. Plasma is blood minus formed elements 35. Serum is blood plasma lacking certain components, especially fibrinogen 36. In the early (4-7 days) lesion you find: OM i. lymphocytes ii. UMC 37. A patient presents to you with a history of lancinating pain in the lower right posterior region. Patient insisting to extract his lower teeth. The right side of his face is unshaved and the teeth in question are vital and out of pathology. Your diagnosis is: a. Odontalgia b. Referred pain c. Trigeminal neuralgia* 38. All of the following are requirements for an adequate mucoperiosteal flap excapt: a. base wider than free margin b. mucous membrane carefully separated from periosteum* c. Base containing blood supply Note: should be with periosteum 39. Multiple Sinuses in chronic inflammation 40. Dry socket can be caused by: a) inability to form a blood clot in an extraction socket b) inability to keep a blood clot in an extraction socket c) infection postoperatively d) b and c only e) a, b and c* note: a dry socket is seen more commonly in the mandible than in the maxilla, and patients present with a throbbing, painful area 41. Treatment of Dry Socket is to alleviate pain

42. Following surgical removal of impacted 3rd molar in the mandibular bone: cold application from outside 43. A young female patient presents with throbbing pain in the left lower posterior jaw with trismus and associated lymphadenopathy. Diagnosis: Pericoronitis 44. Which of the following are not supplied by the mandibular division of trigeminal: a. anterior part of digastric* b. masseter muscle* c. buccinator* 45. The mandibular division of the trigeminal (V) nerve: a) exits from the skull through the foramen ovale b) contains both afferent and efferent nerve fibers c) supplies the muscle of mastication as well as the mylohyoid uscle and the anterior belly of the digastric muscle, among others d) enters the mandible through the mandibular foramen and has an exit at the mental foramen e) all of the above are correct* note: only the mandibular division of (V) contains motor fibers. 46. The argument that favours lingual split-bone technique for the removal of impacted mandibular third molar is that a. no neurological compications can occur b. most impacted 3rd molars inclined towards the lingual c. incision is easy to suture d. bleeding from highly vascular lingual tissue helps maintain the blood clot e. lingual bone is usually thinner than buccal* 47. A patient present with a radiolucency ( cyst) 1 cm in size at the apex of non-vital maxillary Rt central incisor. Which of the following would you do? a. marsupialization b. extraction and removal of cyst through the socket c. mucoperiosteal flap over the labial bone, remove cyst for endodontic treatment* 48. What are the features of autotransplantation? a. healing PDL forming on root surface b. external resorption c. haphazardly arranged collagen fibers* 49. Replantation, we expect ( bad prognosis) * external resorption, apical rarefaction 50. A very quick and wide tooth separation will result in

a. b. c. d.

gingival inflammation vasodilation wide space necrosis of PD-L& bone

51. least possible cause of post operative bleeding a. aspirin b. antibiotic c. codeine* d. poor technique 52. the most important factor in removal of impacted teeth: b. design of the flap c. removal of enough bone to expose the teeth d. pre-operative accessment of the x-ray* 53. Post-OP swelling give cold compress extraorally 54. First movement in extraction apical direction 55. Apical cyst of 1 cm flap mucoperiosteal- apicoectomy 56. Which one of the following is not a vaid indication for apicoectomy? a) broken instrument in the apical third of the canal b) presence of a fistula* c) perforation in apical third d) periapically involved teeth in patients with insufficient time for conventional endodontic treatment e) curve within apical third of the root that cannot be negotiated. Note removal of cyst can be either total enucleation or marsupialization 57. requirements for adequate flap mucous membrane plus mucoperiosteal 58. Residual cyst has epithelial lining 59. Electric cautery refers to a monoterminal electronic instrument that is capable of producing deep tissue dehydration and mass coagulation necrosis. 60. cause of attachment of tissue on the plates of the electrocautery machine too low current 61. Irritational fibroma painless growth 62. After auto-transplant you expect after 12months lamina dura formed, healthy functional PDL

63. Which of the following is true about disinfectant solution: a. It destroys all pathogenic microorganism b. It reduces number of microoganism to a non-infective level* c. It destroys pathogenic microorganism but not spores* 64. A patient with the fracture of maxilofacial, the first consideration a) preservation of airway* b) stop the flow of cerebral fluid c) stop the bleeding 65. The most potent viricidal properties: a. Miltons solution (NAOCL) b. sodium hypochlorite* c. belaching solution d. chlorhexidine e. glutaraldehyde f. alcohol 70% g. quarternary amonium 66. Antibioitics should be used routinely to prevent infection arising from oral surgery in patients suffering from all the following except: a. agranulocytosis b. severe uncontrolled diabetes c. aplastic anaemia d. mumps* 67. At what rate is closed chest cardiac compression given in an adult:? a. 12 times per minute b. 24 times per minute c. 60 times per minute* 68. Which type of cells does an abscess contain: a. mast cells b. PMN* c. Eosinophil d. Epithelial cells 69. In very cold climate which of the following fluoride concentration in water supply suspected to induce teeth flouridosis: a. 1.2 ppm b. 2.4 PPM * 70. In a 10 year old permanent mandibular molar. What is the effect? a. teeth adjacent to the extracted b. teeth in both arches on the same side

c. the remaining teeth in the mouth* d. teeth directedly opposite the extracted e. teeth in the same quadrant 71. The likely cause of xerostomia is: a. sjogrens suyndrome b. emotional reaction c. senility d. antidepressant drugs e. submandibular sialolith* Restorative Materials 1. GB Blacks Classsification? Boucher page 371 Class I Cavities in the structural defects of the teeth; pits and fissures in the occlusal surfaces of the premolars and molars, in the lingual surfaces of the upper incisors, in the occlusal 2/3s of the buccal surfaces of the molars, and the lingual surfaces of the upper incisors Class II Cavities in the proximal surfaces of premolars and molars Class III Cavities in the proximal surfaces of the incisors and canines that do not involve the incisal plane Class IV Cavities in the proximal surfaces of the incisors and canines that involves the incisal angle Class V Cavities in the gingival third, not pit caries of the facial or the lingual surfaces of the teeth Class VI Cavities on the incisal edges and smooth surfaces of the teeth above the height of contour 2. A newly placed restoration interferes with occlusion. What will be the periodontal response: thickening of the periodontal membrane 3. What would be the first stage of periodontal respond of premature contact of filling? Widening of periodontal ligament space 4. In Class II restoration, all of the following considered to occur as probable causes of periodontal problems except: a) flat ridge b) faulty or not proper contour c) not properly polished restoration d) cervical wall is too deeply apically e) overextention rest-lining in the cavity* 5. Depth of cavity beyond dentino-enamel junction minimum depth is 0.1 0.2 mm beyond the dentino enamel junction

a) b) c) d)

0.5 1.5 2.0 0.2 *

6. In a class II restoration, periodontal problems are more probable to occur when: a) ridge flat b) faulty or not proper contact* c) not properly polished d) cervical wall is too deep apically e) over extention vestibulo-lingual in the cavity 7. What is the minimum distance between 2 pits to connect each other? - sturdevant= When 2 pits and fissure cavities have less than 0.5 mm of sound tooth structure between them, they should be joined to eliminate a weak enamel wall between them 8. When you are preparing Class II, the cavo surface angle in the proximal box should be: a) sharp b) obtuse c) right* 9. Pulpal wall in preparing class II in lower first bicuspid parallel to the occlusal (surface) plane 10. A new restoration is left with premature contact. That might cause: a) Pulpal abscess b) Pulpal necrosis c) Apical cemental opposition d) Apical periodontitis* 11. Restorative crown locate within the gingival margin, produces inflammation: a) rarely b) sometimes? c) always d) never 12. Unsupported enamel should be a) reduced in height b) protected by the restoration c) finished at 90 degrees to the tooth surface d) removed* (because it is weak) 13. The design of occlusal anatomy in restoration is dictated primarily by: a) functional factors* b) depth of the restoration in the tooth

c) Patient feeling discomfort while biting check contact areas or overfilled resto 14. A Class II amalgam restoration, after carving, should a) reproduce the sluiceways in the region of the marginal ridge* b) be just out of occlusion with the opposing tooth c) have its marginal ridge at the same level as that of the adjacent tooth* d) reproduce the anatomical form of the tooth as nearly as possible* note: The restoration should restore function; if there is no occlusal contact, teeth will move 15. Resistance is obtained by parallel preparation of walls (wrong) 16. Resistance form is defined as that shape and placement of the cavity walls that best enable both the restoration and the tooth to withstand, without fracture, masticatory forces delivered principally in the long axis of the tooth. 17. Primary retention form is that shape or form of the prepared cavity that resists displacement or removal of the restoration from tipping or lifting forces. 18. Restoration with wrong occlusion what can happen first? a) abscess b) cementosis of the tooth c) acute periodontitis* d) gangrene of the tooth 19. patient has throbbing pain on heat or cold or percussion a) neurosis b) advanced pulpitis* c) gangrene d) hyperaemia 20. Hyperemia you treat it by a) pulpectomy b) pulpotomy c) remove partial caries + ZnO-Eugenol cement* 21. Tooth separation is used in class III lesion to a) assist in convenience form* b) assist in outline form c) for tight contacts 22. Wedges may be used to: a) protect the gingival during hand or rotary instrumentation b) act as a quick separating agent c) stabilize band material placed between teeth to prevent abrading the proximal surface adjacent to the one being treated

d) all of the above are correct* 23. Class V retention sight undercut on the occlusal and gingival wall, retention grooves is placed with a bur along gingivoaxial and incisoaxial line angle 0.2 mm inside the dentinoenamel junction and 0.25 mm deep 24. Which controls the outline form of the cavity extent of the cavity, grooves and pits for extent for prevention 25. Which may be caused by a newly placed restoration which interferes with occlusion e) apical abcess f) pulpal necrosis g) apical periodontitis* 26. When you prepare ClassII, The depth should be 9 below the carious lesion and sightly out of contact a) below the DEJ by 2mm b) just below the DEJ c) crown, 0.5-0.6 mm dentinal wall, enamel 0.2-0.3mm* d) root, .75-.8mm in the dentinal wall* 27. When you prepare the tooth for crown, you found brown line or spot on the DEJ. You have to a) remove it* b) leave it c) line with dycal 28. A matrix band used in packing a Class II amalgam restoration should a) be higher than the occlusal surface of the tooth, to allow overpacking of the amalgam b) support the marginal ridge of the restoration during packing c) fit the cervical margin of the box tightly d) provide a physiological contour for the proximal surface of the restoration e) all are correct* 29. Why do you use the matrix during placing amalgam on class II prepared cavity a) for retention of the walls or act as retaining wall b) contouring and shaping c) prevent the rubber dam to be incorporated with the restoration d) prevent saliva or blood from seeping into the restoration e) all of the above are correct* 30. Why is it hard to fit a matrix in upper first premolar to restore OM cavity, because of a) the canine fossa b) mesial concavity of the crown*

31. During cavity preparation in the lower second molar, which pulp horn is easy to expose? a) mesio-buccal*1 b) mesio-lingual 2 c) disto-buccal 3 d) disto-lingual4 e) distal horn 5 32. What is the biggest possibility to expose a pulp horn? a) mesio buccal of the lower first molar? b) mesio lingual of the lower first molar c) mesio buccal of upper first molar? d) distobuccal of upper first molar 33. Which of the following may be caused by a newly placed restoration which interferes with the occlusion? a) apical abscess b) pulpal necrosis c) apical periodontitis* 34. Why is it important to perform the original occlusal anatomical form of the restoration? a) to obtain the functional factors* b) should establish five anatomical structures c) according to inclined phase between opposing teeth 35. The design of occlusal anatomy of restoration is dictated primarily by? a) functional factors* b) the depth of the restoration in the tooth c) the necessity to restore detailed anatomy d) to control food flow in mastication e) available space 36. To prevent proximal dislodgement of class II restoration we must establish: a) adequate pulpal depth b) occlusal dovetail* c) parallelism of surrounding walls d) buccal and lingual grooves 37. A shallow Class I cavity is prepared and restored only with amalgam, patient comes back in 8 days with occasional sensitivity, What would you do? a) Leave it alone and tell patient pain will disappear within a few days b) Lining and replace restoration* c) Relieve high points

38. One week after insertion of Class II restoration the patient presents with a complainint of tenderness on mastication and bleeding fro the gingival. The dentist should initially: a) check the occlusion b) check the contact area* c) consider the probability of a hyperemia d) explain to the patient that the retainer irritated the surrounding soft tissues and prescribe an analgesic and warm oral rinse note: The most probabale cause of the symptoms would be a faulty contact area. If asymptomatic for a week the wedging of the food particles interproximally, causing inflammation of the innterproximal tissues, is probably the prime cause. More than a day is sually necessary for the impaction of food particles to result in the outlined symptoms. If the patient called shortly following placement one would suspect a high spot which would cause tenderness to percussion. If the patient complained of sensitivity to cold, then a hyperemia would be considered. 39. Restoration of a cusp using dental amalgam requires that? a) all the enamel be removed from the cusp to provide bulk of amalgam b) only the enamel be removed to conserve tooth structure c) atleast 2 mm of the cusp be removed to provide retention form d) atleast 2 mm of ther cusp be removed to provide resistance form* e) a reverse bevel be provided on the cusp to provide retention form 40. The fracture of amalgam at isthmus of Class II because of: a) the pulpoaxial line angle not beveled* b) not box shaped 41. The physical properties of which of the following materials are weakened by the inclusion of pins in the restoration? a) amalgam* b) methyl metacrylates c) composites resin d) cast restoratives note: pins do not seem to impair the physical properties of resin restoratives. Physical properties of cast gold or non-precious castings should not be altered by the inclusion of pins for retentive purposes. 42. Pins in amalgam: a) parallel to the outer surface of the tooth* b) parallel to the long axis of the tooth c) be placed to the enamel-dentinal junction as possible d) be inserted in the greatest number possible for maximum retention

note: ideally the pin channel should be placed parallel to the external surface of the tooth and located midway between the dentinoenamel junction and the pulp chamber. Pins should be inserted on the average 1-2 per missing cusp or line angle, but as few as possible to minimize internal stress 43. Mesio-distal displacement of an inlay in class II cavity can be prevented by a) adequate pulp depth b) an occlusal lock/ dovetail* c) Parallelism of surrounding walls d) An acute line angle at axio-gingival area 44. How would you bevel a class I for amalgam on 14 for extra-retention a) lingual and buccal undercut* b) mesial and distal undercut c) undercutting all around 45. Why do you do over packing of amalgam? To ensure that the marginal amalgam is well condensed before carving and that mercury rich layer is brought to the surface and removed prior to carving and finishing. 46. Why do you burnish after condensation? a) to continue condensation at margins* after it has been carved b) to bring excess mercury to surface* c) smoother surface reducing time for subsequent polishing* d) reduces porosities* 47. Why do you do burnishing after condensation? a) to continue condensation at margins b) to bring excess mercury to the surface c) to obtain smooth amalgam surface d) continuous adaptation to cavity margins* 48. Trituration is the process oxides coating the malgam particles are worn away, allowing the mercury to come in contact with the alloy 49. Dry field in amalgam with zinc is important because (2 or more) a) blisters formation b) loss of compressive strength c) recurrence of corrosion d) post-operative pain (excessive delayed expansion)* note: zinc was added to conventional dental amalgams as a processing aid to suppress oxidation of the key elements in the alloy. Some left and is detrimental in wet field because of production of hydrogen gas causing delayed expansion. Advantage than zinc-free amalgam is longevity 50. amalgam exhibiting high creep lathe cut

51. the amalgam exhibiting the least degree of creep is: a) lathe cut, irregular shape b) spherical c) combinations of spheres and filings (spheres can be compacted better and lesser Hg to use than lathe cut) page 220 sturdevant d) microfine e) dispersed phase high copper* note: recent investigation indicates that there is a ralationship between creep and marginal breakdown. Specifically, the findings indicate that dispersion systems of dental amalgam with a low gamma II phase have greater marginal integrity over longer periods of time. Likewise, the spherical alloys exhibit less percentage of creep deformation than do conventional cut alloys 52. burnishing of amalgam for marginal adaptation 53. there is a marginal breakdown in amalgam a) secondary caries will definitely develop b) secondary caries may develop c) the wider the gap, the more chances for secondary caries* 54. Core build-up for a vital posterior tooth a) glass ionomer b) Amalgam* 55. A shallow class I cavity is prepared and restored only with amalgam, patient came back in 8 days with occasional sensitivity, what would you do? a) leave it alone and tell patient pain will disappear within a few days b) lining and replace restoration* c) relieve high points 56. When you find ditching in amalgam filling replace the old amalgam 57. a patient each medium sized amalgam restoration on an upper premolar requests that it be replaced by a resin for esthetic reasons. In using a dentin bonding agent after cavity debridement a) apply the bonding material directly to cut dentine b) apply a phosphoric acid etch to dentin prior to applying the bonding agent* c) applying a chelating agent (EDTA) before applying the bonding agent 58. Materials:Wells Silver which comprises about 70% is to improve strength and decrease flow Tin which comprises around 26% reduces the expansion and also cuts down the strength and hardness of the resultant alloy

Copper is to decrease brittleness of material and strengthening alloy and decrease flow Zinc - deoxidizer

59. In which material maximum creep is observed? a) low CU amalgam admixed b) low cu amalgam lathe cut max creep* c) low cu spherical (very plastic, hard to mold) d) high in spherical - minimum 60. Zinc in Amalgam a) delayed exapansion* b) decrease strength c) blister formation 61. Main advantage of amalgam with high content of copper? a) better marginal sealing b) less corrosion c) better tensile strength d) higher immediate compressive strength* 62. High copper amalgam: a) High corrosion resistance b) High tensil strength c) High compressive strength* d) Improve marginal integrity 63. Why is copper added to amalgam? a) to increase strength* b) better corrosion resistance c) tensile stress decrease 64. Majority of fractures in class II amalgam restorations occur: a) at the gingival margin b) at the contact area c) in the area of occlusal isthmus* d) at the axiopulpal line angle 65. Amalgams best quality is high compressive strength 66. With the probe penetrating between the tooth and amalgam you will: a) always find a caries b) not always find a caries? 67. Important factors related to decision to use amalgam or composite restoration

a) ability to isolate the tooth/ teeth b) occlusal relationship c) esthetics d) operator ability note: conservative composite restoration is the treatment of choice for the small oclusal restoration 68. The best material to be used for final cavity toilet be amalgam insertion in shallow cavity is? a) alcohol b) phosphoric acid c) carboxylic acid d) water* e) EDTA 69. Why do you do over packing of amalgam through condensation a) to ensure that excess mercury reach the surface can be carved out, condensation* b) to keep enough material at the margins? note: purpose of condensation is to make a homogeneous mixture and express out excess mercury. 70. For which of the following preparations will not require occlusal and gingival grooves a) Class V for amalgam b) Class V for GIC c) Class V for composite with GIC base* 71. How would you give an undercut in a class I preparation for amalgam restoration on 14 for extra retention a) M and D undercutting b) All around undercutting c) Facial and lingual occlusal convergence* 72. Retention of Amalgam fillings self threading pins ( but decreases the strength of amalgam) 73. Core build up for a vital posterior tooth a) glass ionomer b) amalgam* 74. There is a marginal breakdown in amalgam a) Secondary caries will definitely develop b) Secondary caries may develop c) The wider the gap, the more chances for secondary caries*

75. Big core of pins for extensive carious lesions a) one large amalgam b) one basic mix and add mercury as needed c) Increments of amalgam with differential sequential trituration* d) Several mixes with alloy dissolve in mercury 76. To restore big cavity by amalgam, we use: a) mix at once b) small mixes of the same alloy* c) different mixes of different kinds of alloy 77. The compressive strength of amalgam alloy is influence by the ratio between. Amalgam alloy and mercury, which one of the following is correct a) excess of mercury is almost 0.1 b) excess of mercury is almost 0.2 c) excess of mercury is almost 0.3 d) not critical if the excess mercury can be removed by condensation* 78. The most important advantage of Amalgam copper alloy: a) improve marginal integrity b) corrosion resistance c) tensile strangth d) compressive strength* 79. The greatest single factor in the strength of a final amalgam restoration is: a) residual mercury content*? b) silver content c) copper content 80. Why do we need to make proper trituration of amalgam- to let the alloy particles, coated with mercury 81. The purpose for burnishing of amalgam: for adaptation of the margins* 82. Cu in high Cu amalgam to eliminate tin-mercury phase (gamma 2) 83. Newer types of amalgam alloys contain increased amounts of copper. Research demonstrate that copper a) reacts and strengthen the amalgam by its dispersion b) reacts to form a copper-tin rich phase thereby minimizing or eliminating the silver-mercury phase c) reacts to form a copper-tin rich phase thereby minimizing or eliminating the tin-mercury phase or gamma 2* d) Reacts and strengthen the amalgam by its grain diffusion

84. Varnish under amalgam to prevent a) short term leakage* b) Long term leakage 85. To prevent against posterior-anterior dislodgement of class II amalgam restoration? Dovetail and slots 86. Insufficient condensation of amalgam causes: a) mercury staining in dentin tubules b) shrinkage c) premature setting d) less strength in set amalgam* 87. Corrosion of amalgam restoration is usually caused by: a) sulfur (undertrituration) b) oxygen c) chlorides d) overtrituration note: corrosion products of dental malgam has been identified them a spredominantly sulfides of mercury and of silver. Undertrituration would be more ikely to contribute to tarnish and corrosion than would overtrituration 88. Core build-up with amalgam for vital posterior tooth 89. The disadvantage of self threaded pin and frictional pin- more deleterious friction locked pins a) crack tooth* b) need skill c) limited size 90. Cup a cusp with amalgam should be 2-3mm a) 0.5 mm b) 1.0 mm c) 2.0 mm* d) 4.0 mm note: The minimal occlusal thickness for amalgam is 1.5 mm, for gold 1-2 mm(Sturdevant) A cupping cusp with amalgam must be 2mm minimum 91. You decide to make postcore for crown in tooth, which material you prefer: a) amalgam* b) GI c) composite 92. Retention of amalgam fillings self threading pins

Note:pins does not improve strength of amalgam but rather weakens it only aids in retention 93. Occlusal cavity preparation for amalgam, retention a) slight undercut on mesial and distal walls b) slight undecut on buccal and lingual walls* 94. fractured amalgam that cartches with probe replace amalgam 95. condensation of amalgam to remove the mercury rich layer 96. trituration of amalgam to coat the particles with Hg, to bring the mercury in contact with the alloy particles 97. Why the isthmus area is easy to fracture in amalgam a) because of depth? 98. How should pins be placed for an amalgam core build-up? a) close to the dentoenamel junction b) parallel to the external surface of the tooth* c) away from the pulp d) parallel to the long axis of the tooth? 99. Dental Composite is indicated as a mixture of silicate glass particles with an acrylic monomer tat is polymerized during the application. The silicate provides reinforcement to the mixture and for translucency. While the acrylic monomer makes nitial material fluid and moldable for placement in the cavity preparation and penetrate to the micromechanical spaces of etched enamel. 100. a) b) c) d) What solution is used for acid etching 37 or 50% phosphoric acid* 50% silico-phosphoric 50% Fluoric acid several mixes with alloy dissolved in mercury

101. Which of the following materials is not compatible with composite resin: a) calcium hydroxide b) carboxylate cement c) zinc oxide eugenol * (inhibits polymerization of the resins) d) zinc phosphate cement 102. Silicate Cements, the first translucent material used in the anterior for small cavities. Composed of powder, acid soluble glasses, and a liquid contaning phosphoric acid water and buffering agents. It has fluoride release. Box like form of cavity preparation and a butt joint required because poor edge strength and brittle.

Needs mechanical retention and liners and bases to protect the pulp. Failure is because of discoloration and loss of contour. This is very rough also. 103. Which of the following are disadvantages of silicate restorations? a) high acidity initially* b) high alkalinity on setting c) it does not take a smooth finish* d) it will dessicate if not kept moist* note: Following the final setting of a silicate rstoration the pH rises but remains in the range of 5 to 5. 5 104. As it hardens, a properly placed silicate cement may be expected to: a) expand b) expand and later contract c) contract* d) contract and later expand note: subsequent dissolution of the silicate may also be snticipated. The restorative is also subject to syneresis (drying) and imbibition 9absorption of water) which contributes to increased solubility and disintegration 105. One of the claims for superiority of composite resin restorations over silicates, is that resin restorations a) prevent galvanic action b) are practically insoluble* c) have a better coefficient of thermal expansion 106. Composite restoration as substitute for amalgam as patient demand for esthetic treatment. What to do to ensure optimal results? a) acid etch on dentin b) chelating agent c) acid etch on enamel* d) enamel/bonding agent 107. There is a small fracture in composite in an incisal edge a) Resins, glass or quartz filled particles* b) Silicate material c) siliciphosphate d) glassionomer 108. Why is the acid etching done? To increase the surface area 109. A class III cavity is to be restored with a microfill resin, when might be acid etching be most appropriate? a) only in instances requiring retention such as for incisal edges b) not for class III cavities as the adjacent tooth surface will be altered by acid

c) everytime to decrease marginal leakage* d) only in instanced by shallow cavities to avid pulp irritation 110. Composite in premolar are not good because a) poor wear resistance* b) Deform c) Often fracture 111. area 112. a. b. c. d. Why do you bevel the enamel in composite resin to increase the surface Microleakage of composite resin is caused by shrinkage* expansion shrinkage then expansion expansion then contraction

note: composite materials shrink while hardening. This is referred to as polymerization shrinkage, can cause pulling away of material while it hardens. This is the reason why light must be on the opposite side. 113. There is a small fracture in composite at the incisal restoration a) replace restorations b) repair with unfilled composite? 114. When do you finish a composite restoration a) Immediately* b) 24 hours 115. Hybrid composite in Class III = negative fracture of the incisal edge 116. A small class III cavity is filled by composite resin, which one of the following cavity preparation is preferred? a) class III cavity preparation for amalgam b) class III cavity preparation for amalgam with bevel c) just removal of carious tissue and bevel* 117. A class III cavity is to be restored with microfilled resin. When would acid etching be most appropriate a) only in instances requiring retention in incisal edge b) not for class III since adjacent tooth surface can be damaged by acid c) everytime to decrease marginal leakage* d) only in cases with shallow cavities to avoid pulpal irrtation 118. Tooth separation is used in class III lesion to a) assist in convenience form

b) assist in outline form c) for tight contacts 119. When we finish composite resin resotoration? a) Immediately* b) 2-3 days later c) 3 weeks after d) never 120. Cement lining for composite resin a) CaOH* b) Zinc Oxide Eugenol c) Carboxylate d) Zinc Phosphate cement* e) varnish note: the solvent in the varnishes adversely affects polymerization; the resin, in turn, would be destructive to the semipermeable membrane formed by the varnish. Eugenol inhibitspolymerization of resins. If zinc phosphate is used as a baase the pulp should be protected by a cavity varnish or liner. 121. Sizes of Composite Material e. conventional composite f. microfilled g. small particle h. hybrid composite 8-12um 0.04-0.4 um 1-5 um 0.6 1 um

122. In small particle size, composite, the particle size is: a) 0.2 0.5 um b) 2-25 um c) 0.04-0.4 um d) 1-5 um* 123. Hybrid composite resin is used in posterior teeth: false, shold be small particles. Hybrid composite are for anterior restoration but at present are also used in the posterior teeth. 124. How can you improve the adhesion of a fissure sealant:acid etch technique 125. How do you treat dentine before putting GIC a) conditioner (etching gel)-10% polyacrylic acid for 20 secs* b) pumice note: release fluoride like silicates , possess favorable coefficient of thermal expansion, compared with silicate using phosphoric acid this cement uses polyacrylic acid which renders the final resto material less soluble.

126. How do you treat erosion, first step is: a) with pumice and water* b) spray with Na bicarbonate c) GIC 127. For restoration of upper premolar erosion with glass-ionomer cement we use: a) Pummice and water and / or acid* b) Usal cavity with undercuts c) Use of pins d) Do not interfere in surface of the cavity 128. GIC acid/base reaction (auto cure cement) 129. GIC I Clean cavity with water 130. GI prevents recurrent caries - bonding to dentine and enamel - fluoride release - contains polyacrylic acid 131. In shallow cavity you decide to restore it using GI for retention: a) put fist conditioner (citric acid) b) pumice and water c) polyacrylic acid conditioner* 132. Glass-ionomer cement class V upper premolar a) typical class V with retention undercut* b) Cleansing with pumice paste and water 133. GIC filling for an old patient with cementum exposure a) place GIC directly b) After drying with air GIC c) Pumice and directly apply GIC* 134. a) b) c) d) Mixing of Glass Ionomer cool the slab mix slowly mix fast? (within 30 secs) more powder to liquid*

135. When is CaOH Base contraindicated? - intermittent pain for long periods of 1 month - carious exposure 136. badly destroyed posterior tooth you restore with a) root canal treatment

b) casp covering c) gold porcelain crown 137. What type of gold best for filling? Foil gold 138. The mesiodistal displacement of an inlay in Class II cavity can be prevented by establishing: i. adequate pulpal depth j. occlusal lock or dove tail* k. parallelism of walls 139. Good light source for colour matching relatively uniform special energy distributed among various wave length 140. To achieve a wide range of desired color matching in light cured resins, for color stability which of the following systems would you select? a) a system with 6-8 colours* b) a system to allow incorporation of various amounts of tinting powders c) a system of mixing together base resin colors to achieve intermediate colours d) the use of tinted monomer additives achieving a smooth mix 141. There is a small surface in composite in an incisal the restoration, replace the restoration or repair with unfilled composite complete restoration 142. for small incisal edge fractures, material with the greatest reisstance to intrnal fractures a) glass ionomer b) silicophosphate material c) silicate material d) resins * 143. etching is performed by a) 50% phosphoric acid b) 37% phosphoric acid* 144. Gold is the most noble of metals, seldom tarnishing or corroding in the oral cavity. In this and certain other respects, it is an almost ideal dental restorative material for permanently preserving tooth structure. Its chief disadvantages are color, high thermal conductivity, and technical difficulties in forming a dense restoration. Types (gold foil, electrolytic precipitate and gold powder) 145. Gold restoration the most important for classification is hardness (compactness) (hardness is more casted gold). Tensile is related to cohesion. It is unfortunate that the restoration made with direct filling golds do not exhibit as high physical properties as those made with dental casting alloys. Consequently they cannot be

used to encompass a tooth (cast crown) nor can withstand masticatory stresses. Therefore they are used as direct filling materials. They are used for pits, small Class I , repair of casting margins and for class III and V resto. 146. Gold is classified according to: a) corrosion resistance b) compressive strenghth c) hardness*

147. Cast gold alloy advantages a) it is indestructible in the fluids of the mouth b) it is capable of restoring anatomical form and taking a high polish c) it does not discolor surrounding tooth structure d) it remains free from volume change after placement 148. zinc-oxide eugenol cements are effective as temporary filling in class II cavities because these cements a) may have good compressive strength b) elicit little pulpal response* c) have high resistance to abrasion d) minimize leakage and adaptation to dentine e) are insoluble

149. Retention for full veneer crown: a) Tapering b) long path of insertion* 150. The most retentive pins are depth in dentin (1.3 to 2 mm) a) friction locked pins - second b) self threading pins* c) cemented pins least retentive 151. There is a lateral perforation just below the gingival sulcus when using a post a) surgical exposure and fill the perforation with Zinc Oxide eugenol b) just continue to put the post and wait, fill later with cement c) arrest the bleeding by flushing with sterile water or anesthetic solution then fill it with CaOH and amalgam* 152. Temporary cementation of bridge a) ZNO eugenol* b) Composite c) Glass ionomer

153. The preparation of Class II cavity for gold restoration. The cavosurface angle should be at a) right angle b) obtuse* c) acute 154. Finishing the cavity walls is to: a) smooth the walls b) remove the residual enamel rods* 155. In what class of cavity is composite restoration most long lasting? a) I b) II c) III* d) IV e) V 156. What is important about adaptation of matrix band? a) mesial concavity of the root 157. Advantage of self threading pin over the cemented one? a) friction locked b) too expensive c) not all sizes available d) ____ cracking of tooth* e) most retentive* 144. Class IV gold difficult to burnish, long span bridge and sometimes even partial denture frameworks 145. 146. Gold alloy for inlay type I and II can be burnished Type IV and III crown and bridge alloy

147. Cast gold alloy show porosity which is attributed to: localized shrinkage, occluded gases and metallic oxides 148. A woman patient complaining of pain from a contact occlusion on a filled tooth. Tooth is vital with some degree of inflammation and patient is in transit treatment is: a. pulp extirpation and CaOH dressing b. Pulp extirpation and Ledermix pressing c. Pulp capping d. Remove filling and replace with ZnOE* e. Extraction

149. Self threaded friction pins present the least? Disadvantage compared with cemented pins: a. They are more expensive b. They require more skill c. They cause cracking in the teeth*. 150. The advantage of friction-locked and self treading pins: a) they require no luting agent b) they are more retentive than luted pins c) the required depth of pin channels is less than luted pins d) they are readily bent after insertion e) a,b and c*

note: Ideally, a pin channel should be placed parallel to the external surface of the tooth and located midway between the DEJ and pulp chamber. Pins shold be inserted on the average of one or two per missing cusp or line angle, but as few as necessary shold be used to minimize nternal stress. 151. One pin per missing axial line angle, Increase pin ( increase crzing in dentin, amount of dentin available between pns decreased and decresase strength of amalgam) there should be 1mm of dentin around the pinhole. The pinholes shuld be no closer to DEJ by 1 mm and no closer to th enamel surface by 1.5 mm., pin holes shld be avoided in the lingual of maxi molar and distal of mandi molar. 152. A patient with marked resoprtion of the gingivae around the remaining dentition, namely the lower bicuspids and anterior teeth. The oral hygiene is not good and some of the cementum appears to be in soft areas. Which is your preferred treatment? a. surface grinding followed by fluoride treatment b. surface grinding followed by glass ionomer restorations* c. class IV cavity preparation for glass ionomer restorations d. cavity preparation for amalgam e. application of fluoride solution without surface preparation? 153. Porosity in porcelain a. contamination by moisture* 154.Fracture in incisal angle in a lateral upper incisor with a class III restoration present and it is thin labio-lingually, what restoration would you do? a. incisal step inlay b. lingual dovetail inlay (no thin) c. Pinlay d. veneer crown ( no thin) e. porcelain fused to gold crown* or PJC ( MCQ, page 39) 155. You have to put inlay rest class IV, is badly destroyed tooth, What do you do? a. rebuild with cement to utilize to insert inlay

b. the same with amalgam c. no rebuilding utilize it by creation of correct shapes 156. resistance form in inlay preparation a. flat pulpal floor and gingival floor b. parallel walls* c. increase depth of axial walls d. divergence of walls

157. What cement do you use to cement a bridge in a patient with a moderate rate zinc ____? a. zinc phosphate* b. Polycarboxylate c. Glass ionomer d. Silico phosphate 158.On a carbide bur, a greater number of cutting blades results in a. less efficient cutting and smoother surface b. less efficient cutting and roughness surface c. more efficentt cutting and a smoother surface* d. more efficient cutting and a rougher surface

Dental Materials 1. Modulus of elasticity (Youngs modulus) stiffness of material, stress/stain ratio, within proportional limit. This is the slope of the stress and strain curve; it is the ratio of stress to strain (within proportional limit). A stiff material requires more stress to produce a given elastic stain (deformation); that is, its modulus is higher. The proportional limit denotes the highest stress within which stress and strain are proportional; it is not a measure of stiffness but of strength. 2. the stress-strain ratio within the proportional limit stiffness of an alloy* 3. the proportional limit is defined as: the maximum stress that can be induced without permanent deformation* the maximum elongation under tension that can be measured before failure the strain at the proportional limit the stress that causes failure the strain in compression note: Within the proportional limit, the strain (deformation) is proportional to the stress (load applied), so that the stress-strain curve is a straight line. The proportionality of stress and strain is referred to as Hookes law. Upon release

a) b) c) d) e)

of the stress, the structure will return to its original dimensions. The proportional limit is most often determined in tension. 4. The elastic limit is defined as: The minimum stress (force) required to induce permanent deformation of a structure. Is usually close to the proportional limit, although it may be slightly higher. 5. Elastic Limit: a. The minimum force applied that cause distortion* b. Is the greatest stress to which a material can be subjected* 6. the difference between the particles of dental plaster and stone die a. the shape and size of the particles* b. the chemical composition c. the solubility in water d. shelf life e. mixing time note: Plaster and stone are both calcium sulfate hemihydrates (same chemical composition and solubility). Shelf life depends upon storage conditions, and mixing time is not a main difference. note: Plaster of Paris (b-hemihydrate) - produced by calcinations (heating) of ground gympsum in open containers - the particles formed are irregular and porous - the water powder ration normally is 0.50 Stone: (a-hemihydrate) - produced by heating of ground gympsum under wet conditions (saturated steam) in pressure vessels - the particles formed are dense, crystalline, and quite regular (until ground for use as a dental product) - the water powder ratio normally used is 0.30 7. dental gympsum products are classified as - Type I, impression plaster - Type II, model plaster *manufactured from gympsum by heat in open vessels *requires approximately 50-60 ml H2O for 100 g powder - Type III, dental stone *manufactured from gympsum autoclaving under steam pressure at 120-130 degrees centigrade * requires approximately 33 ml H2O for 100 g powder - Type IV, high-strength dental stone *manufactured from gympsum by heating in boiling 30% sol. of CaCl2 or Mg Cl2 or in autoclave at 140 degrees centigrade with NaCl or Organic Acid

8. Potassium citrate and borax are retarders and potassium chloride, potassium sulfate and Rochelle salt are accelerators of plaster of paris 9. Increasing the water in plaster will decrease compressive and tensile strength and will decrease setting expansion and increase the setting time because more aqueous phase will decrease interaction of the crystals 10. The gypsum in casting investment can not only strengthen the investment, but also a. increase the setting expansion of mold b. prevent excessive setting expansion of mold c. constitute overall expansion of the mold through its setting and hygroscopic expansion* note: hygroscopic expansion of dental casting investment sets in contact with water, for example, by use of wet asbestos liner in the ring, by immersionin water or by water added technique. 11. Disadvatage of gypsum dies lack of edge strength 12. If the investment burnout rapidly, what will happen a. back pressure porosity b. cracking of the investment

13. What is the disadvantage of the gympsum dies? a. They lack accurate reproduction of surface detail b. Their overall dimensions are slightly smaller than the original impression c. Their edge strength is unacceptable for burnishing margins* d. The hazard of aspiration of toxic materials during trimming Note: electroplated or epoxy resin dies are better for adjusting the fit of the casting. 14. The working time of a polysulfide syringe impression material can be safely and effectivey increased by a. greatly altering the base/accelerator ratio. Can be used but this can alter the mechanical properties on the other hand, it is effective way of changing curing rate of condensation silicones. b. Doubling the mixing time c. Chilling the glass slab* d. Adding 2 drops of H2O 9 accelerates curing rate) e. Reducing the mixing time to 20 secs 15. Alginate impression should be removed fast because: a. It can produce syneresis b. Poor surface detail c. Minimize distortion*

Note: hydrocoliod impression should be removed fast from the mouth with a sharp jerk because of the elastic behavior of hydrocolloids, a sudden displacement of short duration will cause less permanent deformation than that caused by slow removal from the mouth 16. Polysulphide hydrophobic, high surface, tensile strength a. adhesive to tray (nope, in general polysulfides, polyether and condensation silicones need adhesives for resin trays or perforated trays. b. individual tray (note the less bulky and even ( 2-4 mm thick) impression material the more accurate compared for hydrocolloids) c. carefully casting of die low coefficient of thermal expansion 17. What is the significance of Hydrophobic in polysulfide? a. there _____ to be included in the impression need not be dried as the material will displace saliva (wet field technique used on reversible hydrocolloid only) b. prevent bubbles in pouring c. custom tray constructed d. use of adhesive?

18. Impression material which is long standing? a. vinyl polysiloxane* 19. For which material would it proved detrimental to leave in water until its poured 1 hr later? a. thiolo mercaptan rubber b. polyether* (hydrophilic nature) c. condensation silicone d. polysiloxane (additon silicone) (poly-vinyl siloxane) note: due to hydrophobicity of silicone impression materials they made a hydrophilic vinyl polysoloxanes, adding a surfactant) better details, soft tissue and stone affinity for hydrophilic surfaces. 20. Which impressin materials is more stable, so the lab can clean it for 24 hours before pouring: vinyl polysiloxane impression material 21. You use special (custom) tray for rubber base impression material fro drown work: a. to allow homogenous thickness of the material* b. to allow thick mix 22. An impression in an elastomeric material is taken for a seriese of inlays. After removal from mouth the impression is washed to remove little blood and mucous.

For each material what would be detrimental to leave in water before being poured one hour later a. Thiokol and mercaptan polysulfide b. Polyether* - absorbs water c. Condensation silicone* - prone to shrinkage, immediate pouring d. plysiloxane 23. What kind of impression material has to be put in water for 1 hour (dental mat book) a. ZnO, critical in humidity b. Vinyl silicone?, delayed pouring for 20 mins c. Rubber polysulphide after 30 mins, distortion d. Silicone rubber, after 30mins distortion 24. Which of the following elastic impression materials may be put aside for 1 hour before pouring a model? (boucher, page 389) a. agar hydrocolloids* (elastic, reversible hydrocolloid) b. alginate c. mercaptan rubber (polysulfides)* d. silicone rubber note: agar may be set aside for up to 1 hor only if kept at 100% relatively humidity and the mercaptan rubber up to one hour on the bench. Other materials must be poured immediately 25. Using adhesive in polysulphide impression material: thin and dry. 26. How to remove alginate impression material from mouth without permanent distortion rapid jerk in one motion 27. Reversible hydrocolloid in comparison with alginate: a. sol -> gel b. more stronger for undercuts (one piece impression) c. zinc-oxide impression paste used with ( impression compounds) 28. Wax in indirect technique ( type II soft wax), is different from the wax in direct technique (type I, medium wax) a. lower solidification temp* b. higher flow in room temp* c. some coefficient of thermal expansion d. higher solidification temp 29. Direct wax patterns should be constructed with a. type A inlay casting wax b. type B inlay casting wax * c. type C inlay casting wax ( indirect ) 30. In melting gold, which part of the flame we will use:

a. reduced cone* b. oxidizing cone c. liquidus cone d. any part of the flame will do note: the reducing cone should be used to minimize oxide contamination of the metal prior to casting 31. The proper zone of the gas-air blowpipe flame used for melting casting gold alloys is: a. the oxidizing zone b. the reducing zone* c. the zone closest to the nozzle d. combination of the oxidizing and reducing zones 32. Immediating heating of casting will cause a. distortion b. shrinkage porosity c. release of air bubbles d. no effect as long as temperature is maintained e. Expansion of mould? 33. Gold is cast into a hot mold to: a. hasten solidification of the gold b. maintain thermal expansion of the investment* c. eliminate oxides on the gold casting d. maintain molecular tension note: when using the thermal expansion technique to compensate for shrinkage of gold during casting this is particularly important. 34. If the investment ring is heated in the burnout oven too quickly, which of the following will result? a. release of PO2 b. back pressure porosity c. no problems, provided the oven temperature is kept below 1292 farenheight d. cracking of the investment resulting in finned castings* note: rapid burnout results in cracking of the investment, causing fins on the casting 35. The invested soldered assembly is preheated primarily: a. to drive off the sulfides in the investment that would contaminate the solderd conections b. to obtain a reducing atmosphere that allows the solder to flow more readily c. to firm mup the soldering investment so that it wont flake off during the soldering operation d. to eliminate water from the investment so that its presence does not hold down the temperature of the investment.*

note: the water in the investment would slow heating of the casting and, if heated too rapidly, could cause fracture of the investment due to escape of steam. 36. Gold alloy, Type 1 type Iv (hardest) 37. Gold casted in hot mold to maintain thermal expansion of the investement 38. Causes of acrylic porosities pacing with an acrylic mix that has too much monomer in it, under packing, curing too fast, and excessive tickness of acrylic are common causes of porosity The rapid curing of acrylic or the heat formed during the polymerization process is not dissipated rapidly enough. This results ion small voids form the entrapped monomer 39. The greatest cause of porosity in porcelain jacket crown is: a. moisture contamination b. excessive firing temperature c. failure to anneal the platinum matrix d. excessive condensation of the porcelain e. inadequate condensation of the porcelain* note: the others would not caus eporosities 40. the most frequent casue of breakage of porcelain jacket crown may be due to: a. inclusion of platinum foil b. use of weak canal c. voids in porcelain d. porcelain thinner by 1 mm* 41. What type of metal is used for a Maryland bridge? Base metal alloy 42. Chrome cobalt clasp compared to gold clasp about modulus of elasticity- more stiff chrome cobalt 43. The purpose of flux use: a. prevention of too much melting of the gold b. to prevent oxidation of the metal* 44. Casting flux should be used a. to prevent contamination of the gold from the asbestos liner b. to prevent too rapid melting of gold c. only with gold tat has been previously cast d. to prevent oxidation of the metal during melting* note: the purpose of flux is to reduce surface oxides on the metal prior to casting 45. The purpose of flux in dental soldering is to:

a. reduce metallic oxides* b. enhance a reducing environment while soldering* c. lower the melting temperature of the alloy d. lower the melting temperature of the solder note: the flux reduces oxides and maintains that environment while soldering. It does not lower the temperature of the alloy or the solder 46. Strain hardening, deformation in room temperature to make the metal stonger and harder. The surface hardness and strength, and proportional limit of the metal are increased with strain hardening, whereas the ductility and resistance to corrosion are decreased. However, the elastic modulus is not changed appreciably. 47. Annealing, the effects associated with cold hardening can be reversed by annealing. This is comprised of 3 stages: recovery , recrystallization, and grain growth. Uses half the temperature to melt the alloy 48. Sudden heat of the instrument crazing/ cracking? 49. Wrought structures are those that are a. properly annealed b. quenched from a high temperature c. constitute to overall thermal expansion of the mould through its setting and hygroscopic expansion d. cold worked* 50. Use of flux to reduce or remove the oxides 51. Narrow sprue will cause a. fracture of casting b. distortion of casting* c. shrinkage porosity of casting 52. The sprue for wax pattern should in an MOD a. be placed at periphery of pattern b. at point of greatest bulk of wax* c. at point of least bulk d. proximal areas 53. A sprue should be: a. placed so that the wax pattern is atleast 12 mm from the top of the crucible b. 18 gauge in diameter c. luted to the smooth proximal surface of a class II wax pattern d. formed to include a reservoir for large only* Note: the length of the sprue between the height of thec rucible and the wax pattern should be short, from to 3/8 inches. The diameter should be equal of

the thickest segment of the wax pattern, in the range of 10-14 gauge. It should be sprued at the locale tat will result in the least turbulence of the molten material flowing into the mold. Reservoir are required to prevent shrink spots especially when casting with air pressure or if the casting is large. 54. Which of the following describes requirements of a sprue? a. to lead the molten metal into the mold cavity with as much turbulence as possible in order to force out trapped gases b. to be as long as possible in order to speed the flow of gold into the mold cavity c. to have the thinnest diameter possible to insure that the sprue solidifies before the wax pattern d. to be smooth, with no sharp corners, and to funnel into the thickest portion of the wax pattern* note: turbulence is not desireable, since it tends to produce voids in the casting; long or thin sprues would tend to solidify before the casting, resulting in an incomplete casting. 55. Denture resins are usually available as a powder and a liquid that are mixed to form a plastic dough. The powder referred to as the: a) initiator b) polymer* c) inhibitor d) monomer e) dimmer The liquid usually contains: f) an accelerator g) benzoyl peroxide h) an inhibitor* i) a buffer j) a catalyst 56. The catalyst employed in self curing resins is: a. Hydroquinone b. Methyl metacrylate c. Styrene d. A tertiary amine* e. Benzoin methyl ether Note: hydroqiunone is the retarder for the monomer; methyl methacrylate is the main component of the monomer. Styrene is a monomer not normally found in self curing resins; benzoin methyl ether is used to activate sealnt and composites that are cured by ultaviolet light. 57. Polymerization of heat cured methyl metacrylate is initiated by: a. a tertiary amine

b. c. d. e.

a benzyol peroxide free radical* double bonds in the monomer molecule hydroquinone formation of long polymer chains

Note: benzyol peroxide is activated by heat. Hydroqiunone is present in minute quantities as an inhibitor to increase shelf life. Tertiary amine are used as activator for benzyol peroxide in self curing resins. 58. Compare cold cure with heat cures resin- more residual monomer 59. Polymerization of heat cured methylmethacrylate is initiated by a benzoyl peroxide free radical 60. The boiling point of methyl methacrylate is = 100.8 degrees centigrade (213.4F)

61. List 2 acceptable time-temperature curing cycles for scrylic dentures. - 160 degrees farenheight for 9 hours - 160 degrees farenheight for 1 hours, then boil for 30 mins 62. Polymerization of methyl metacrylate monomer is accompanied by volumetric shrinkage of: a. 7% b. 21%* c. 10% note: denture base resins are usually mixed in the polymer to monomer ratio of 3 to 1 by volume, so that the volumetric shrinkage is 7%

Prosthodontics 1. Antes Law: the combined pericemental area of the abutment teeth should be equal to or greater in pericemental area than the tooth or teeth to be replaced 2. What is the common cause of failure of PJC? a) failure to keep at least 1 mm of thickness* b) voids 3. Porcelain failures in porcelain fused to gold restorations are most commonly due to a) inadequate thickness of porcelain b) poorly designed gold frame work* c) failure to gold plate the metal prior to porcelain application d) too great a thickness of porcelain

note: Indaequate framework design resulting in flexing of the meatl and fracture of the porcelain is the most common cause of failure. 4. The reduction of labial surface for the porcelain fused to metal: Tylman a) 1.5 mm- 2 mm* b) 1 mm 5. The labial aspect of the porcelain bonded to metal restoration should be: (Boucher) a) reduced mm overall plus as additional mm in the incisal third b) reduced in two planes, following the original contours of the tooth* c) reduced 1 mm overall d) reduced parallel to the long axis of the tooth note: failure to follow the contours of the tooth will result in too little space for metal and porcelain in the incisal or occlusal portion of the labial surface or excessive reduction in the gingival portion. Choice (a) is incorrect, as a minimum of 1.0 mm should be removed in the gingival area, while (c) will normally result in reducing too close to the pulp in the gingival area. Choice (d) is often undesireable for obtaining the best path of insertion. 6. What kind of bridge for anterior segment of class II div II a) Maryland b) Cantilever type? c) Classic bridge 7. A dentist primarily splints adjacent abutment teeth in a fixed partial denture in order to a) improve the distributin of occlusal load* b) improve retention of the prosthesis c) facilitate plaque control d) improve embrasure contours e) preserve remaining alveolar support f) stabilize the abutment teeth g) improve mesio-distal spacing note: a) is generally the reason for splinting, although occasionally b) is a factor. There is no evidence that splinting would promote better plaque control or preserve alveolar support and the original, natural, embrasures are generally as good as or better than those produced by crowns. 8. Failure of the maryland bridge. No problem with design and occlusion what would you do? a) reduced the enamel and re-etched and cement b) reduce enamel, reduce the tags and recement

c) prophylaxis with pumice and H20, retched and recement* 9. Maryland bridge removed a) polish, pumice, re-acid etch, recement* b) reduce lingual c) make new bridge d) reduce mesial e) reduce distal 10. Labially placed gold PJC how will the appearance be? a) narrow b) longer c) broad* 11. Type IV Gold inlay: is not used for inlay as because it is difficult to burnish 12. Fixed Bridge: Gold used a) gold alloy ductile b) ductile c) hard* d) high elasticity 13. Porcelain jacket Crown, bluish margin means a) poor marginal adaptation* b) cement improper 14. When Maryland loses retention this usuaaly occurs a) metal resin function* b) within resin c) resin enamel interface 15. Maryland bridges are best made with a) Gold alloy formulated for bonding with porcelain b) Single base metal alloy? c) Multi phase alloy 16. Failure of Porcelain jacket crown less than 1 mm thickness 17. Failure of Porcelain in veneer metal restoration poor back metal design 18. Porcelain failure in porcelain fused to gold restorations are most commonly dueto a) inadequate thickness of porcelain b) poorly designed gold frame work* c) failure to gold plate the metal prior to porcelain application d) too greate a tickness of porcelain

note: inadequate framework design resulting in flexing of the metal and fracture of the porcelain is the most common cause of failure 19. Fixed bridges requires the use of: a) ductile gold alloys b) hard gold alloy III* c) gold alloy with high elasticity 20. Fixed bridge contraindicated when span is long 21. Porcelain bonded to metal restorations are contraindicated in long spans because: a) the shrinkage of the large bulk of porcelain distorts the metal framework* b) the metal used is inherently too flexible for long spans c) the metal used is inherently too prone to fracture in long spans d) the porcelain shrinkage will cause crazing of porcelain in long span note: the metal use is sufficiently strong and stiff if properly designed but shrinkage of the porcelain during firing will cause distortion in a long span bridge unless the porcelain mass is separated into 2 or more parts. Crazing is a problem of material handling not reated to length of span. 22. Bluish line gingival margin porcelain jacket crown (Skinner) a) incorrect cementation b) poor marginal adaptation* c) trauma of occlusion 23. Toxic product of porcelain 24. Impregnated cord in the gingival (during preparing for the crown) a) will make permanent damage of the gingival b) packing string is necessary to avoid destroying the CT 25. Cantilever bridge for upper canine use 4 and 5 for abutments 26. What would be the contraindication for a crown construction a) short clinical crown b) high caries rate* c) bruxism 27. Causes of fixed partial denture failure are many. The cause with the highest percentage of occurrence is: a) fracture of the components b) pulpal involvement and pathology

c) caries* d) failure of the luting media note: Recurrent caries is the most frequently seen failure in fixed restorations 28. Rough surface of porcelain ( or porosity) resulted a) in lack of compression or condensation* b) of sudden high temperature 29. A porcelain crown may appear too opaque 9 not translucent enough) because: a) the porcelain was not properly condenced b) the porcelain was dried too rapidly c) a proper vacuum was not used during baking d) all of the above are correct* note: all of the factors listed can contribute to excessive opacity by tending to increase porosity, which produces a more opaque porcelain 30. The most common failure in constructing porcelain to metal: improper metal framework* 31. Cantilever bridge has a pontic connected to a reatainer at one end only 32. The maxillary canine is missing the best way for making cantilever bridge a) both premolars bonded porcelain one piece cast* b) incisors and premolars 33. Why dont you use porcelain in long span bridgework? a) because of high casting shrinkage of porcelain* b) the metal used is inherently too flexible for long spans c) the metal used is inherentlytoo prone to fracture in long spans d) the porcelain shrinkage will cause crazing of the porcelain in long spans note: the metal used is sufficiently strong and stiffif properly designed but shrinkage of the porcelainduring firing will cuase distortion in a long span bridge unless the porcelain mass is separated into two or more parts. Crazing is a problem of material handling not related to the length of span. 34. When a Maryland bridge loses retention at the retainer and occurs during the first month. Fracture is at the: a) metal-resin junction* b) within the resin layer c) at the resin enamel junction d) enamel surface

35. Which is the most retentive for an anterior bridge? a) crown* b) pinlay c) inlay 36. Why gold type I not good for simple inlay? a) not abrade like enamel b) not good burnishing c) difficult to cast d) bad corrosion resistance note: Type I (soft) small inlays, easily burnishable and subject to slight stress Type II (medium) inlays subject to moderate stress, thick crown, abutments, pontics and full crowns Type III (hard) inlays subject to high stress, thin crowns, thin casting backings, abutments, pontics, full crowns and deture base, and short span fixed partial dentures and short span fixed partial dentures Type IV (extra hard) Inlays subject to very high stresses, denture base bars and clasps, partial denture frameworks, and long- span fixed partial dentures (Full crown are often made of this type) Note: Type III and IV crown and bridge alloys 37. The upper molar tooth in order to gain retention, diminish periodontal trauma and conserve tissue ( tooth), which is more advisable? a) Full veneer crown b) veneer gold grown* c) Wrought full veneer crown d) Porcelain fused to gold crown 38. The reduction of labial surface in porcelain to metal crown, should be at least: a) 1.5 mm* b) 1.0 mm 39. Gold bridges requires use of a) ductile gold alloys b) hard gold alloys* c) gold alloys with high elasticity 40. Most frequent cause of breakage of porcelain jacket crown restoration may result from? a) inclusion of platinum foil b) use of weak cement c) voids in the porcelain d) porcelain thinner than 1 mm* 41. Blue line in porcelain jacket crown poor marginal adaptation

42. What is the cement used for Maryland? a) restorative resin b) resin cement* c) ZnPO4 43. Temporary cementation of bridge a) IRM* b) Composite c) Glass Ionomer 44. How would you improve retention of Maryland bridge? a) incorporate mesh work in wax b) salt technique c) electrolytic etching* d) perforation in cast 45. Preparation of the tooth for Maryland bridge a) reduction of the cingulum* b) slight reduction of the incisal edge lingually 46. A patient comes to you with upper canine missing, which of the following will you suggest? a) cantilever from central to lateral b) cantilever from 1st and 2nd premolar* c) cantilever from 1st premolar 47. Where should the gingival margins of crowns and other restoration be placed for maximal teeth supragingival 48. Pontics of the anterior bridge a) just touches the alveolar ridge* b) impinges the alveolar ridge c) anterior of the alveolar ridge 49. A crown casting with a chamfer margin fits the die. In the mouth. The casting is open approximately 0.3 mm. A satisfactory fit and an accurate physioilogic contour of the gingival area of the crown can be best achieved by? a) hand brushing b) mechanical burnishing c) using finishing burs ans points to remove enamel margins on the tooth d) making a new impression and remaking the crown * e) relieveing the inside of the occlusal surface of the casting to allow for further sealing

50. What is the relationship of retentive portion of partial deture retainer to survey line off abutment tooth? a) towards gingival* b) towards occlusal c) midway between occlusal and gingival 51. surveying? a) 1st: parallel occlusal plane to serveyor table b) 2nd : tilt antero-posterior to get parallel proximal surfaces c) 3rd: get undercut locations 52. Location of rest seat in attrited teeth? 53. A patient aged 50 years presents with loss of all lower molars, with the bicuspids and ananteriors remaining in good condition. It is decided to make. A CAST METAL DENTURE INCORPORATED AS A SADDLE CONNECTOR WITH INDIRECT RETAINER CLASPS AND OCCLUSAL REST. What is the design of the metal or major connectors plate, touching the lingual part of the lower and slightly clear of the lingual mucosa 54. What is the relationship of the retentive portion of a partial denture retainer to the survey line? a) above ( occlusal or incisal) b) Below (gingival)* c) On the survey line d) Partly on the survey line and partly below e) Not related to the survey line 55. The partial denture is stable in the master cast but in the patient not stable. The source of error is: a) in the impression* b) in the lab 56. Clasp fracture when it: exceeds its elastic limit 57. In an RPD, the principal function of the indirect retainer is to a) stabilize against lateral movement* b) prevent sitting of the major connector c) aids in retention note: indirect retainers ia s part of a removable partial denture that assists the direct retainers in preventing displacement of other than tooth supported denture bases away from the basal seats by functioning through lever action on the opposite side of fulcrum line.

58. Even when not used as indirect retainers, canine extensions, continuous bar retainers and linguoplates should not be used without terminal rests because of the resultant forces effective when they are placed on inclined planes. True 59. Occlusal rest should be designed so that masticatory forces are directed: a) horizontally b) along the long axis of supporting tooth* c) mesially

60. The non-rigid connector in fixed removable bridge distal o anterior retainer 61. An old patient with very healthy anterior teeth, he lost the molars of the lower jaw. Youd ecided to construct a free-end saddle. The design of the major connector should be: no touch to teeth or gingiva 62. undercut for cobalt (retentive arm clasp) a) 0.75 b) 0.50 c) 0.25* 63. Ideally, reciprocal elements of the clasp assembly should be located at the junction of the gingival and middle thirds of the crowns of the abutment teeth. The terminal end of the retentive arm of the clasps is optimally placed in the gingival third of the crown. 64. Tightening a clasp against the tooth or looseningh it away from the tooth increases or decreases frictional resistance rather than adjusting the retentive potential of the clasps 65. Why do you polish the teeth before insertion of partial denture? To smoothen the rough surface 66. The contact between artificial and natural teeth in partial denture (like natural teeth) slight tough in the balancing side 67. Designing of ligual bar? Usually oval in cross section, approximately 3 mm deep and 1.5 mm thick, although occasionally a bar of a half pear shaped section is used. The bar should be positioned so that the upper border is atleast 2 mm below the gingival margins of the standing teeth, and its lower border is not less than 2 mm above the floor of the mouth.. Therefor the needed space is 7 mm. If less use a liguoplate and if more the bar should be placed as low as possible to prevent irritation to the tongue. Cannot be used in sever lingual undercut or lingual placent of the lower anterior teeth.

68. Lingual plate 2mm below the incisal edge and 2 mm relief from the functional level of the floor of the mouth. 69. Labial bar when the dentition is retroinclined. The labial bar often give rise to aesthetic problems. It mat aso cause irritation to the lips and cheeks. On the basis, its use is usually reserved for those cases where no alternative is possible. 70. Gold clasp is more elastic than cobalt chrome, but C.C. has high modulus of elasticity. The first is true the second statement is false 71. If the major connector before relining Cl I Kennedy was touching the teeth but after relining it is not touching you should have done, added more impression material during taking the impression Note: reining of distal extension removable partial dentures requires the same impression procedures and observations that are associated with a secondary impression technique. State threee objectives that may be realized by this method. (1) denture framework may be returned to its intended relationship with the supporting teeth. (2) Reestablishment of optimal tissue support for the denture base (3) Restoration of the original occlusal relationship with opposing teeth. 72. A dentist making an impression to reline a distal extension base notes that indirect retainers are not in their respective abutment teeth. He should: a) finish the reline procedure, realizing that the denture bas will settle agin b) use additional impression material in the denture base c) not to be concerned with the relationship of the indirect retainers to the abutment teeth d) start over and do it correctly* note: use less pressure impression technique. 73. Tooth borne removable partial denture bases should be relined when: a) food routinely s entrapped between the residual ridge and the denture base* b) the supplied teeth on the denture bases are not in occlusion c) an occlusal rest is broken d) the denture has been in service for 18 months. 74. Relining of tooth borne deture bases, when indicated may be accomplished with atopolymerizing acrylic resin as a chairside procedure because a) it is a less expensive process b) it eliminates laboratory procedure c) the denture base receives its support from the teeth,as opposed to receiving it from residual ridges*

d) the dental assistant can perform the service. 75. RPI provides more favorable loading distribution 76. You decide to replace the anterior missing teeth for a partial denture using the bridge, why? a) because of esthetic* b) overjet 77. When surveying, tilting the cast 78. Some uses of a dental cast surveyor are: a) surveying the diagnostic cast b) recountering abutment teeth on the diagnostic cast c) contouring wax patterns d) measuring a specific depth of undercut e) surveying ceramic veneer crowns f) placing internal rests g) machining cast restoration h) surveying and blocking out master cast i) all of the above* 79. What factors determine the path of placement and removal partial denture? a) guiding planes* b) sex of patient c) retentive areas* d) areas of interference* e) esthetics* f) all of the above are correct 80. The end result of selecting a suitable anteroposterior tilt shuld be provided the greatest area of parallel proximal surface that may act as guiding planes. True 81. The lateral tilt of a master cast will be the position that provides equal retentive areas on all principal abutments in relation to planned clasp design True 82. What statement is false: not to survey when making the crown? 83. Retentive part of the clasp position is below the survey line 84. Free end saddle partial denture, to reduce the load? 85. A distal extension removable partial denture gains its support through abutment teeth and residual ridges (True)

86. Retentive clasps criteria? Which of the following factors determines the amount of retention that a clasp is capable of generating? a) degree of the angle of cervical convergence* b) how far into the angle of cervical convergence the clasp terminal is placed* c) flexibility of the clasp arm* 87. Lingual bar and lingual plate: a) accumulation of plaque on labial bar is less than the accumulated on labial plate 88. When a removable partial denture is completely seated, the retentive terminal of the clasp arms should be a) passive and applying no pressure on the teeth* b) contacting thhe abutment teeth only in the suprabuldge areas c) resting lightly on the height of contour line on the abutment tooth d) applying a definite, positive force on the aburtment tooth in order to prevent dislodgement of the removable denture. 89. Which of the following are the common errors in designing an RPD? a) overloading of supporting teeth* b) suspending a free end saddle between an occlusal rest on an anterior abutment and posterior border of the ridge c) failing to recognize that an RPDs prime function is to stabilize and restore each arch d) failing to understand that the 2 prime function of occlusal rest are to provide indirect retention and to tansfer load to abutment teeth e) inadequate use of saddle areas to support tooth borne restorations 90. In a removable partial denture principal function of indirect retainer is to a) stabilize against lateral movement* b) prevent tilting of major connector c) restrict tissue ward movement of the distal extension base of the partial denture note: indirect retainer is a part of a removable partial denture that assists the direct retainers in preventing displacement of other than tooth supported denture bases away from basal seats by functioning through lever action on the opposite side of the fulcrum 91. The rest seat for an occlusal rest should be prepared so that it is a) parallel to plane of occlusion b) at right angle to the minor connector c) slopping apically towards the center of the occlusal fossa of the abutment teeth* d) at right angle to the long axis of the tooth

92. The outline form of an occlusal rest should be: a) a rounded triangular shape with the apex nearest the center of the tooth b) as long as it is wide and the base of the triangle shape should be the same dimension as one-half of the distance between the tips of the adjacent buccal and lingual cusps of the abutment tooth c) usually the marginal ridge at the site of the rest should be lowered approximately 1.5 mm d) all of the above are correct* 93. The floor of the occlusal rest seat should be inclined slightly toward: a) the marginal ridge b) the buccal surface c) the lingual surface d) the center fo the tooth* 94. The angle formed by the occlusal rest and vertical minor connector from which it originates should be less than a right angle. True 95. The rationale for providing a mesio-occlusal rest on a terminal abutment adjacent to a distal extension area as part of the direct retainer assembly is: a) easier to prepare b) esthetically more acceptable c) more of the residual ridge is utilized to support the denture base* d) all of the above are correct 96. Where should occlusal rest seats logically be prepared on posterior tooth abutments of a class III partially edentulous arch? a) on the portion of occlusal surfaces adjacent to the edentulous area* b) on the portion of occlusal surfaces farthest from the edentulous area c) lingual surface of the posterior abutment 97. We polish the natural abutment before seating the RPD a) to match the colors b) smooth surfaces* c) to prevent plaque formation and good oral hygiene 98. The difference between chromium cobalt and gold clasps a) chromium cobalt is more flexible (elastic) than gold b) Gold is more elastic (flexible) than chromium cobalt*

Note: Gold alloys have a modlus of elasticity (stiffness) approximately one-half of that of chromium-cobalt alloys for similar use 99. The greatest aount of stress an alloy will withstand and still return to its oroginal shape in an unweakened condition is called = Yield Strength 100. Chromium cobalt alloys generally have a higher yield strength than do the gold alloys used for removable partial dentures. False 101. Chromium cobalt alloys alloys will deform permanently at lower loads than will gold alloys. True 102. Stiffness of an alloy is known as the Modulus of elasticity

103. Gold alloys have a modulus of elasticity approximately of that of chromium-cobalt alloys for similar uses (True) 104. A High yield strength and a low modulus of elasticity produce high flexibility 105. Chromium cobalt easier to work harden than gold

106. Chromium cobalt have a lower density than gold alloys in comparable sections and are therefore about as heavy as the gold alloys (True) 107. Indirect retainer: a) stability of the denture against horizontal forces b) prevention of tissue- ward movement in distal area c) assist the direct retainers in preventing displacement of the denture than tooth supported denture bases away from the basal seat *

108. The direction of the occlusal rest: RPD - It is spoon shape, directly apically at the middle of the occlusal surface 109. 110. A cast chrome cobalt clasp in a premolar should be short circumferential clasp inrfrabulge clasp (roach) 0.025 in undercut the seat for the occlsal rest should be prepared so that it is a) parallel to the plane of occlusion b) at right angle to the minor connector

c) slopping apically from the center of the occlusal fossae of the abutment tooth* d) at right angle to the long axis of the tooth should be less than a right angle 111. Function of an indirect retainer prevents saddle movement away from the tissues, wrong, assist the direct retainer only 112. A partial denture that fits the master cast fails to fit in the patients mouth distortion of the impression 113. To release a chromic clasp you need more force than that needed to release gold clasp because less flexible 114. Precision attachment partial detures (retainers) is by: a) the clasps, should be below the undercut serve as indirect retainer action b) male and female keys

115. Precision attachment - direct retention by friction resistance and spring action ( book). This consist of two units, one being attached to an butment tooth, the other being attached to the denture saddle. 116. Relining free-end saddle a) Put the framework in its original relation to the supporting teeth* b) Tapping with opposite teeth Undercut for CoCr is .25 mm, retentive arm clasp of horizontal undercut

117.

118. In removable partial denture the principal function of the indirect retainer is to restrict tissue ward movement of the distal extension base of the partial denture. Nope, rest 119. muscle that acts on the border of the mandibular deture (complete) a) temporalis b) mentalis* c) Lateral pterygoid d) Levator anguli oris e) Orbicularis oris Note: lateral - Masseter muscle, buccinator, modiolus, orbicularis oris, tongue, medial pterygoid muscle and ramus of mandible Note: borders mylohyoid, masseter, lingual, labial and bucall fenum, buccinator Ulceration on the crest of the mandibular ridge under a complete denture?

120.

a) Unbalanced occlusion* b) did not relieve the denture before processing c) pressure spots at the time of impression- adjust with disclosing paste* d) spiney ridge crest provides relief in the denture* e) excessive vertical dimension* f) rough and sharp projections fro the inside of the denture* g) allergy to denture base* h) fungal, bacterial and viral infections i) dietary, vitamin or mineral deficiencies j) hormonal imbalance and bruxism 121. Areas of abrasion are indicative of faulty occlusion. An area ahowing lacerated mucosa at the border area of a denture is indicative of overextension 122. A patient want to replace missing anterior teeth, what would you consider before replacement? a) amount of ridge resorption 123. 124. Overdenture is good because of: proprioceptors For the stability of the lower denture: a) the occlusal plane should be below the tongue b) the occlusal plane should be above the tongue c) The lingual flangs should be concave d) Occlusal plane should be at the level of the tongue*

125. Fovea palatini, landmark to determine the posterior border of the upper denture: Posterior dam = wrong anterior area, The incisive papilla is usually 8-10 mm anterior to the center of the incisive papilla. The tips of the canines are also related to the center of the papilla, and a high percentage of canines are +/- 1 mm at the center of the incisive papilla. 126. Tissue conditioning materials(silicon resilient) lining are more resilient than plastic acrylic 127. What are tissue conditioning material? They are resilient soft flowing reline materials that transmit the continuous stress of force and motion through the occluding dentures to the basal seat tissues.The materials allows for conditioning of the mucosa but also allows the mucosa to regain its original form. The o0bjective of this material is to reduce inflamed and istorted soft tissues and to provide a dynamaic impression for denture relining.

128. Cusps of maxillary first premolar during setting of teeth, when you look at it occlusally, what is the position of the palatal cusp:

a) Mesially* b) Distally 129. Post dam extension should be; a) on the hard palate b) thicker anteriorly and thin posteriorly c) on the soft palate* note: vibrating line of palate is an imaginary line drawn across the palate that marks the beginning of motion in the soft palate when the patient says ah. It usually passes about 2 mm in front of the fovea palatinae. Theis is on soft palate not a junction between the soft and hard palate 130. A patient comes back within two days of an immediate denture fit is complaining of soreness a) biopsy b) relief* c) antibiotic d) reline 131. A patient aged 50 years presents with loss of all lowere molar, with the bicuspids and anteriors remaining in good condition. It is decided to make a cast metal denture incorporating a saddle connector with indirect retainer, clasps and occlusal rest. What is the design of the major connector Plate, the metal touching the lingual of the lower and slightly clear of the lingual mucosa a) clear of the teeth, gingival and lingual mucosa b) slightly embedded in the lingual mucosa to stop food getting under c) touching the lingual of the lower incisors and slightly embedded into lingual mucosa d) touching the lingual of the lowerincisors and clear of the lingual mucosa* 132. Patient with complete denture and bilateral angular chelitis, the most likely cause is: a) vit. B deficiency b) closed vertical dimension* 133. Angular cheilitisis the inflammation of the angles of the lips. 1) loss of vertical dimension may cause fissures at the corner of the mouth. Seen often in edentulous patients wearing dentures with insufficient vertical dimension. Drying of the saliva drooled into the fissures may result in ulcerations resembling a vitamin deficiency. Candida albicans is often a causative organisms of associated inflammation 2) Cheilosis ; Vitamin B Complex deficiency 3) plummer-vinson syndrome: resembles B avitaminosis

Treatment is as follows: a) remove from environment, application of moisteners, shielding by clothing or hood. b) elimination of smoking or changing to another form of smoking, perhaps even filter-tipped cigarettes application of mild antiseptic antibiotic salves mycostatin. c) restore correct vertical relationship d) prescribe a b complex group vitamin. Posterior border of upper denture is placed : a) on hard palate b) immobile soft palate* c) junction of soft and hard palate note: the distal end of the upper denture must extend at least to the vibrating line which is in the soft palate anterior to the fovea palatinae ( vibrating line) 135. Oral tissues supporting upper denture get blood supply from: a) descending palatine artery and posterior superior alveolar artery* b) glossopharyngeal artery In edentulous mouth hypertrophied tissue a) may be caused by trauma under old denture* b) will take very great biting stress c) will be irritated by further pressure 134.

136.

137. TMJ problem has pain, soreness and tenderness to palpation over the joint muscular imbalance (fatigue, trismus); midline diviation when opening; crepitus of TMJ, head, neck and back pains; disturbances in the middle ear causing vertigo, nausea, tinnitus or hearing loss. 138. What is the treatment plan for a denture patient with TMJ symptoms and a closed vertical dimension? Place a temporary acrylic splint over the teeth. Bilateral comfortand convenience of chewing for a period of one month are required before final restorations are made. Opening gradually the vertical dimension shold be done gradually. Complete restoration of the vertical in one step might well result in failure of the muscular to adapt. 139. What is the treatment for a patient with TMJ symptoms when the vertical apparently is correct? Put the patient in soft diet, apply hot, wet packsto TMJ area and use muscle relaxant drugs. Check the dentures carefully for faulty occlusions and eliminate the possibility of clenching or bruxing. 140. increased free way space, over closure, TMJ pain

141. cause of cheek biting inadequate buccal overjet of upper posterior teeth, grossly overclosed vertical dimension (decrease freeway space) and poor patient coordination and control.

142. A patient has denture stomatitis, treatment is to leave the denture out, not worn for a period or relieved extensively 143. Which is not used to determine vertical length of occlusion ( freeway space) interocclusal clearance) 144. clicking noise of complete denture during speech is generally caused by: a) a loose fitting denture b) newly rebased denture c) insufficient freeway space(too much vertical dimension)* d) too much free way space centric relation - occlusion in the most retruded position of the mandible Desirable to change VD a) new facebow b) new centric occlusion* c) different teeth d) increase VD

145. 146.

147. Centric occlusion is the raltion of the opposing occlusal surfaces, which can provide the maximum planned contact, intercuspation or the occlusion of teeth when the mandible is in centric relation with the maxilla. 148. Terminal Hinge Position is the mandibular position (condyles in the most retruded position) from which further opeing of the mandible would produce translatory rather than hinge movement. 149. Error in remounting denture a) in impression b) jaw relationship* c) in flasking d) unstable baseplate Splint-cast technique correction of occlusal errors?

150.

151. Porosity of thick part of the denture (localized): due to incorrect heating (rapid curing of acrylic) 152. 153. TMJ joint pain (myofacial discomfort) a) at people with minor chronic illnesses* b) with age Major- connectors in lower partial dentures: a) lingual bar* b) lingual plate

154.

c) labial bar Resting posture of the mandible: not changing throughout life (CJR)

155. The ala-tragus line : a line that runs from the direction of the inferior border of the ala of the nose to the tragus of the ear. 156. Displacement about the fulcrum, prevented by: good stability

157. Bilateral balanced occlusion: is simultatneous contact of the occluding surfaces of the teeth of both sides of the mouth in various jaw positions 158. the stability of the mandibular complete denture will be enhanced when: the tongue rest on the occlusal surfaces (nope) it should be in balanced with the lips and buccinator muscles 159. Oral para-functional movements are potentially harmful a) in old aged b) because of premature contacts* removal of teeth in healthy individual make changes in a) centric relation b) centric occlusion*. Movement of teeth, tilting etc. c) vertical dimension d) loss of vertical dimension protrution recorded made with wax to be sure about a) patient didnt close in lateral plane b) incisors made contact* c) all cusp penetrate the wax d) uniform perforation What are the firve principal factors in laws of protrusive movement? a) The inclination of the condylar guidance* b) The prominence of the compensating curve* c) The orientation of the plane of occlusion* d) Inclination of the cusps* e) The inclination of theincisal guidance* What doesnt control mandibular movement? a) Teeth b) TMJ c) reflexes conditioned in neuromuscular system d) clearance interocclusal* e) proprioceptors

160.

161.

162.

163.

164. Resorption of bone after extraction of posterior teeth is in the lingual of the lower ridge and in the buccal side of the upper ridge, making the lower ridge wider than the upper 165. Set teeth in front of the incisive papillae 8-10 mm

166. Patient with complete denture is loose and causing sores, you decide to rebase the loose denture a) use resilient material (liquid resin tissue conditioner)* b) tell the patient not to wear it for a while 167. Overbite is the distance than the maxillary incisal edge closes vertical ly past the mandibular incisal edge when the posterior teeth are in occlusion. The overlap may be measured absolutely or relatively. The degree of overbite may increase, decrease or remain the same as a child gets older. 168. Overdentures are best for canines and premolars

169. overdentures are better than complete detures because: the presence of proprioreceptors 170. Denture stomatitis can be casued by: a) an ill-fitting denture* b) leaving the denture in place constantly* c) not cleaning the denture adequately and daily* d) a rough denture base*

171. What are aids in determining the correct occlusal vertical dimension? Some aids are pre-extraction records, such as study casts prior to extractions, cephalometric x-ray, or pre-extraction photographs of the patient; post extraction evaluations such as establishment of physiological rest position, the use of phonetics, evaluation of tactile sense of the patient 9enurologic perception0 or facial measurements are good postextraction aids to help determine the correct occlusal vertical dimensions. 172. What is the interocclusal distance? Interocclusal distance is not always constant. This is the distance is the space between the upper and lower teeth when the mandible is at rest. This sace varies only slightly during the waking hours, being slightly greater at the end of the day. During sleep, it is non-existent due to the lack of muscle tonus. It changes slightly during growth, is reatively stable through adulthood, and changes slightly again as the tissues functions and tone decreases 173. The rest position of the mandible depends on the tonus of the musculature. It varies slightly at different times of the day, being the greatest at the end of the day and absent during sleeping hours. It also varies during chronic illness and

varies with the different positions of the head in respect to the rest of the body. The rest position is the most constant, non-varying position of the mandible to use as a strating ppint in the determination of the free way space. Increasing the occlusal vertical dimension beyond the rest dimension usually results in denture failure. 174. Centric Relation is the most retruded relation of the mandible to the maxilla when the condyles are in their most unstrained osition in the glenoid fossae, from which lateral movement can be made at any given degree of separation. 175. Face bow is a caliper like device that is used to record the relationship of the maxillae to the temporomandibuar joints or the opening axis of the jaws and to orient the maxillary cast in this same relationship to the opening axis of the articulator. 176. Kinematic face bow used for recording (to locate) -hinge movement position axis. 177. To make a partial denture for a cleft palate patient. a) extract all remaining teeth b) maximum conservation of remaining teeth* c) not for a child under 6 years of age In a patient with full denture, clicking sound is heard, what is the cause? a) insufficient freeway space* b) loose dentures How should the pontic replace the first premolar? Lightly touching How to eliminate the defects that arises after processing the deture? a) remove posterior teeth and replace b) new dentures* or rebase Protrusive relation of mandible condylar guidance and incisal guidance Ala-Tragal line. Horizontal plane of the teeth or occlusal plane a) ala of the ear tragus of the nose* b) parallel to the Frankfort plane c) guide to occlusal face height d) guide to occlusal plane of denture* Frankfurt Plane:Horizontal plane of the Head a) Ala of the Nose Tragus of the ear b) Sella nacion line

178.

179. 180.

181. 182.

183.

c) Porion ( highest point on the margins of the external auditory meatus)to orbitale ( the lowest point on the nfraorbital margin)* d) NAB Line 184. What makes the mandible in a depressed position, except a) contraction of the lateral pterygoid b) contraction of the mylohyoid muscle c) contraction of the temporalis muscle* d) relaxation of the muscle and let the gravity depress mandible wax pattern, not to leave it on the bench for long time because of distortion

185.

186. Articulator used to record: occlusal plane relationship in relation to orbit and condyles 187. flask 188. The cause of diffuse porosity in complete denture: improper packing in Sudden heat to investment , cause: a) crazing or fissuring in the investment* b) rough surface of the die c) back pressure porosity

189. A facebow records orientation of the occlusal plane to axis orbital and FF plane 190. 191. Hinge axis: using Kinematic face bow When it is desirable to change vertical dimension a) new face bow transfer b) new centric relation (CR)* c) Selection of different teeth Antes law: a) ratio between crown and root of abutment tooth b) perioidontal areas of the teeth being replaced by abutment tooth* Incorrectly processed acrylic denture porosity is more likely to occur a) throughout denture b) thickest part of denture* c) thinnest part of denture VDR changes when patient changes position The rest position of the mandible (posture position) is:

192.

193.

194. 195.

a) vertical dimension + freeway space* b) not changing throughout life 196. There is an irregular area in the casting of gold crown due to inclusion of the investment 197. Physiologic rest position is useful to determine a) vertical dimension* b) centric relation c) centric occlusion

198. High vertical dimension pain with diffuse distribution under a mandibular complete denture 199. In determining the posterior limit of the maxillary denture base, which of the following is on the posterior border? a) hamular notch* b) hamulus process c) fovea palatini* d) vibrating line e) pterygopalatine raphe* 200. List all the muscles that directly influence the flange extensions of the maxillary denture = The upper ncisal, the biuccinator, the tensor palatini, and the depressor septi. 201. Hyperplastic tissue formation after long time of immediate denture a) change the design* b) surgical interference* c) relieve the fanges d) ask the patient to leave the denture out for rest 203. Generalized discomfort with or without this soarness excess occlusal vertical dimension faulty occlusion heavy biting force reduce buccal and lingual dimension and reduce vertical dimension Cheilitis, the most common cause is a) vitamin B deficiency b) a closed vertical dimension * c) improper balance of occlusion

202.

204. The area of the denture covering the incisive foramen is relieved to prevent impingement on the nasopalatine nerve and blood vessels

205.

A record of the protrusive relation is used to a) aid in making a facebow record and to adjust the incisal guidance plane b) aid in determining the free way space and to adjust the path of incisal guidance plane c) register the condylar path and to adjust the path of incisal guidance plane.*

206.

Using a Sprue that has a too narrow gauge causes: a) shrinkage porosity in the casting* b) no alteration in the casting c) fracture of the casting d) distortion of the casting Wrought structures are those which have been: a) properly annealed (annealing is heating of metal to relieve the effect of cold work) b) subjected to cold work during fabrication* c) quenched from a high temperature (strain hardening increase surface hardening and strength but decreases ductility and resistance to corrosion)

207.

208. The area of the denture covering the insicive foramen is relieved to prevent impingement on: a) the nasopalatine nerve and blood vessels* b) degeneration of the labial frenum c) internal nerve and blood vessels d) stensens duct. 209. Hyperplasia under unfit denture: a) remove it surgically* b) biopsy c) relieve the denture at this area Vertical dimension at rest position equeals: a) the vertical dimension of occlusion plus the interocclusal space* b) the interocclusal space c) the interocclusal space plus the freeway space *interocclusal space = freeway space

210.

211. Which of the following does not control mandibular movement during mastication? a) the teeth b) TMJ

c) Conditional reflexes in neuromuscular system d) Interocclusal clearance or freeway space* e) Proprioreceptors in the periodontal ligament 212. In a mandibular denture midline fracture repair: a) when using heat cure resin, there is possibility of warpage b) when using cold cure resin, it is more prone to fracture c) there is more possibility of error in assemblage due to the limited fracture area d) all of the above are correct*

213. The greatest possibility bearing area should be used when constructing a complete mandibular denture to a) improve esthetics b) improve the plane of occlusion c) provide balance occlusion d) better withstand the forces of mastication and to increase border seal* 214. Definition of articulation lateral excursion from the centric relation protrusive movement, posterior teeth are in contact

215. Full maxillary denture occludes with the natural anterior teeth, which one of the following is correct a) resorption of the maxillary anterior ridge* b) hypercementosis of the mandibular anterior teeth 216. Why we use splint cast technique? a) because of distortion of the impression b) improper flasking (improper pressure during curing) c) for correction of occlusion errors* Localized porosities found in thick sections of the acrylic dentures because: a) rapid curing of the acrylic* b) improper pressure in flask Why we prefer to use elastic impression materials: a) it does not distort when we remove it from the mouth* b) It is accurate The impression material for study cast: a) irreversible hydrocolloid b) complete denture modeling compound

217.

218.

219.

220. TMJ or myofunctional discomfort occur in:- patients with minor chronic illness 221. Before positioning the cast on surveyor, we think about a) number of missing teeth b) aesthetic c) way of removal d) undercut and tilting* Cranio-mandibular discomfort: a) pain through the inferior dental nerve b) facial muscles* The difference between self-cure and heat cure acrylic resin is the a) higher residual monomer after complete poplymerization The articulator in which the joint member is in the maxilla a) Non-arcon b) Hanau - Arcon* Patient with lower denture complains of paresthesia of lower lip a) pressing the mental nerve* b) pressing the genioglossial and mylohyoid nerve Pain on the ridge of the mandible (on the crest there is an ulcer): a) not well finished the fitting surface Balanced occlusion the most retruded position of the mandible

222.

223. 224.

225.

226. 227.

228. Definition of Articulation lateral excursion from the centric relation, protrusive movement and the posterior teeth are in contact, dynamic continuous range of contact of the upper and lower teeth. 229. Contraindication for upper and lower complete denture? erosive lichen planus* after radiotherapy* children under 6 years of age* In relaxed mandibular position, the physiological rest position is related to centric relation centric occlusion Which of the materials can be poured after 2 days: a) polyester b) polysulphide c) condensation silicone

230.

231.

d) vinyl silicone* 232. A reliable method of registering centric relation partial denture? a) a wax record with the remaining teeth in occlusion* b) a record of all the remaining occluding surfaces and the mandible

233. A patient comes because of soarness associated with denture. What is the first step a) an impression b) new denture c) history and find cause* d) correct occlusion Endodontics 1. A child of 9 years of age, fracture of apical third, treatment i. wait to recall after a month* ii. endodontics of the coronal part

2. The best treatment of an acute periodontal abscess. What is your preferred treatment? iii. incision and drainage* (Yes, Grossman) iv. endodontic treatment v. extraction 3. Electrical vitalometer is: (2 or more answers) a) to test recently erupted tooth b) a response to all electrical stimulus* c) reveal potential necrosis* note: electric pulp tester is an instrument used in testing the vitality of the pulp of the tooth. 4. The electric pulp tester can: a) register diseases of the pulp b) be used to test teeth with full coverage c) always elicit a satisfactory response in multirooted teeth d) all of the above are correct e) all of the above are incorrect* note: the result of pulp testing in multirooted teeth is sometimes questionable because the pulp may be vital in one or two canals and the necrotic in another. It is even possible that the tooth is unreactive and yet when opened is vital. 5. The palatal root of the maxillary molars is located a) in the pulp chamber under the mesio lingual cusp*

b) in the pulp chamber opposite the mesiodistal cusp c) under the distolingual cusp 6. In root canal filling, cement protrudes beyond the apex, what treatment should be done? a) wait and recall for x-ray* b) surgical, apicoectomy 7. What is the effect of corticosteriod in the pulp? Anti-inflammatory 8. Teeth with endodontic treatment a) is the same as one without treatment b) is stronger as one without treatment c) is weaker as one without treatment* 9. Reversible pulpitis diagnosed by application of: a) hot b) cold* c) sweet 10. Fistula a) no special treatment* b) cauterization with phenol c) curettage of the fistula 11. To avoid strip perforation in curved root canals, do filling in which direction - anticurvature filling (mesial root surface) maintain mesial pressure on the enlarging instrument to avoid the delicate danger zone of the distal wall 12. Acute Apical Abscess treatment a) Root filling and immediate apicoectomy b) Incision and drainage (can also be from the tooth) alone* c) Extraction 13. What is the treatment of caries exposure in upper premolar of 20 years old patient with sharply curved apical part H. root a) Pulpectomy b) Partial pulpectomy? c) Pulpotomy d) Pulp capping 14. Pain in tooth with pulpal inflammation and periapical involvement (percussion sensitive localizable) a) patient has difficulty to localize b) It presents referred pain c) It has tenderness to percussion*

d) It relieves with heat e) It relieves with cold*? 15. Throbing pain which increases with heat and is sensitive to percussion: a) cyst b) occlusal trauma c) advanced pulpitis* 16. Intermittent pain of the upper premolar, MOD restoration, Split cusp suspected diagnosis ( multiple response) a) radiograph b) history* c) visual inspection d) percussion e) pressure on each cusp* 17. In case of a cyst of lateral incisors, where would the lesion be positioned? a) nasopalatine b) buccal c) labial d) palatally*? 18. Strip perforation: mesiobuccal of upper 6s or mesial of 6s* 19. A cyst of was found between 2 upper incisors, what kind of cyst would it be? a) Primordial b) Nasopalatine* c) Lateral periodontal d) Radicular 20. Symptoms of internal resorption of RC is: a) no pain*/ completely symptom free or mild pain at the tolerable level* b) pain in one quick stab c) pulpal discomfort in the morning d) constant unexplained toothache 21. In the RCT you may use corticosteroids for: a) elimination of pain* b) antibiotic therapy 22. If corticosteriods are used as a component of root canal medicaments: a) microorganisms are destroyed b) leukocytic infiltration results c) antibacterial action is enhanced d) exacerbation of infection may occur*

23. You open the orifices in root canal and there is small periapical radiolucency. You found it completely obstructed with secondary dentin. What is your plan? a) fill and obscure the lesion b) file and use the reamer to open the canal c) apicoectomy and retrograde filling*

24. Internal resorption because of: a) systemic disease b) trauma* 25. Access cavity for:determination of the orifice of the canal 26. the enamel rods in permanent teeth are perpendicular to surface tangent 27. Radiographic examination revealed non-vital two upper central incisors. Clinical assess to the pulp chamber shows calcific the best management to the case a) try to file the root canal and proceed to the treatment b) try (EDTA) periapical access and retro grade amalgam filling* c) put dressing into the cavities and observe 28. The length of the RCT is 0.5-1.5 mm short of the apex 29. Internal resorption of RC is usually a) painless/ asymptomatic* b) painful 30. Which of the following is not irritant to the periapical tissue a) Nahypochloride b) Gutta percha points*? c) Gutta percha points and cement 31. If the canal is too narrow you may use: a) reamer b) K-type file* 32. The safest instrument for removing the pulp from a very fine canal is a: a) barbed broach b) small k-type file* c) tempered universal hedstrom file d) smooth broach 33. In upper first molar you may find two root canals in mesio buccal root

34. The most common cause of failure of RCT is caused by a) Incomplete obliteration of the root canal/ underfilled* b) Overfilled c) Coexistent periodontal-periapical lesion d) Occlusal interference e) An apical cyst 35. The crown of an endodontically treated maxillary lateral incisor is fractures near the gingival margin. The coronal end of the silver used in filling that canal is visible at that level. The finding reveals that the existing root canal filing meet the criteria to be judged successfully. Which of the following is the best way to obtain the neededpost space a) to prepare it alongside the silver cones using burs and peeso reemer b) to carefully grind away the coronal part of the silver cones using round bur or end cutting bur = can press it apically c) to remove the silver cones and re-treat the canal using gutta percha technique and then to create the needed post space d) to remove the silver cones, notched it 4 mm from the apical end, coat it with freshly mixed sealer, replace it in the canal and twist-off the coronal segment* 35. How do you prepare the floor of the pulp chamber in ______? e) swab and dry with cotton wool and excavate* f) use round-bur to flatten it g) use flat fissure bur to make it flat h) undercuts 36. The ideal preparation of root canal filling cement? a) radio opacity b) non-irritant* c) quick d) adherent 37. a patient reports to your office for routine restorative therapy. After the usual examination, radiographs, and so on, you see a maxillary lateral incisor overfilled (1 mm) with gutta percha. There is a radioluscent area at the apex; however the endodontic therapy was completed one month ago. You would: a) redo the tooth with nonsurgical endodontic approach b) take recall radiographs every 3-6 months observation* c) perform surgical endodontic therapy (retrograde filling) d) prescribe antibiotic 38. Incisive foramen is superimposed over apex of root on radiograph it may be mistaken for: a) cyst*

b) cementoma c) odontoma 39. An apical radiolucent region when open the canal is obliterated. What would you do: retrograde filling

40. Access cavity in central incisor (upper) a) oval b) round c) triangular* d) none of the above 41. What is the best method for preparing the floor of the pulp chamber prior to reaming the canals for a tooth that requires endodontic treatment: a) only swab the floor and clean with excavator* b) clean and smooth the floor with a fissure bur c) squar the floor with a fissure bur 42. Strip perforation common in mesiobuccal root of upper first molar 43. The most common feature of acute apical periodontitis? a) tenderness of the tooth to pressure* b) intra-oral swelling c) intermittent pain d) sensitivity to heat e) sensitivity to cold 44. The response of the pulp is pain 45. Internal resorption is related to CaOH 46. Replantation external resorption ( bad prognosis) 47. Replanted a tooth after 25 minutes, rct will be done after a resorption or necrosis happens with evident radiographic findings 48. Avulsed teeth came to dentist after 25 mins with open apex RCT will be performed after one sees a radiolucent area 49. Up to which point do you fill the root canal?dento-cemental junction (1.5 mm from apex)

50. Most common cause of RCT to fail short filling, incomplete obturationcalcified canals, continue filling RCT 51. The most desirable form of tissue response at the apical foramen following RCT is by: a) cementum deposition into the apical foramen* b) formation of a connective tissuecapsule over the foramen c) proliferation of epithelium from the apical periodontal ligament d) proliferation of the periodontal connective tissue into the apical foramen 52. Following auto-transplantation of teeth to a prepared socket, what would you expect after 12 months? a) the formation of a standard pattern of periodontal membrane b) some fibrous tissue material, varying degrees of ankylosis* c) internal resorption of the tooth d) a well formed lamina dura 53. Vitality testing using an electric pulp tester a) reveal that a recently erupted tooth is vital b) indicate a sensory nerve response to a current of electricity* c) has no limitations d) reveals a potentially necrotic pulp* 54. The loss of translucency in tooth is an indication of all the following except: a) pulpal hyperemia* b) pulpal death grey c) pulp stones dull d) internal resorption pink e) calcified pulp chamber - yellow 55. Leadermix in RCT for relief of pain, because of the presence of a) antibiotics b) corticosteriods* 56. The main ingredient of corticosteriod pulp dressing is a) Prednisolone* b) tetracycline c) neomycin 57. In infected root canal, the 2 most common microorganism are ? a) streptococcus and staphylococcus* b) staphylococcus and lactobacillus c) staphylococcus and spirochetes d) lactobacillus and cornebacterium

58. The technique of placing gutta-percha cones against the RC walls providing space for additional GP is termed : lateral condensation 59. Pain from pulpal inflammation with periapical involvement is usually characterized by: a) inability of the patient to localized it b) referral to the other side of the mouth c) tenderness to percussion of the tooth involved*

60. Cervical resorption in bleached pulpless tooth may result from a) bleach placed apical to the CEJ* b) tooth becoming pulpless before patient reaches age 25 c) a defect in the CEJ d) All of the above Note; avoid placing bleaching solution at the CEJ to prevent cervical resorption. 61. The most common treatment for cervical resorption following intracoronal bleaching is: a) surgical repair * b) orthodontic extrusion c) extraction d) calcium hydroxide therapy 62. After bleaching of teeth with non-vital pulp, the dentinal tubules are sealed using: a) a cavity varnish b) an unfilled resin c) glass ionomer cement *? d) Zinc phosphate cement 63. How to detect vertical root fracture from x-ray periodontal area 64. When a normal pulp is subjected to thermal test, it: a) will not respond to heat b) will not respond to cold c) will respond painfully; but will return to normalshortly after the stimulus is removed* 65. Acute apical abcess is best treated by a) Root filling and immediate apicoectomy b) Endodontic therapy or extraction c) Incision and drainage alone* Note: treatment consist of establishing drainage and controlling the systemic reaction. When symptoms subsides the tooth should be treated endodontically by conservative means. (Grossman)

66. Following a calcium hydroxide pulpotomy, the dentist would expect a dentine bridge to form at a) A level somewhat below the amputation* b) Exact level of amputation c) Apical region of the root 67. In the file No. 15, it means that the diameter of the working tip is 0.15mm 68. The cemento-dentinal junction has been shown in an average to be 0.5 mm to 1.5 mm 69. Internal resorption of root, which is correct a) radiographically appears as super imposed over the canal b) it develops as a result of previous trauma* c) it occurs in elderly patients d) it accompanies pulp stones 70. Sterilization of reamers and files in glassbeas sterilizer or hot-salt sterilizer a) 450degrees F for 5 secs* in glassbead sterilizer b) 150degrees F for 5 secs c) 150 degrees F for 30 secs d) 450 degrees F for 5 secs 71. The commonly used irrigation solution for root canals are a) H2O2 b) Normal saline c) Sodium hypochlorite* 72. Sterilization of Gutta percha: a) by chemicals 55% sodium hypochlorite for 1 minute* b) dry heat 73. What do you expect to find after a pulpotomy with CaOH in a central incisor after sometime? Reparative dentine bridge under the cement 74. What is the first stage of the pulp after a pulp capping? Chronic-inflammation reaction or acute? 75. What material would you use for pulp capping? Ca(OH)2 76. If a tooth has a fracture at the apical 1/3, What would you do, observation and radiograph and other test 77. Final materials used for endodontically treated deciduous molar is: a) amalgam

b) composite resin c) glass IC d) wrought base metal chrome* 78. How would you definitely diagnose the nonvitality of a tooth? a) electric pulp testing b) hot gutta percha test c) cold test d) radiolucency at the apex* 79. Primary endodontic Lesion, secondary periodontal involvement usually a) endodontic treatment only b) periodontal therapy only c) periodontal therapy following endodontic therapy* d) endodontic therapy following periodontal therapy 80. A cyst at the apex of an upper central incisor measuring 1 cm in diameter in visualizaed in radiograph. You would expect to remove this lesion: making a muco periosteal flap and removing the cyst following endodontic treatment apicoectomy 81. To diagnose a fractured cusps by : pressure 82. Vertical fracture of the root, to achieve a radiographic view: a) periapical b) displacement of fragments 83. The emergency treatment of painless necrotic pulp a) drainage through the canals b) none* 84. Swelling after RCT caused by: entrapped bacteria pushed at the apical area

Traumatology 1. A patient knocked out a tooth while running into the kitchen during the half-time show of a Monday night football game. Arrange in order, those procedures you would most likely follow in replanting tooth: a) prescribe an antibiotic and advise the patient to check his tetanus immunization schedule b) replace the tooth in the alveolus and reduce any alveolar fractures c) advise the patient to place the tooth in the socket or hold it in his mouth and drive to the office

d) immobilize the tooth e) perform conventional endodontics through the crown of the tooth at a later date f) amesthetixe the area surrounding the tooth. Note: 3, 6, 2, 4, 1, 5

2. Apical third fracture of the root a) CaOH pulpotomy and restoration of the fracture by band b) Leave it but monitor by radiographs* c) Pulpectomy d) Extraction 3. A month ago a 20 years old patient sustained a traumatic blow to a maxillary central incisor. The tooth is asymptomatic and gives a normal vital response to heat, cold and electric pulp tester. A radiograph shows a horizntal fracture on the apical third of the root with the segments in close apposition. The tooth is not mobile. Treatment is: a) extract the tooth b) initiate conservative rot canal terapy c) instrument and fill the root canal and remove the fractured segment surgically d) do nothing at present, plan to check the vitality and radiographic appearance periodically* note: a horizontal fracture associated with a pulp-containing tooth appears to have a more favorable prognosis than does a horizontal fracture associated with a pulpless tooth 4. Middle third fracture of the root? Observe if mobile splint for a month 5. What should the immediate treatment of a tooth that has sustained a fracture to the middle tird of the root include? a) pulpectomy to the coronal portion and apicoectomy of the root portion b) pulpectomy to both portions of the tooth c) splint* d) no treatment required note: the emergency treatment of a root fracture involves the apposition of the fractured parts, immobilization and control of infection 6. Which of the following areas of root fracture is least conducive to a favorable prognosis? a) apical third b) middle third

c) cervical third* d) prognosis is equal in all cases provided te tooth remains vital note: Prognosis is least favorable when the fracture occursin the cervical third of the root. This is because of the difficulty of stabilizing the crown segment and because of the easy access of oral microorganisms to the fracture areas. 7. What is the best result you would expect from apexification at periapex? a) calcified tissue at the periapex (cementum formation)* b) connective tissue 8. The most desireable form of tissue response at the apical foramen following root canal therapy is by: a) cementum deposition into the apical foramen* b) formation of a connective tissue capsule over the foramen c) proliferation of epithelium from the apical periodontal ligament d) proliferation of periodontal connective tissue into the apical foramen 9. A patient comes to you at age 8 a fractured 11. 25 mins after accident with a large pulp. What would you do? a) pulpotomy with formocresol b) pulpotomy with CaOH* c) pulp capping using calcium hydroxide d) pulpectomy and immediate root filling e) pulpectomy and apexification 10. A patient comes to you with a fractured central incisor at age 8, 3 hours after accident, what will you do? a) Apexification b) Pulpotomy with calcium hydroxide* Note for #6 and 7, Pulpotomy with calcium hydroxide is indicated when there is a relatively large pulp exposure and the patient is seen within 72 hours or 3 days. While the pulp is infected, it is considered recoverable. A tooth with a wide, immature apex is considered a good candidate for this technique because of the recuperative powers and degree of vascularization of the young pulp. 11. An eight year old child sustains a fracture of a maxillary permanent central incisor in which a large portion of the pulp is involved. The child was not in acute pain and did not present to the dental office for several days. What treatment of the pulp should be performed at this time? a) pulpotomy using calcium hydroxide b) pulp capping using calcium hydroxide c) pulpotomy using formocresol d) pulpectomy and immediate root filling* e) pulpectomy and apexification

note: pulpectomy is indicated if the exposure is no longer than 72 hours duration 3 days, since the pulp is generally infected beyond recovery. The apexification technique will allow apical constriction to occur in a non-vital immature permanent tooth. The root end narrows sufficiently to subsequently enable complete closure of the root apex by conventional endodontic procedures. 12. An 11 year old presents to the clinic because of a fracture on the lateral incisor immediately followng bicycle accident. Clinical examination shown that the pulp is not exposed and is vital which is your preferred treatment? a) Placement of an orthodontic band with ZnPO4 cement b) Formocresol Pulpotomy and stainless steel crown c) Placement of CaOH2 preparation on exposed dentine and then restoration with an acid bonded composite resin* d) Glasss ionomer cement e) Pin retained silicate anterior restoration 13. When you get a patient with intruded teeth (CI) what is the first thing you will do? a) replace position b) advice the patient about the consequences c) leave it and observe d) x-ray* 14. When a split cusp is suspected in a premolar that has an old amalgam. How would you diagnose it accurately? a) percussion- pressure under the cusp x-ray 15. A patient 8 years old comes with an intruded incisor. What is your first step? a) radiograph, periapical 16. A healthy pulp responds to an injury by: a) An effective collateral circulation to transport inflammatory elements to the area b) Deposition of well mineralized and highly tubular reparative dentine c) Initiation of an inflammatory response followed by partial or complete ___ d) The formation of reparative dentine the pulpal surface corresponding to area of irritation* 17. Two hours elapsed before a patient with an avulsed tooth was able to see a dentist. The dentist replanted and splinted the tooth and performed nonsurgical endodontic therapy. Which of the following probably resulted after the treatment a) radicular cyst formation b) condensing osteitis c) external root resorption* d) chronic periapical pariodontitis

18. The immediate concern in the management of facial trauma should be: a) replacing blood loss b) reduction and fixation of fracture c) airway* d) neurologic consultation e) prevention of loss of cerebrospinal fluid 19. Nine year old child comes to surgery with trauma exposure of one incisor, your treatment of choice a) Pulpotomy* b) Pulp Cap c) Pulpectomy d) Corticosteroid therapy 20. Following auto-transplantation of teeth to prepared socket, which would you normally expect after 12 months a) Formation of Standard pattern of PDL b) Chaotic set of periodontal elements filling the periodontal space c) Some fibrous tissue material with varying degrees of akylosis* d) Internal resorption of the root e) Well formed lamina dura 21. In the horizontal root fracture, the most important a) keep the vitality of the pulp b) keep the tooth immobilized* 22. Which artery/s supply upper and lower teeth a) internal carotid artery b) branches of maxillary artery only* c) maxillary and lingual ateries d) lingual artery only 23. For small incisal edge fractures, material with the greatest resistance to internal fractures a) glass ionomer b) silicophosphate material c) silicate materials poor wear resistance d) resins* 24. The most favorable root fracture 1. apical third* 2. middle third 3. coronal third 25. Vertical fracture poor prognosis

26. Good prognosis following fracture of the root of maxillary permanent incisor is a) apical one third of the root* b) coronal one third of the root c) middle one third of the root 27. Patient received a heavy blow to right body of the mandible sustaining a fracture there. Where should you suspect a second fracture. Most likely to be present: a) the symphysis region b) left body of mandible c) left subcondylar region* d) right subcondylar region e) both subcondylar regions 28. A young patient broken his permanent incisor, pulp is exposed, he presented on the next day. What would you do? a) pulpotomy if vital b) apexification if not vital* 29. A preschool child has an intruded upper incisor, what would be your treatment: a) x-ray* b) put in place and splint c) control after one month 30. What kind of fear, the child feels on the dental chair: a) fear of the unknown* b) fear of pain 31. A patient aged 9 years received a blow, fracturing an upper central incisor. Small pulp exposure is observed the following day when the patient attends surgery which is the most appropriate approach to treat? a) a pulp capping with a corticosteriod dressing b) a pulp capping with calcium hydroxide dressing c) pulpotomy* d) pulpectomy 32. Following a calcium hydroxide pulpotomy, the dentist would expect a dentin bridge to form at a) a level below amputation site* b) alevel halfway between amputation and apex c) adjacent to lining 33. 11 year old present fratures lateral incisor pulp not exposed and vital. a) Caoh and acid etch + composite* b) Formocresol Pulpotomy and stainless steel crown c) Ortho band with ZnPO4 cement

Anatomy 1.Which nerve may be stimulated when touching the dorsum of the tongue e) hypoglossal n* f) glossopharyngeal n (pharynx) 2. In the mature tooth the forming cells are absent in which of the following structures: a) enamel* b) dentin c) cementum 3. Dentin and enamel differ with regards to other organic matrix in that enamel: a) does not contain collagen b) contains collagen c) contains elastin d) does not contain elastin *enamel of ectodermal origin does not contain collagen as part of the matrix 4. Histology of dentin a) change in composition and amount increase age* b) no change 5. Dentin in which the tubules have become obliterated with calcium salts, rendering the detin equally refractive throughout Transparent dentin (Sclerotic Dentin) 6. Dentin deposited slowly during the functional life of the tooth, not deposited evenly along the pulpal wall regular secondary dentin 7. dentin in which the dentinal tubules are filled with gaseous substance following degeneration of the odontoblastic processes Dead Tracts 8. Dentin deposited before the apical foramen was formed primary dentin 9. Dentin deposited following severe damage of the odontoblast by extensive wear, caries or operative procedures. Deposited by damaged or newly differentiated odontoblasts, this tissue is highly irregular and localized to the area of dentin tubules invoved in the original insult Reactive, Reparative, Irregular secondary dentin. 10. How does the thickness of dentin in primary to permanent teeth 1/2

11. Fontanelles a) Ant and Post, where parietal meets occipital and frontal* b) Ant, lateral and posterior lateral parietal and lateral 12. Commonly missing teeth = 3rd molars > premolars > lupper lateral incisors a) first lower 2nd premolar b) second upper lateral incisor c) third 2nd premolar upper 13. Main function of the zygomatic process? a) to insert the masseter muscle* b) to insert the temporal muscle 14. Enamel rods: follows a curving path thru 1/3 of the enamel end to the DEJ but more direct path up to enamel surface 15. Gnarled enamel : group of enamel rods that may entwined adjacent groups of rods and they follow a curving irregular path towards the tooth surface and occurs near cervical and incisal and occlusal regions. Optical appearance of enamel is gnarled enamel. 16. Hunter schreger bands light and dark zones of varying widths and have slightly different permeability and organic content. Found in different areas of each tooth 17. Enamel tufts: hypomineralized or consist of hypocalcified enamel rods and interprimatic substances. , projects from the DEJ to the enamel about 1/5 or 1/3 the thickness. 18. Enamel lamellae, thin leaflike structures that extend from the enamel surface toward the dentinoenamel surafec toward the dentinoenamel junction, consist of organic materials with little minerals 19. Enamel cuticle Nasmyths membrane , covers the entire crown until removed by mastication 20. Enamel spindles projectins of the odontoblastic processes in the enamel. 21. Enamel Lamellae, thin leaflike faults between enamel rod groups extending from enamel surface to DEJ sometimes in the dentin, mostly inorganic mat, may lead to entry of bacteria and caries. 22. Enamel spindles odontoblastic process sometimes cross DEJ into enamel and called enamel spindles when ends are thickened. May serve as pain receptors, thereby explaining channel sensitivity

23. Incremental striae of Retzius variations in mineralizations and can be considered as growth rings 24. Nasmyth mebrane/ primary enamel cuticle final ameloblast covering, covers the newly erupted tooth, worn out by mastication 25. Tomes fibers, odontoblastic process that is left in a mineralized tubule of dentin 26. The normal range of gingival depth is: a) 1-2 mm b) 0-3 mm* c) 2-3 mm d) 0-5 mm 27. The root surface are of maxillary teeth ( per cemental area) a) canine> second premolar> central incisor* b) canine> central incisor> second premolar 28. Pericemnatal area of mandibular teeth areas a) canine> first premolar > lateral incisor b) first premolar> canine> lateral incisor* 29. Mainly in what form Ca is found in enamel? a) Ca hydroxy apatite b) Ca Fl c) Ca Cl2 30. Which statement is correct? a) remnants of odontoblastic process forms enamel cuticle b) last secondary odontoblast cementum c) last secretion of ameloblast produces acquired enamel cuticle* d) remnanst of ameloblast produces primary enamel cuticle? 31. Composition of enamel following which one is 97% of inorganic substance? a) calcium b) hydroxyapatite* c) Ca3(PO4)2 32. The direction of enamel prisms in permanent teeth are: at the right angle to the tangent of the outer surface of the tooth 33. What makes the mandible in a depressed position, except? a) contraction of lateral pterygoid b) contracxtion of suprahyoid muscle c) contraction of mylohyoid muscle

d) contraction of temporalis muscle* e) relaxation of muscle and let gravity depress the mandible

34. Opening the mouth can be carried out by all of the following muscles except: a) mylohyoid b) anterior belly of the digastric c) platysma d) lateral pterygoid e) temporalis* 35. Any muscle attached to the lower jjaw and whose line of action passes below the axis of roattion of the temporomandibular joint can effectively open mouth. 36. Incomplete Lip defined as the lip which does not meet when mandible is at rest* 37. Lip to tongue swallowing associated with incomplete lip* 38. Incomplete lip are associated with increased face height 39. Cause congenital absence of lateral incisors: initiation stage 40. What is the proper sequence of the histological stages of tooth development: initiation, proliferation, histodifferentiation, morphodifferentiation and mineralization 41. Function of the anterior belly of the digastric muscle is: a) to depress the mandible* b) to elevate the mandible 42. Alveolar process is composed of the alveolar bone proper (inside or inner wall of the socket) cribriform plate (lamelated and bundle bone) and the supporting bone 43. Alveolar bone is characterized by: Haversian canals around bony canals or bundle bone 44. Attrition process of normal wear on the crown or the wearing of tooth structures resulting from masticatory friction 45. Attrition wearing of the occlusal surface because of certain factors eg bruxism 46. Causes of attrition a) bruxism* b) improper tooth brushing (abrasion) c) coarse food* d) ill fiiitng clasp (abrasion)

47. Abrasion is the mechanical wearing of the tooth by physical agents acting as abrasives 48. Erosion is the chemicomechanical wearingt of the gingival third of the tooth, the etiology of which is unknown. 49. Frenae is a ? fribrous tissue? a) keratinized epithelium b) squamous epithelium c) white fibers d) collagenous fibers 50. The main component of enamel is d) calcium phosphate e) hydroxyapatite* f) calcium carbonate note: enamel 95-98%, Dentin 75%, 45-50% (inorganic) 51. Hydroxiapatite which is basic structure of bone mineral has: Ca10(PO4)6(OH)2 as a chemical prototype. 52. Which of the periodontal fibers have no attachment into bone of the alveolar process? g) oblique h) free gingival i) transeptal* j) alveolar crest 53. Periodontal fibers a) taut b) wavy* c) firm d) stippled 54. All of the following are keratinized except: a) crevicular epithelium* b) palatal epithelium c) alveolar mucosa d) free gingival e) attached gingiva 55. Fenestration alveolar bone defect which predisposes tooth to gingival recession, most common sites are; buccal surface of the upper and lower canine, lower incisors, palatal surface of upper first molar

56. Most exposed pulp in permanent teeth mesiobuccal of lower first molar 57. Cemento-Dentinal Junction : 0.5 1.5 mm of anatomical apex 58. Peri-cemental area of Max. teeth, canine> 1st premolar> central 59. When compared with young pulp, the aging pulp contains fewer cells and more collagen 60. Direction of enamel prism is perpendicular to the tangent of the outer enamel surface 61. lateral canals are usually found in: the apical third of the root 62. Cause of the development of lateral canals : cracks of hertwigs epithelial root sheath 63. The muscle which moves the bolus of food Buccinator muscle 64. Which cusp is more liable for cracked tooth syndrome DL of upper 6 and 7 65. Oligodontia problem with initiation of the tooth bud 66. Gemination twining correct no. of teeth, large tooth, looks split 67. Fusion no. of teeth decreased by one. Tooth appears joined and xray shows 2 canals 68. The mandibular division of the trigeminal nerve: a) exits from the skull through the foramen ovale b) contans both afferent and efferent nerve fibers c) supplies the muscle of mastication as well as the mylohyoid muscle and the anterior belly of the digastric muscle among others d) enters the mandible through the mandibular foramen and has an exit at the mnetal foramen e) all of the above are correct note: only the mandibular division of the trigeminal nerve ahs a motor fiber. 69. Which is not supplied by the 3rd division of the trigeminal nerve buccinator muscle 70. The anterior 2/3 of the tongue taste ( chorda typani nerve branch of VII ), motor is hypoglossal nerve XII 71. Branch IX (Glossopharyngeal nerve)innervates the mucosa and both general sensaion and taste of the posterior of the tongue

72. The lingual nerve is anterior and medial to the IAN 73. After the facial nerve exits through the stylomastoid foramen, it ramifies in the substance of the parotid gland in five branches. All are correctly listed below except: a) zygomatic b) temporal c) cervical d) buccal e) auricuar * f) mandibular 74. The union of roots of adjacent teeth through the cementum is referred to as; concrescence 75. Haversian System cells in the middle surrounded by bone in a circular pattern 76. the periodontal membrane, what epithelial cells you can find: epithelial rests of malassez 77. Loss of several teeth produces a) TMJ dysfunction b) Periodontal pocket c) Premature contact d) Loss of proximal contact*

Anesthesia 1. When is it advisable to do dental treatment under GA?mental retardation* 2. Anesthetic gas not to use less than 30% oxygen 3. mandibular branches of the trigeminal not supply the buccinator, nope, facial nerve 4. Buccal nerve, cheeks = long buccal nerve of V3 (sensory) Buccal branches of VII (motor) 5. Lingual nerve, branch of the mandibular nerve 6. What is the ain reason for premedication before general naesthesia? To provide a good induction

7. Why is nitrous oxide is not use as general anesthetic? a) Oxidation to tissue not enough b) difficult to maintain O2 c) poor analgesia d) toxic to liver e) not used alone, because it is not as potent and lack of muscle relaxation* 8. For long action (180 minutes) of LA Bupivacaine, Etidocaine 9. Medium acting (90-150 minutes) prilocaine and lidocaine 10. Short acting (45-75 minutes) 45-75 minutes, idocaine, mepivacain, prilocaine and procaine 11. Ultra-short acting less than 30 minutes, procaine, prilocaine and lidocaine without vasoconstrictors. 12. Ester group cocaine and benzocaine (topical), procaine(Novocain), tatracaine (Pontocaine), propoxycaine (Ravocaine, can be used in low ph and cannot be used as a topical), 2-chloroprocaine (Nesacaine) 13. Amide group Bupivacaine (Marcaine), Etidocaine(Duranest), Lidocaine (Xylocaine) Mepivacaine(Carbocaine), Prilocaine (Citanest) 14. Tetracaine (Pontocaine) ester, is a high potency, high toxicity, local anesthetic that has been known to rpoduce toxicity following even oropharyngeal topical administration. 15. LA adverse effect intravascular injection 16. Transmisison of pain in dentinal tubules by hydrodynamic pressure* 17. Gate control theory of pain: one hypothesis of pain modulation is based upon the inhibitory-excitatory interaction of afferent fiber synapses 18. The nerve supplies the TMJ a) auriculotemporal nerve and masseteric branch of the mandibular nerve* b) facial nerve c) nerve for the masseter muscle* 19. General anesthesia is often used for the patients a) mental retardation* b) spasm patient* c) complicated operation* d) children

20. Which is the most important to define the puncture point in dental inferior nerve block? a) pterygotemporal depression b) buccal pad c) pterygomandibular depression* d) anterior border of the mandible e) internal oblique ridge 21. Nitrous oxide alone is not in used in general anesthesia because a) difficult to mainatain O2 b) is explosive c) toxic to liver? d) poor analgesic e) all of the above f) lack of potency and muscle relaxation* 22. To prevent adverse effect of the aneasthetic solution it should be: a) injected rapidly b) injected slowly c) aspirate before depositing the solution* 23. Facial paralysis 10 mins after inferior dental nerve block, which one of the following is correct?, injection to the parotid gland 24. Posterior Superior alveolar nerve supplies a) maxillary molars except mesio-buccal root of the first molar* b) maxillary and first premolars 25. How many ml of lignocain can be injected safely? 10 ml cartridge?, depends on weigth, 5mg per kilogram 26. When should you extract the first maxi molar?, Which nerve should you anesthetize? a) posterior superior alveolar nerve* b) middle superior alveolar nerve* c) greater palatine nerve* d) incisive palatal nerve 27. NO2 is contraindicated in: a) pregnancy b) heart murmur c) heart diseases d) hypertension

28. Nitrous oxide (N2O) in 70 per cent concentration in oxygen: a) is a potent anesthesia b) produces good muscle relaxation c) produces slow recovery d) reduces the capacity of red blood cells to carry oxygen e) has low toxicity* note: Nitrous oxide in 70 percent concentration is one of the safest anethetic agents 29. In concentrations commonly used for inhalation sedation, N2O will not produce: a) hypnosis b) analgesia c) anesthesia* d) amnesia e) high margin of safety note: N2O does not usually produce anesthesia at concentration under 80 percent. 30. Patient contraindications or precautions in the sue of N2O for inhalation sedation all of the following except: a) severe hypertension b) iron deficiency anemia c) coronary artery disease d) pregnancy* e) concomitant antipsychotic drugs note: Normal pregnancy does not contraindicate the use of nitrous oxide which is, in fact, used in obstetrics. 31. Gas anesthetic Halothane hepatotoxic reaction 32. A general anesthetic that has been implicated in producing liver damage as an allergic manefestation on repeated administrtation is : a) Halothane* b) Nitrous oxide c) Ethylene d) Thiopental e) Divinyl ether Note: Halothane has been reported in producing liver damage following a second administration within a two year period.

33. In the inferior alveolar nerve block the needle passes between and closes to two muscles a) middle pterygoid and lateral pterygoid b) middle pterygoid and superiorconstrictor c) temporal and lateral pterygoid d) temporal and medial pterygoid* 34. Which of the following nerves would be adequately anesthetized for removal upper first molar a) anterior palatine* b) posterior palatine c) middle superior alveolar* d) anterior superior alveolar e) posterior superior alveolar* 35. Maximum safe dose of lidocain 2% without vasoconstrictor in a 70 kg weight patient (maximum 4.4 mg / kg) a) 2.2 ml b) 10 ml* c) 25 ml d) 50 ml e) 100 ml 36. Safest amount of 2% LA with vasoconstrictor for a 70 kg adult is (7 mg/kg) a) 5 ml b) 10 ml* c) 50 ml 37. Transmission of pain (Hypothesis of pain modulus) is by:gate control theory* 38. Which nerve fibers carry pain stimulation in the pulp? a) Dorsal C fibers* (small, unmylinated nerve fibers from .05-1 micron in diameter, conduct the slow or second pain at a rate of 0.5 to 2 meters per second) b) A delta fibers* ( large myelinated fibers from 3 to 20 micra in diameter, conduct impulse at a rate of 3-14 meters per second 39. The following landmarks used in making an inferior nerve block injection. Having established the height of the horizontal plane in which the injection will be made. Which landmark defines the puncture point most accurately: a) pterygotemporal depression b) pterygomandibular ligament raphe*

40. Chief cause of adverse reaction to LA a) Vascular injection* b) Hypersensitivity (rare) 41. Which of the following is correct about N2O a) N2O has high analgesic property and low anaesthetic at its minimum anaesthetic dose* 42. A patient taking guanethidine to control severe hypertension. His physician advices you not to use epinephrine in you local anesthetic. Your choice of anesthetic for a complicated extraction should be: a) procaine 2% with phenylephrine 1:2500 b) lidocaine 2% without vasoconstrictor c) mepivacaine 3% without vasoconstrctor* d) prilocaine 4% without vasoconstrictor e) mepivacaine 2% with levonordephrin 1:20,000 note: neither lidocaine nor prilocaine without vasocontrictor provides sufficient duration of anesthesia, and both phenylephrine and levonordefrine are catecholamines with alpha-adrenergic activity. 43. A safe suggestionfor anesthesia of questionable patient woud be the administration of not more than two cartridges containing 1:100,000 epinephrine or a comparable amount of related drugs. Lidocaine (Xylocaine), mepivacaine (Carbocaine)and prilicaine(Citanest) without epinephrine would be a satisfactory choice for patients who have cardiac condition or hypertension. 44. Hypertensive patient 3% pilocarpine with fellypressin 45. Mepivacain 3% is vasodilator, metabolized in liver and excreted in urine, indicated for hypertensive patient 46. Prilocaine with felypressin is contraindicated for pregnant women 47. Contraindication for LA is hypersensitivity reaction to the drug 48. To reduce the risk of side effects from a local anesthesia injection. You should do the following except: a) aspirate before injection b) use the smallest possible volume c) use the weakest efficient percentage strength d) inject rapidly (should inject slowly always)* 49. The blood supply of all the teeth by: small branches from the maxillary artery

50. Which is best to administer if one were to anticipate both severe and prolonged post-operative dental pain: Bupivacaine 51. Indication for general anesthetic for dental treatment include: a) mental retardation* b) cardiac insufficiency c) complexity of surgery* d) spastic patients* e) old age 52. Nitrous oxide inhalation sedation is contraindicated to which of the foloowing cases a) Pregnancy b) coronary heart diseas c) iron deficiency anemia* d) mental retardation e) none of the above 53. Which of the following least to cause toxicity from local anesthesia injection: a) injecting in the supine position* b) injecting into vascular area c) injecting without vasoconstriction d) intravenous injection 54. One of the chief cause of reaction to local anesthesia is a) contamination of solution b) deterioration of solution because of its short shelf life c) deposition into vein or artery* 55. Toxic dose of lignocaine (2%) with 1:800000 adrenalin not exceeding 500 mg lidocaine. a) 2 ml b) 10 ml c) 22 ml 56. What is the most usual failure of mandibular block? a) too low* b) too mesially c) too distally* d) too high 57. Which of the following are not supplied by the mandibular division of the trigeminal? a) anterior 2/3 of tongue b) anterior part of digastric c) masseter d) mucosa over busccinator

e) buccinator* 58. In inferior dental block, the needle passes in between and close to 2 muscles a) medial and lateral pterygoid b) medial pterygoid and superior constrictor c) temporal muscle and medial pterygoid* d) buccinator and superior constrictor muscle 59. Muscle of the pharynx a) temporal muscle and lateral ptyrigoid b) temporal muscle and medial pterygoid c) buccinator and superior constrictor muscle* 60. Muscle that acts on the border of the mandibular denture (complete) a) temporalis b) mentalis c) lateral pterygoid d) levetor anguli oris e) orbicularis oris* 61. When injected without vasoconstrictor, the maximum safe dose of 21 lignocaine solution for 70 kgm adult is a) 2.2 ml b) 10 ml c) 25 ml 62. A dentist administers three cartridges of lidocaine 2 percent. Approximately how much lidocaine did the patient receive? a) 0.05 mg b) 10 mg c) 54 mg d) 60 mg* e) 108 mg note: each ml lidocaine 2 percent contains 20 mg of lidocaine and each cartridge contains 1.8 ml 63. Approximately how many cartridges of idocaine 2 percent could be injected safely into a patient weighing 70 kilograms? a) 3 b) 5 c) 7 d) 9* e) 12 note: the maximum safe dosage of lidocaine is 5 mg per kg 70 kg x 5 mg = 350 mg divided by 20 mg per ml = 17.5 ml divided by 1.8 ml per cartridge = 9.7 but sa book without vasoconstrictor 4.4 mg/kg, with vasoconstrictor 7 mg/ kg

64. The longest duration of anesthetic action achieved by: a) Bupivacaine (marcain) 8hrs without vaso, 10 hrs with vaso b) 3 hours only or 180 minutes with vasoconstrictor* 65. When you do injection near the anterior border of the ramus of the mandible 1mm above the occlusal plane of the molar, which one of the following nerves do you inject? a) lingual nerve b) inferior dental nerve c) long buccal nerve* d) mental nerve 66. Anatomic landmark of the inferior dental nerve block is used to punch the needle a) internal obliqe ridge b) external oblique ridge c) pterygoid mandibular raphe* d) retromolar fossae 67. Which of the following local anesthetics does not have a chemical configuration like the others? a) Lignocaine b) mepivaciaine c) Prilocaine d) Amethocaine* 68. The common failure of the inferior dental nerve block a) injection too low b) injection too high c) insufficient anaesthetic solution d) injection too back*

Pediatrics 1. Where would you affix a space maintaner if at 8 years the 74 is loose and 34 is erupting? = 6s lingual holding arch 2. the most important criterion in a partial pulpotomy? a) the root end remains open,e.g. not fully developed when treatment was started b) the root end if not completed would complete its fully development?

c) dentinary bridge became visible in x-ray d) pulp stones after some months 3. An 8 year old with carious exposure of 75 and anodontia of 35, what would you do? a) extract 75 allowing 36 to move anteriorly b) endodontic in 75 and indefinite retention* c) extraction 75 and space maintainer replaced by bridge later d) extraction 75 and 65 allowing move 36 and 26 4. If the childs teeth do not form, this would mostly affect the growth? a) alveolar bone* b) whole face c) mandible maxilla 5. Effects of early tooth loss i. function and oral health ii. over eruption iii. effect on mandibular posture iv. physiological effects on child and parent v. on permanent teeth, Shape (collapse of lower anteriors because of lost of lower canines) and Size( loss of arch space) 6. After the age of 6, site of increase in the mandible growth distal to first molar 7. Movement done during extraction of Es buccal and lingual not rotation because of developing tooth bud 8. Calcified teeth at birth a) all deciduous and permanent incisors b) all primary teeth only c) all primary teeth and first permanent molars* d) all deciduous teeth and all permanent molars 9. Sedation of children: a) diazepam 10. Why is an ankylosed 2nd primary molar not a good space maintainer? a) the first permanent molar may drift mesially over it b) it does not keep up with the rest of the occlusion*

11. The most common cause of palatally locked maxillary left permanent central incisor is a child with otherwise normal occlusion is a) premature extraction of maxillary left deciduous central incisor

b) prolonged retention of maxillary left deciduous central incisor* c) premature extraction of the left deciduous lateral incisor 12. Primary teeth compared with permanent a) more prominent lower? b) smaller pulp chamber c) narrowest prox contact area d) thinner enamel* e) thinner dentin* f) shorter crown* g) narrower occlusal table* h) contact areas broad and flat* i) same mineral content j) lighter in color k) pulp chamber and horns bigger than permanent 13. Which primary molar crown most resembles the crown of a permanent premolar? a) maxillary first* b) maxillary second c) mandibular first d) mandibular second note the primary second molars resemble first molars; the mandibular first molars doesn not resemble any premolar or permanent tooth 14. How do pulp chambers in primary teeth compared proportionally with those in permanent teeth? a) smaller in primary teeth b) larger in primary teeth* c) similar in both dentitions d) variable in permanent teeth note: although the primary teeth are generally smaller than their succcessors, their pulp chambera are proportionally larger 15. Child lived in a fluoridated area with 4 ppm till 8 years then moved in nonfluoridated area what teeth can show fluorosis? a) all teeth b) Central incisors c) All teeth except third molars* 16. Enamel formation occurs between: a) 2 months in utero and 7 years b) 2 moths in utero and 9 years c) 4 months in utero and 9-10 years d) 4 months in utero and 12-16 years* note: enamel formation begins at 4 months in the maxillary incisors and ends between 12-16 years in the third molars.

17. Treatment of choice for one permanent incisor of 9 years old child with pulp exposure due to fracture a) pulp cup b) pupotomy* c) pulpectomy 18. Ten years old with non-vital central incisor a) root canal therapy can lead to apical closure b) Pulpal testing periodically for possible pulpal regeneration c) Root filling with caoh for periapical closure* (apexification) 19. The anatomy of the second primary molar closely resembles that of the a) first permanent molar* b) second premolar c) first primary molar 20. When you use formaldehyde materials in a vital pulp it cause: a) fixation to pulp tissue* b) secondary dentine formation c) mumification 21. How does the denture for cleft retains? mechanical retention 22. extraction of primary molars - cut the cusp and root of the tooth as not to disturbed the developing dentition 23. How should a primary molar with relatively unresorbed roots encompassing the permanent tooth bud be extracted to avoid the inadvertent removal of a developing bicuspid? a) roll a mandibular tooth to the buccal and a maxillary tooth to the palatal b) raise a bussal flap c) remove the crown portion only, allowing the roots to resorb d) section the tooth vertically and remove each root separately* note: The furcation of a primary tooth is located much closer to the crown than in a permanent tooth; therefore, sectioning is easily accomplished with a highspeed tapered fissure bu. The tooth is then removed in sections without disturbing the underlying developing permanent tooth. 24. A six year old child presents with a carious maxillary second primary molar with a necrotic pulp. What treatment should be performed? a) Pulpectomy necrotic pulp *

b) Extraction c) Indirect pulp capping d) Antibiotic coverage 25. Failure of CaOH pulpotomy in primary teeth is internal resorption 26. Most cause of failure in case of replantation external resoprtion 27. Why do you use dental floss in with rubber dam - to guide the rubber dam through the contact areas 28. Retention can be obtained in the proximo-occlucal filling of a deciduous tooth through:occlusal dovetail and lock 29. Most common consequence arsing from premature extraction of deciduous molars is: a) loss of arch length* b) loss of speech sound c) loss of facial contour d) loss of tooth width* 30. Delayed shedding of primary teeth resulting in a permanent incisor erupting lingually to the primary one. 31. After 6 years of age, the greatest increase in the size of the mandible occurs: a) at the symphysis b) between the canines c) distal to first molar* 32. Ankylosis of mandibular primary 2nd molar is not always good space maintainer because of mesial inclination of the 1st permanent molar 33. In an upper deciduous molar has a caries exposure and after x-ray, the corresponding 2nd permanent premolar is agenesis (absent). What treatment would you do to the deciduous tooth? Endodontic treatment (conservation), indefinite retention (depends on age) 34. Which is the most likely place of bone resorption after a deciduous molar has a pulpal gangrene: interradicular septum 35. Cement indicated for root canal filling in deciduous molars zinc oxide eugenol without a catalyst (resorptive cement) 36. When opening the pulp chamber from the occlusal surface of endo a maxillary, E. How many pulpal horns could be exposed? Pulp horn primary usually equavalent to no of cusp.

i. ii. iii. iv.

Maxi 2nd primary molar 4-5 Mandi 2nd primary molar 5 Maxi 1st primary molar 3 Mandi 1st primary molar - 4

37. Roots = Maxi 3, Mandi 3 38. How many pulp horn are present in a typical mandibular deciduous second molar a) 2 b) 3 c) 4 d) 5* 39. Dental nerve block for children is: slightly lower than that in adults. The childs ramus is shorter than an adults. this is compensated for by inserting the needle a few millimeters closer to the mandibular occlusal plabe than in adult. 40. A child, when you check his erupted teeth, you found all incisor, first permanent molars, and the first premolars, How old is this patient? a) 13 b) 11* c) 7 41. What level of caries reduction is assocated with optimal fluoride of community water supplies? a) 15% b) 30% c) 55%* d) 80% 42. What is the range of concentration of fluoride in drinking water for optimal dental therapeutic effects with no significant dental fluorosis? a) 0.1 0.6 ppm b) 0.7 1.2 ppm * c) 1.2 2.0 ppm d) 2.0 3.5 ppm note: There is no single optimal level of fluride in the drinking water. The concentration depends upon the annual average daily temperature in the community, which influences the amount of water that local residents will consume. When the annual average maximum daily temperature is between 79.3 to 90.5 oF (26.3 to 32.5oC) as it is for some of our southern states, the recommended level of fluoride is 0.7 ppm. When this temperature is 50.0 to 53.7oF (10 to 12.1 oC) the recommendation is 1.2 ppm fuoride.

43. A child 5 (3-12) year old in an unfluoridated area, what is the daily dose of fluoride? a) 1 ug* b) .5 ug c) .25 ug 44. What is the recommended daily dose of fluoride for 2-3 years old child? a) mg b) mg* c) 1 mg d) 2 mg 45. What is the recoomended daily dose of fluoride for a child from birth to 24 months? a) mg* b) mg c) 1 mg d) 1 mg 46. A child has a rampant caries, you found that he is suffering from emphysema and is taking expectorant 3 times daily, which has a lot of sugar contents. What do you do to this patient? a) change the sugar type content to sorbitol* (alcohol based sugar) b) report that the patient is having the expectorant c) give him the syrup while sleeping d) give him inverted sugar 47. Loss of a tooth in mixed dentition phase, affects: a) same quadrant only* b) the relevant jaw only c) whole mouth* d) the relevant quadrant note- loss of arch length 48. Tooth brush for a child: i. head smaller ii. 180-400 um iii. nylon iv. soft 49. Endodontic treatment of primary molars are indicated when: a) removal of caries has exposed the pulp b) when carious lesion has just penetrated DEJ c) carious lesion is suspected to produce exposure* 50. Where can you find the proximal caries in primary teeth a) sightly gingival to the contact area*

51. Which permanent teeth has the highest caries prevalence in children: a) first molar* b) incisors 52. Indirect pulp capping procedures on deciduous molars are indicated when: a) removal of decay has exposed the pulp b) carious lesion is suspected of producing exposure of pulp c) carious lesion has just penetrated dentino-enamel junction d) not indicated* 53. Direct pulp capping should also be limited to permanent teeth. better to do pulpotomy 54. Which of the following is the best space maintainer? a) A Nance holding arch b) A fixed lingual arch c) A band and loop appliance d) A removable acryic appliance e) Pulpectomized primary molars* 55. An 8 year old patient with all primary molars still present exhibits a cusp to cusp relationship, relationship of permanent maxillary and madibular first molars. The dentist should a) continue regular recalls* b) plan serial extraction for more nomal adjustment of the occlusion c) refer the patient to an orthodontist for consultation d) place a cervical headgear to reposition maxillary molars e) disk the distal surface of primary mandibular 2nd molars to allow normal adjustment of permanent molars 56. Retention can be obtained in the proximo-occlusal of deciduous teeth a) occlusal lock* b) buccal and lingual divergence not beyond self cleansing areas c) line c.s. d) sharp undercut 57. What variation is necessary to do in class II in deciduous? a) deeper in occlusal b) isthmus is wider* c) proximal box narrower d) proximal box shallower

Orthodontics

1. Frankfort plane PO a) nasionand sella b) porion and sella c) porion and nasiion d) porion and orbitale* e) basion and orbitale note: porion is the highest point on the margina of the external auditory meatus, while orbitale the lowest point in the infraorbital margin 2. Canine retractor is used if the maxillary canine erupted a) palatally b) labially* 3. Leeway space: the difference between the primary first and second and width of the 1st and 2nd premolars (0.9 maxi, and 1.7 mandi) 4. Incompetent lip the 2 lips at rest position, not in contact 5. Prominent feature of Class II div II a) ant proinclination b) lateral incisor overlapping the anterior* c) deep overbite d) greater overyjet 6. Simple orthodontic patient, which of the following is necessary? a) History* b) Models* c) Articulators* d) x-rays* 7. Orthodontic planes and axial inclination of teeth is oriented a) Frankfort plane* b) Horizontal plane c) Occlusal plane d) Bolten plane sella nasion Note: this plane is commonly used for orientation of the head in clinical and radiographic assessment.

8. Major etiologic factor responsible for class II is a) thumb sucking

b) growth discrepancy* c) tongue habit d) tooth to jaw size discrepancy 9. Mixed Dentition Analysis - measure the space available for permanent canine and premolars 10. Primate Space > mesial to upper canine and distal to lower canine 11. Maxillary growth at age 6-7 ( growth of the maxilla and mandible forward and downward) 12. Cartilage growth nasal septum, head of the condyle, spheno-occipital synchhondrosis 13. Thumb sucking: Boucher 461 a) stop spontaneously* b) protrudes the mandible c) constrict the vault of the hard alate d) growth discrepancy 14. A four year old child presents with normal occlusion except for a one-millimeter anterior open bite. The patient is a thumb sucker who demonstrates immature adult speech development. A proper clinical course of action would be include: a) immediate placement of a fixed habit-reminding appliance b) Placement of a removable habit reminder after coundelling the patient and parents about the oral habit so that removal of the appliane by the child would be limited c) Referral of the patient to a speech therapist for myofunctional and speech therapy d) No tongue therapy or sppliance use at this time: periodic observation of the patient until the maxillary and mandibular permanent teeth rupts and mandibular permanent incisors erupt, with evaluation at that time.* Note: the proper stage for instituting a habit therapy is in the early stages of the transitional dentition age 8. 15. How would you go about to correct a single retroclined upper lateral incisor in a class I malocclusion with sufficient arch length a) extraction and fixed appliance b) removable (hawley appliance) c) Anteriior inclined plane* 16. Anterior bite plane, it should be: a) 5 mm out of occlusion in post b) less than 5mm out of occlusion from post* (4mm- school)

c) more than 5 mm 17. In small children what is the most common cause for I and D a) acute periodontal abscess* b) chronic periodontal abscess c) apical periodontal abscess 18. Commonest disase in children a) acute periodontitis b) chronic periodontitis c) furcal involvement d) chronic abscess* 19. What would have to do first in the correction of anterior cross bite? a) inclined plane b) Posterior capping c) Increasng vertical dimension* 20. The most dominant single emotional factor encountered in child management is fear of a) pain b) unknown* c) dentist d) white coat e) instrument and equiptment 21. Which is present in Angles class I division 2 malcocclusion: a) open bite b) retrusion of maxillary central incisors* 22. Which etiologic factor is responsible for Class II division 2 malocclusion Angles Classification is: a) thumb sucking b) growth discrepancy c) tongue thrusting habit d) tooth to jaw size discrepancy e) skeletal cause* 23. After 6 years of age, the place that grows most in the dental arch is: a) condyle b) from canine to canine c) behind the first permanent molars* d) base border of the mandibular body e) the angle of the mandible

24. Lateral in cross bite rest all normal occlusion and there is enough space use anterior guiding plane 25. Which of these case you expect to find severe class II case: a) ANB angle of + 8 degrees* b) ANB angle of - 8 degrees c) ANB angle of +2 degrees 26. SNA angle on cephalogram best signifies the (B 441) a) to determine the relation of the maxilla to the cranial base* b) to determine the relationship of the mandible to the cranial base c) relationship of the maxilla to mandible d) relationship of the mandible and porion 27. Most important fact in removal of impacted teeth a) adequate exposure by removal of bone b) preparative assessment* c) design of flap d) use of general naesthesia 28. A patient received a heavy blow to the right body of the mandible sustaining a fracture there. Where should you suspect a second fracture most likely be present? a) The symphysis region b) The left subcondylar region* c) The right subcondylar region 29. *Deviation to the left side in opening. The fracture of? a) left subcondylar region b) right subcondylar region c) left body of the mandible* 30. Simple orthodontic patient which of the following is necessary? a) History b) Models* c) Articulation d) X-ray 31. The best definition of articulation is: a) relation during movement in cetric and eccentric relation b) relation when the face is not moving in retrusive and lateral contact c) relation during lateral movement from CR d) Position of maxillary planes of the teeth are in maximum contact* 32. Which can be see in an x-ray at birth? a) deciduous incisors 2/3 of crown b) deciduous canine 1/3 of cron

c) d) e) f)

occlusal surfaces of 1st molar individual cusps of 2nd molar lower first upper and first perment molars all deciduous teeth and the first permanent molars* Cariology

1. For fluoride application to be cost effective should be used in: a) caries prone individual b) all community c) after irradiation d) diabetic patients e) a and c * 2. Streptococcus mutans play an important role in caries progress, because it produces polysaccharides: a) glucans b) dextrans* 3. Streptococcus mutans utilizes which substrate to form dextran? a) glucose b) fructose c) sucrose* d) amylopectin note: Dextran is a glucose polymer formed from the glucose moiety of sucrose by oral microorganisms. 4. A soluble polysaccharide found in dental plaque and formed from the fructose moiety of sucrose is called a) dextran b) levan c) hyaluronic acid d) cellulose note: levan is one of the polysaccharides formed from sucrose by oral microorganisms. 5. All dental Plaque: a) Not necessarily produce acid b) Produce chelation c) Produce caries

6. Plaque is defined as? a) bacteria and polysaccharides

b) c) d) e)

food debris microorganisms + desquamated epithelial cells + erythrocytes + PMN oral bacteia in an organized arrangement* It is a soft film composed mainly of bacteria and cannot be rinsed off the teeth.*

7. Kind of polysaccharide in plaque a) Glucose b) Glucans c) Levans and dextrans* 8. What is the most important factor for caries incidence sucrose, carbohydrate 9. Where does the proximal caries begin: a) Between the contact and the ridges (occlusal0 b) Apically to the free gingival c) Apically to the contact and coronally to the free gingival* 10. In root cavities (abrasion from toothbrush) the patient feels: a) spontaneaos pain b) long periods of pain then remission c) hypersensitivity to sweets and acidic food* d) pain during brushing teeth* 11. Dental service in the community the first priority is a) restoration of carious teeth b) treat periodontal disease c) relief of pain d) primary preventive measure* 12. To prevent cariogenic factor of medicine and syrups a) inert sugar is acceptable substitue for sucrose in the preparation b) sucrose should be replaced by sugar alcohol (sorbitol)* c) the medicament should be used only in the bed d) dentist should monitor and keep record of the patients medicines 13. Advantage of using of dental floss over rubber point interdentally: a) removes plaque and debris in the interproximal surfaces* b) polish c) massage of the interdental papillae d) aid to recognize calculus subgingivally 14. Change the acidogenic flora to non-acidogenic flora by means of changing the diet. How long it takes to achieve this change: a) few weeks

b) several months or longer 15. Cool climate fluoridated community at 0.75 ppm what ppm will result to fluorosis? a) 0 1 PPM optimum even up to 1.2ppm (cold climate- less drink water) b) 1.4 PPM c) 2.5 PPM* d) 0.7 PPM note; hot climate less than .7 ppm, cool climate 1ppm? 16. A Child who is 2-3 years of age with fluoridated water, the fluoride supplement will be a) 0 ppm b) .25 ppm c) .50 ppm* d) 1 ppm 17. The common denominator for all caries forming theories is presence of microorganisms 18. How can you detect the most accurately a carious lesion a) Explorer* b) bite wing x ray -proximal c) visual 19. Mutans streptococci and lactobacillus can produce great amounts of acids (acidogenic). Vigorously stimulated by sucrose a appear to be the primary organisms associated with caries in man. Recent evidence suggests that mutans streptococci onset of caries (enamel). While lactobacilli in active progression of cavitated lesions(dentin) 20. After prophylaxis, acidogenic oral flora turns into non-acidogenic flora in a) several hours* b) 1 day c) a few days d) a week 21. Plaque is more accurately described as bacterial plaque because it is composed almost completely of bacteria and their by-products. The accumulation of plaque on teeth is highly organized and ordered sequence of events. Adherent bacteria have special receptors for adhesion to the tooth surface and also producea sticky matrix that allows them to adhere to each other. After the first 24 hours, changes in the proportion of the microorganism take place: streptococci decrease to about 45% and gram increases to 20%. After 3 days gram negative cocci and rods continue to increase. At 7days,complex plaque flora consists of; spirochetes, fisiform bacilli, filamentous organisms, gram cocci and gram + bacilli.

22. The hardest dentin: a) affected<infected<sound b) infected<affected<sound* c) sound> infected> affected d) affected>sound>infected 23. Enamel caries (zones) a) Zone 1 translucent zone b) Zone 2 - Dark zone c) Zone 3 Body of the lesion d) Zone 4 the surface zone 24. Dentinal caries (zone) a) zone 1 normal dentin b) zone 2 sub-transparent dentin (demineralization) c) zone 3 transparent dentin (intact collagen) d) zone 4- turbid dentin cannot self repair with bacteia e) zone 5 infected dentin with bacteria and demineralization note: affected dentin is softened demineralized dentin that is not yet invaded by bacteria (2 and 3) infected dentin , sofetened and contaminated by bacteria (4 and 5) 25. Patient having caries in all his teeth check saliva ( test saliva) 26. A patient comes with lactobacillus of more than 100,000. What will you do? Reduce sugar in diet 27. Px with multiple class 2 and class v check diet 28. What is the important role of saliva in dental caries: buffering action 29. Why does pulpal inflammation take place before bacterial infiltration : toxins reaches first ( acid ) 30. When there is an injury, what kind and where will be the dentin be formed: i. reparative dentin* ii. sclerotic dentin* - lumen of dentin becomes smaller iii. irregular* iv. dentin forms at localized areas around pulpal surface reactionary dentin* 31. The most effective in making teeth more resistant to caries: a) general nutrition b) systemic fluoride during mineralization* c) topical fluoride d) intake of calcium 32. Caries activity is directly proportional to each of the following except? a) oral retention of fermentable carbohydrate eaten

b) frequency of eating fermentable carbohydrate c) total daily intake of fermentable carbohydrate d) physical form of food items eaten * 33. A non-caloric sweetner (low cariogenic sucrose-substitutes) a) Manitol (sugar alcohol)* b) saccharin c) xylitol (sugar alcohol)* 34. Water fluoridation, reduces caries by 50-60% 35. For fluoride to be cost effective it should be used in a) caries prone individual b) all community c) after irradiation d) diabeteic patients e) a and c* 36. Cause of rampant caries in children:high carbohydrate intake* 37. In which tissues you find the highest fluoride: a) Bone* b) saliva c) liver d) blood e) brain note: the fluoride content of bone in a community with fluoride deficient water supply was fould to be approximately 500 ppm. Fluoride does not concentrate in the soft tissues of the body. About 1 ppm is found in most tissues on a fresh weight basis. The level of fluoride in the blood and saliva is approximately 0.1 ppm 38. Fluoride in plaque 39. Fluoride in 2.2 mg NaF tablet or one teaspoonful 0.5 % NaF solution a) 2 mg b) 1.5 mg c) 1 mg* d) 0.25 mg this can also be received 1 mg fluoride by consuming 1 liter of water with 1ppm fluoride in water. 40. Fluoride in high concentration is bacteriocidal but in low concentration is bacteriostatic

41. Fluoride in 3 years old, fluoridated area 0.5 ppm give 0.25 mg tabs 42. The oral lethal dose of fluoride for humans is estimated at: a) 0.1 g - 0.2 g b) 0.2 g - 0.4 g c) 0.4 g 1.0 g d) 2.0 g 3.0 g* 43. What is the largest amount of sodium fluoride tat should be prescribed at any one tme to a given individual as recommended by the American dental association? a) 20 mg b) 260 mg* c) 720 mg d) 990 mg 44. A fluoride at a concentration of 1 ppm contans 1.0 mg fluoride per: a) 0.1 ml b) 1ml c) 100 ml d) 1000 ml* e) 1,000,000 ml note: at a concentration of 1 ppm, which is the optimal concentration for preventing tooth decay without producing fluorosis, 1000 ml or slightly more than a quart, would contain 1 mg fluoride 45. Fluorides are inhibited by milk and other dairy products. It reacts with the fluoride ions and forms insoluble salts that cannot be absorbed. 46. Which fluoride not used in fluoridation of water a) NaF* b) Na2S1F6 c) H2S1F6 d) SnF2 47. Water fluoridation, this material cant be used ( bitter and not stable in solution causing staining at margins or restoration) a) Sn F2 b) APF c) CaF d) NaF? 48. Several applications have been suggested to increase fixation of prophylactic application of topical fluoride which includes All Excapt: g) increase F+ in solution (concentration) h) increase PH of F+ * i) increase exposure time to topical F+(time)

j) pretreat enamel with 0.5% phosphoric acid k) use NHH4F+ instead of Na F+ 49. Optimally fluoridated water in temperate climate contains a) 0.8 ppm of F-* b) 1 ppm of Fc) 1.2 ppm of F50. The amount of F+ for caries reduction according to patients age and level of F+ in drinking H2O. Which is incorrect? a) 1 year old child requires no F+ when drinking water is 0.3 ppm b) 3 year old child requires NaF+ when drining water is 0.7ppm c) 6 year old child requires 1 mg of F+ when drinking water contains .5 ppm* note: 6m 2y (<.3mm) 0.25 ppm(..3-.7um) none 2-6y (<.3um) 0.5 ppm(.3-.7um) 0.25ppm >6y (<.3um) 1 ppm (.3-.7 um) 0.5ppm 51. To prevent caries one should advice a) fluoride in dentrifices b) honey should be substituted for sugar in the diet* c) raw and white sugar leave the same cariogenic power d) a daily calcium supplement is an effective preventive measure 52. Mature dental plaque can be described as containing anaerobic, mostly gram and filamentous bacteria 53. Most anticaries effect in saiva a) PH b) Amylase activity c) Buffering capacity* d) Sulfactare activity 54. The most effective in to make teeth resistant to caries? a) general nutrition b) fluoride during mineralization* c) topical fluoride d) calcium

55. In which of the following body tissues would the highest fluoride concentration be expected? a) brain b) saliva c) liver

d) bone (0.3 ppm)* e) blood 56. The 2.2 mg NaF will give rise to how many m. grams of F a) 1 mg b) 0.1 mg* 57. The most effective immediate action for children who has accidentally swallowed 10 cc of 10% fluoride solution is to: have the child drink milk or some other calcium containing liquid 58. Aciduric organisms (strep mutans) they produce the extracellualr polysaccharide glucans and fructans, from metabolism of sucrose, as well as form matrix of plaque 59. The effect of systemic fluoride in adults no effect because they are already calcified 60. Transillumination method is used to diagnose (detect) caries. 61. Which have the highest sucrose content: a) ice cream b) cough syrups (highest)* c) canned juice d) breakfast cereal e) sweet potato (lowest) 62. Bacteria synthetize plaque from sucrose (most cariogenic sugar) 63. double blind technique study of agents effects in which both the recepient and administrator knows whether the active or inert is given 64. Primary prevention of pit and fissure cavity is PFS 65. Pit and fissure cavity starts on the wall ( on enamel on each side of the fissure) 66. The effective method to prevent dental decay Water fluoridization 67. Teeth decrease enamel solubility with water fluoridation as fluoride increases 68. Permanent six most susceptible tooth to have cavities 69. Tooth erosion- generalized tooth loss 70. tooth loss on upper anterior teeth bulimia 71. Furcation involvement check radioucency

72. pulpal hyperemia is best treated by e) partial pulpectomy f) root canal therapy g) application of zinc oxide eugenol dressing because of low irritation to pulp* and sedative effect 73. Transillumination as an aid can be used for each of the following condition? a) dental caries* b) calculus c) acute maxillary sinusitis d) pulp stnes e) hemorrhagic pulp 74. What factors are important in determining the cariogenicity of a patients diet: a) type of carbohydrate consumed b) physical form of the food c) frequency of consumption d) all of the above are correct* 75. The form and frequency of carbohydrate intake that is most cariogenic is:frequent intakes of retentive sweets throughout the day 76. High sucrose and low sucrose in foods a) ice cream* b) cough syrup c) Cordially* d) sweet potato 77. The purpose of the snyder test is to?predict the nature of the combined acidogenic organisms in the oral cavity 78. Using the fluoride in the root surface caries to protect a) enamel b) dentin and cementum* c) cementum 79. Dental plaque produces/ a) chelation b) dental caries c) acids* 80. Electric pulp test does not detect? a) capped teeth* b) not in all instances c) necrotic pulp d) pulpitis

81. The tooth is vital with some degree of inflammation and patient is in transit pain, treatment is? a) Pulp extirpation and Ca(OH)2 dressing b) Pulp Capping* c) Pulp extirpation and ledermix dressing d) Remove filling and replace with ZOE? e) Extraction 82. Senile caries root caries 83. Patient with multiple root surface caries should be advised; low abrasive toothpaste 84. the caries preventive effect of fluoride depends on-their incorporation uniformly throughout the enamel and making it caries resistant 85. Which of the following would contraindicate dental sealant application : evidence of frank caries 86. cariogenic bacterias a) streptococcus mutans very acidogenic b) lactobacillus acidophilus-very acidogenic c) lactobacillus casei-less acidogenic d) streptococcuss sanguis- less acidogenic e) streptococcus salivarius-less acidogenic f) streptococcus mitior- less acidogenic g) streptococcus milleri- less acidogenic h) streptococcus faecallis- less acidogenic i) streptococcus viscosus

Radiology 1. Long cone should be used for paralleling technique for the following reasons: a) to minimize the exposure dose to the patient b) to get proper angulation* 2. Lamina dura in x-ray a) radiolucency between b) the cribriform plate of bone making up the tooth socket* (supporting alveolar bone) c) dense part of bone consistent with healthy periodontal status d) the pattern of radiopaque lines in supporting alveolar bone

3. RAD is defined as: measurement of the amount/energy of x-ray absorbed on mass surface 4. Exposure of the film to x-ray, the silver bromide emulsifies, rearranged. This causes: a) radiopacity b) radioucency c) latent image (black metallic silver)* 5. The extra-oral radiograph, best demonstrates a) subcondylar portion of the mandible is: b) Townes view (reverse townes)* c) Submental vertex 6. The extra-oral radiograph that best shows maxillary sinuses ( also midfacial fracture) is: a) waters view* b) AP Skull 7. How can you make sure a tooth is ankylosed, radiographically? a) thick lamina dura b) no periodontal ligament space* 8. Radiolucent area close to apexof incisor (central) which moved in the second x-ray is likely a: a) cyst b) incisive foramen* c) granuloma d) abscess e) artifact 9. Which of the following doesnt reduce radiation (answer more than one) a) decreasing of kilo voltage* b) type of film c) collimation d) used of open end lead lined cone

10. The most important factore to protect the patient from radiation:fastest film* 11. Radiographs of proximal caries shows? a) clinical caries is bigger than radiographic features* b) clinical caries is smaller than radiographic features 12. The earliest apical radiographic change seen in a pulply involved tooth is

a) b) c) d)

apical resorption loss of lamina dura widening of periodontal ligament space* hypercementosis

13. As a diagnostic bitewing should be taken a) every year if the parent consent b) when requested by parent c) as part of regular examination * d) every two years 14. Bitewing x-ray is useful for a) proximal caries* b) occlusal cares 15. A caries in an x-ray a) is bigger than the same one in the teeth b) Is smaller than the same one in the teeth* c) Is the same 16. If the developing solution is too old, the resulting film will be: a) too light* b) blurred 17. Dentist is allowed to hold the film in the patients mouth when: a) the patient is unable to hold the film because of physical problem b) dentist should never hold the film* c) time consuming for the dentist to hold the film d) all of the above 18. KVP- with time 19. time related to intensity (inverse square law) 20. Speed of film most effective way to reduce rediation 21. RAD- radiation absorbed dose 22. metal layer in the film packet- to reduce radiation to patient 23. Which of the following does not reduce exposure of patient to x-ray radiation a) fast film b) filtration c) collimation of the beam d) open end, eald lined cone e) reduction of KVP*

24. On examination of an x-ray of upper 6, it is noted that buccal roots have been elongated considerably. This result of a) mesially directed horizontal angulation b) distally directed horizontal angulation c) too little verical angulation* d) too great vertical angulation e) excessive object-film distance 25. Impacted canine on the palatal side, the x-ray cone is moved distally for the second film in relation to the root of the first premolar, the image: a) moves diatlly if canine is on the palatal side b) moves mesially if canine is on the palatal side* c) moves distally if the canine is on the buccal side 26. In an x-ray the lamina dura appears as thin white line* 27. The earliest apical radiographic changes seen in pulpally involved tooth is: a) resorptioin of bone b) loss of lamina dura c) widening of periodontal ligament space* 28. Radiographic absence of lamina dura is a feature of all the following except a) Pagets diseas b) Hyperparathyroidism c) Fibrous dysplasia d) Osteogenesis imperfecta* 29. Late Pagets Disease x-ray a) orange peel b) cotton wool* c) mosaic histo d) copper wire 30. The x-ray tube to film distance is increased fom 8 to 16 inches. What fraction of the original radiation intensity at 8 inches in the film now exposed to at 16 inches? a) 20% b) 25% c) 33% d) 50%* 31. The early paget disease radiograph shows? a) radioluscent area* b) cotton wool appearance c) radiopaque area

32. Which x-ray do you use for assessing periodontal condition? a) periapical b) bite-wing c) occlusal d) panoramic* e) intraoral source radiation 33. The correct definition of RAD is a) amount of x-ray radiation the patient absorbs into the body tissue Radiation Absorbed Dose b) absorbed dose, is a measure of energy 34. REM roentgen equivalent man: The amount of any ionizing radiation that has the same biologic effect in man as one R of x-radiation 35. Incisive foramen is super imposed over apex of root on radiograph it may be mistaken for? cyst* 36. Blow in the mandible, signes are: deviation to the left in opening and in x-ray appears a unilateral fracture, where is the fracture? a) neck of the condyle (right) b) body of the mandible(left) c) neck of the condyle (left)* d) body of the mandible (right) 37. The root image is too long angulation is too short 38. In which disease you can see radiiopacity? a) pagets disease cotton wool appearance* b) central cells c) cyst 39. Bitewing film is used to detect: a) proximal caries* b) periapical lesions c) alveolar bone loss 40. A round radiolucent area near the apex of the maxillary central incisors mmoves in the second radiographs a) incisive foramen? b) Periapical abscess c) granuloma 41. What are ways to decrease the amount of radiation to the patient a) filtration of x-ray* b) increase the KVP*

c) inclination of the x-ray* (parallel less) 42. According to inverse square la, if you change the focal spot-film distance from 200 mm to 400 mm. When exposure time was 0.25 second, then the time should be change to a) 0.5 sec b) 1 sec* 43. If you have to chose an x-ray machine with one switch to convert 60 kjw to 70 kjw. What changes in time will be: no change 44. Which is the best view to disclose any pathologies in the maxillary sinus? a) (waters view) occipitomental view* b) PA view 45. When the temperature of the processing solution too warm and the time is normal room temperature the processed film will be ; dark film

CASES: 1. What are the provisions of dental treatment during pregnancy? > routine dental treatment of pregnant women under local anesthesia is safe but general anaesthesia, some drugs and possibly radiography may endanger either fetus or mother >Oral Complications 1) aggravation of preexisting gingivitis (pregnancy gingivitis), which begins in about 2 months and persist until parturition. Improve oral hygien and scaling reduce gingivitis and cboth conditions may resolve after parturition. 2) The teeth do not of course lose calcium as a result of fetal demands and there is no reason to expect caries to become more active unless the mother develops a capricious fro sweets 3) In few women subject to recurrent aphthae, ulcers may stop during pregnancy or even worsen 4) Radiography should be avoided, especially in the first trimester where the the fetus is most vulnerable to developmenatal defects, although dental x-ray is a very minimal or insignificant risk. Nevertheless, if needed patient must wear a lead apron and exposure must be minimal. Drug Treatment: This should be avoided where possible, especially in the first trimester. Any drug may endanger the fetus but general anesthesia and possibly sedation with diazepam or midazolam are particular

hazards and must be avoided in the first trimester and in the last month of pregnancy. Tetracycline may cause tooth discoloration but other drugs may be terratogenic. Unfortunately many pregnant patients are unaware of their pregnancy in the early part of the trimester and therefore it is wise to avoid giving drugs to women of child bearing age, unless absolutely essential. Dental treatment: best carried out in the second trimester, but same precautions apply. Advance restorative must be postponed until the periodontal state improves after partuiution and prolonged sessions of treatment are better tolerated. In the third trimester it would be hard to do elective procedure because of te risk of hypotention and premature labour. Better to avoid dental treatment at thelast month. Nitrous oxide and other anesthetics, mercury vapor and infections such as with viruses may pose occupational risk to pregnant dental staff. 2. As part of a medical history the patient told you that he had hepatitis 10 years ago. What other information will you require form the patient regarding his present condition and how these informations will affect your treatment plan? Details on the cause, onset, duration, symptoms type and treatment of the patients hepatitis condition 10 years ago. Present medical condition, last medical check-up Last dental treatment and complications if any Type of dental treatment needed Laboratory examination on Hepatitis, antigen and antibiody I. Hepatitis B A. Clinical Aspect the effects of hepatitis B infection range from subclnical infections without jjaundice in the vast majority of cases, to fulminating hepatitis, acute hepatitis failure and death. Most patients recover completely and suffer no untoward effect apart perhaphs from some persistent malaise. The prosromal period of 1-2 weeks is characterized by anorexia, malaise and nausea. Muscle pains, arthralgia and rashes are more common in hepatitis B than hepatitis A and there is often fever. As jaudice becomes clinically evident the stools become pale and the urine dark due to bilirubinuria. The liver is enlarged and tender and pruritis may be traoublesome. Serum enzyme estimations are useful. This can persist into a carrier state, can remain positive for 20 years. Most carriers are healthy but other, especially those with persistently abnormal liver function tests, develop chronic disease. B. Dental Management Sources: blood. Plasma, serum, saliva Inmunity: Hepatitis Surface antibody Universal precautions

Dental staff shuld consider vaccination against hepatitis B. With extremely high risk patients it may be advisable that only staff who are immune should carry out dental treatment. It is clear that not only is there risk from known positive patients: there are also dental patients who are totally unsuspected carriers, some other suspects are negative at the time of testing but may become positive before treatment. It is important to avoid penalizing HbsAg(+) patients by refusing them treatment since such actions may lead the patient to conceal the fact that he may be posistive or is at risk. Furthermore, since most positive patients are unidentified, refusal to treat known cariiers would not significantly reduce the risk to the operator. Emergency dental care necessary during incubation or acute hepatitis should be carried out in the hospital department with appropriate precautions against transmission of infection. Due regard must be taken for the fact that the liver damage may influence dental treatment. II. Hepatitis A this is communicable for only 2-3 weeks for the latter half of the incubation period until a few days after the onset of jaundice. The infection has not been reported to be transmitted from an infected oral surgeon and patient. III. NonA and NonB most common type of post-transfusion hepatitis in some areas and is responsible for much sporadic viral hepatitis, particularly intravenous drug abusers Precautions in patients with viral hepatitis infection in dental treatment: 1. The patient should be treated at the end of session 2. All dental staff should wear gloves, goggles, mask, gowns. Staff with any exposed skin wounds must be covered. 3. All working surfaces must be covered 4. Wherever possible, disposable instruments should be used. 5. To avoid any possible aerosol spread of HBV, HIV, other viruses and opportunistic organisms, ultrasonic scalers should not be used. Air-rotor should be used with a rubber dam. 6. Intraoral Radiographs can be taken provided each packet of film will be coverd by a sealable plastic envelope before use. The cone of the x-ray should be wrapped 7. A portable suction system should be used and a metal container used as a spittoon 8. Impression used must be silicone basd and disinfected before sending to the laboratory 9. Disposable instryments and sharps be placed in a nonpuncture container 10. Personels with hepa B shots 3. the patient came to your surgery asking you to replace his avulsed 11 with tooth screwed to the bone. He said he watched a documentary in the television about

osteoimplantation what informations will you give the patient regarding the request for treatment? Preoperative Medical Evaluation of the Implant Patient - The contraindication for implant is similar to any elective surgical procedure - Contraindications ti implant placement: Acute illness, Terminal illness, Pregnancy, Uncontrolled metabolic disease (Diabetes) Tumorocidal radiation to implany site, unrealistic expectationsImproper motivation, lack of operator experience (poor oral hygiene) and unable to prosthodontically restore. Evaluation of the implant site clinical and radiographic evaluation to evaluate is there is adequate bone and to evaluate the proximity of anatomic structures that may interfere with implant placement. The combined surgical and restorative plan along with feasible nonimplant alternatives are then presented to the patient so that he she can make an informed decision whether to proceed with treatment. - Clinical examination a) visual inspection and palpation will allow the detection of flabby excess tissue, narrow bony ridges and sharp underying ridges and undercuts that may not be adequate if the thick overlying soft tissue is dense, immobile, fibrous tissue. - Radiographic Evaluation a) initial film using panoramic radiograph b) bone width of maxi and mandi cephalometric x-ray c) posterior mandible and maxilla clinical examination d) CT-scan determine the locatin of the inferior alveolar canal and maxillary sinus, but high radiation so limit routine use - Anatomic Limitation to implant placement a) Buccal Plate = 0.5 mm b) Lingual plate = 1.0 mm c) Maxillary sinus = 1.0 mm d) Nasal cavity = 1.0 mm e) Incisive Canal = avoid midline maxilla f) Interimplant distance = 3.0 mm between outer edge of implant g) Inferior alveolar canal = 2.0 mm from superior aspect of bony foramen h) Inferior border = 1.0 mm i) Adjacent natural tooth = 0.5 mm - Quality of bone = The bone quality in the maxilla, particularly in the posterior maxilla, is poorer than the mandibular bone. There are larger marrow spacesand thinner, less dense cortical bone , which affects treatment planning, since increased time mmust be allowed for integration of implants. Generaly, a minimum of 6 months is necessary for adequate integration of implants placed in the maxilla

Minimum integration time : Anterior mandible ( 3 months) Posterior mandible ( 4 months Anterior maxilla (6 months) Posterior maxilla ( 6 months) Into bone graft (6-9 months)

Informmed Consent with both the surgeon and restorative dentist and patient prior to surgery and demonstrate proposed treatment. The patient should be informed regarding time of surgery, the need for 2 surgical procedure and the delivery of the finished prosthesis. The patient should also be informed concerning the need to leave existing dentures out and the length of time this would be necessary. The patient should be informedabout potential short and long term risks such as nerve injury, infection, and implant failre. Alternative treatment options, including conventional dentures or bridges, should be presented. Finally a clear understanding of the expected cost of the proposed treatment should be reached. After this information is discussed, the patient should then sign a written informed consent document. Surgical Treatment : This can be done in an ambulatory setting with local anesthesia. This type of surgery requires more time than other surgical procedures, so that the sue of conscious sedation is beneficial. Although implant placement is less traumatic than tooth extraction, the patient will have the expectations that it will be more so. Preoperative patient education and conscious sedation both help to lessen their anxiety. Preoperative antibiotic is usually recommended. An oral dose of prophylaxis is usually recommended 2 grams penicillin V 1 hour prior to treatment. No post operative antibiotic needed Profound anesthesia is required for precise implant placement Adequate aseptic technique to minimize risk of infection. The patient should rinse with 15 ml of 0.12% Chlorhexidine gluconate (Peridex) for 30 secs immediately prior to the start of the surgery, which is effective iin significantly reducing the oral microbial count and maintaining a reduced level for an hour or more. A perioral facial preparation using iodine or chlorhexidine based antiseptic solution is useful. The field is then isolated with sterile towels. The surgeon and assistants should follow sound sterile techniques using masks, sterle gloves and sterile nstrumentation. 1. soft tissue incision 2. preparation of implant site 3. implant placement

Post-operative care: a) radiographic assessment to evaluate the position of the implant inrelation to adjacent structures, such as sinus and inferior alveolar canal and relative to other implants. b) Analgesics ( mild to moderate strength) c) 0.12% chlorhexidine gluconate rinses for 2 weeks after surgery to keep bacterial population s at a minimum during healing. d) The patient is evaluated in a weekly basis until soft tissue wound healing is complete ( 2-3 weeks) e) After 1 week the patient can wear denture that is relined with soft liner Stage II Surgery: 1. implant uncovering : The goal is to accurately attach the abutment to the implant, preserve attached tissue and recontour and thin tissue as necessary. Advanced Surgical technique (Postextraction Placement of Implant) Time: The implant may be placed immediately ( at the time of extraction). Earlier (within a few weeks of extraction) or later ( more than 2 months following extraction) 1) Immediate over-all shortest possible healing time, if tooth removed not infected, and removal without bone loss, the implant will be placed 4 mm apical to the apex of the tooth. And countersunk by about 2 mm below the crestal bone to allow for resorption. 1mm gap implant and socket, no modifications needed if more than 1mm must use guided tooth eruption. Primary soft tissue closure must be achieved. Integration time plus additional 12 months. 2) 3-4 weeks after during this time the overlying soft tissue will heal and primary closure can be achived. This does not do anything to improve the quality of the bone but does help with soft tissue closure. There should be an increase in integration by 1 month. 3) If bone quality or quantity is compromised better to wait for a minimum of 2 months prior to implant placement. This is long enough to allow remodelingof the socket and for multirooted teeth some filling of the socket with bone. Placement same as the technique for rutine implant placement.

4. What are the disadvantages and advantages of moisture control used in restorative dentistry

Operative dentistry cannot be qxecuted properly unless te moisture in the mouth is controlled. Moisture control refers to excluding sulcular flid, saliva, and gingival bleeding from the operative field. It also refers to preventing handpiece spray and restorative debris from being swallowed or aspirated by the patient . The rubber dam, suction devices and absorbents are varyingly effective in misture control. These techniques and oters are discussed in detail later. 1) Rubber Dam is used to define the operating field by isolating one or more teeth from the oral environment. The dam eliminates saliva from the oral environmet. The dam eliminates saliva from the operating site and retracts the soft tissue. b) Advantages: - dry, clean operating field - access and visibility - improve properties of dental materials ( restorative materials will not achieve their maximal physical properties if used in a wet field - protection of patient an ddentist (aspiration) (infections) - Operating efficiency and increased productivity (less talk with patient, quadrant treatment can be done) c) Disadvantage time consuming - patient objection - unerupted tooth, third molar and malposed tooth 1) High Volume Evacuator/ Saliva Ejectors Advantages of hi-vac a) cutting both of tooth and restorative materials, as well as other debris are removed from the operating site b) a washed operating field improves access and visibility c) there is no dehydration of the oral tissues d) without an anesthetic the patient experiences less pain e) pauses that are sometimes annoying and time consuming are eliminated f) precious metals are mmore readily salvaged g) quadrant dentistry is facilitated 2) Absorbents and throat Shields = are helpful in short periods of isolation 3) Retraction Cord used for isolation and retraction in the direct procedures of treatment of cervical lesions and in facial veneering as well as indirect procedures involving gingival margins. 5. A 15 years old boy came to your surgery with deciduous canine still intact and no evidence of permanent upper canine. What measures will you use to see if the

permanent canine is erupting?And upon examining it. What you carry on as part of your treatment plan? 1) Examination clinical and radiographic, position, eruption, root development and presence of the tooth 2) Depends on root development extraction of primary to hasten eruption or surgical exposure plus orthodontic traction to pull the tooth down. 3) Osteoectomy and flap is needed 4) Vital pulp 2-3 mm dring 4-8 months in order to protect tooth vitality 5) Non-vital can be extruded more rapidly, 3-5 mm during 3-6 weeks 6) Retention period of not less than 6 months to prevent relapse 6. A deep carious lesion in an asymptomatic vital tooth is being excavated. The possibility is that continued excavation might result in an exposure, What will you do? 1) what tooth 2) isolate with rubber dam 3) caries removal, lateral walls before deeper area, just in case pulp exposure occurs ( Caoh, liner and temporary cement) observation before permanent filling 4) remove infected dentin but leave letehry dentina dn place Glass Ionomer and then observe indirect pulp capping 5) others 7. Bulimia j) depression, self-deprecating thoughts k) awareness that eating pattern is abnormal nutritional counceling l) gastrointestinal disturbance m) self-inducing vomiting cause swelling of salivary glands and esophagus n) oral manifestations : burning sensation of tongue, soar throat, angular stomatitis, enamel erosion, dentinal hypersensitivity(permylolysis), lingual surface of maxillary incisors most often affected, rampant caries from high consumption of sucrose, irritated soft tissue from acid in vomiting o) treatment: full coverage plastic splints, protect the teeth with magnesium hydroxide, topical fluoride, fluoride rinse, bicarbonate after vomiting incident 8. Effects of early tooth loss p) Function and oral health q) Supraeruption r) Effect on madibular posture s) Physiological effects on child and parent t) On permanent teeth (shape and size) 9. Gingival Hyperplasis u) occurs in 40-50% of children taking phenytoin for seizure control v) 30% of these lesions warrant excision

w) hyperplastic tissue is thought to be an exaggerated response to plaque and therefore can be partially controlled with institution of good oral hygiene practices 10. 3 stages of syphilis: i. primary (chancre): shallow, re, hard, ulcer, painless, demonstrating T ii. secondary : rash of red, flat or raised macules erupts over entire body demonstrates spirochetes through lymphocytes blood test serology negative iii. tertiary lesion appears betweeb 7 months to 5 years after initial contact 11. carcinoma of the tongue: i. 25-40% of oral carcinoma (excluding lip lesions) ii. uncommon in very young iii. typically asymptomatic, later stage develop pain or dysphagia iv. clinically: indurated, non-healing ulcer with elevated margin v. most common location is posterior lateral border: 45% of tongue uncommon to develop in the dorsum of the tongue 12. Topical Fluorides: vi. effective in reducing caries during adolescence vii. can be carried out professionally or by self- application viii. most common used agents: NaF, SnF, & APF. Solution or gel ix. effectiveness between 18% - 80% 13. Professionally administered topical F compared with self-application topical F; the self-applied F is: b) more frequent applications are possible c) more sustained and higher levels of fluoride in E have been reported d) more economical e) higher caries reduction have been reported over 70% f) excretion of fluoride via the main route: kidney - urine 14. Pain- dysfunction syndrome of TMJ, there is a spasm of: masticatory muscles leading to limited movement of the mandible if the spasm not relieved 15. Treatment of Candidiasis: x) majority of cases simply treated with topical applications of Nystatin suspension y) chronic mucocutaneous candidiasis or oral candidiasis associated with immunosuppression, topical agents may not be effective. Systemic administration of: amphotericin B maybe necessary 16. A vitamin which cannot be synthesized or stored in the body is: i. vit. A ii. vit. B iii. vit. C* iv. vit. E

17. Phenytoin-induced gingival hyperplasia z) treatment should emphasize elimination of local gingival irritants, scrupulous oral hygiene, and interdental massage aa) hyperplasia of any degree will not resolve simply by removing local gingival irritants, and excision of hyperplastic gingival, root planning and elimination of rough margins on restorations are usually necessary before adequate gingival hygiene and plaque control can be established bb) if gingivectomy is not followed by adequate home care and use of interdental massage, hyperplasia will recur 18. Oral Cancer, usually squamous cell carcinoma, como sites are lips, lateral border of the tongue and floor of the mouth. Cancer of the buccal mucosa and gingivae is less common and hard palate cancer is rare. Causes are obscure and the role of alcohol and cigarette smoking are unconfirmed. General management: any lesion of dubious nature should be biopsied. Patients with oral cancer are best managed by a team of specialists including the dental surgeon. Opinion varies as to the value of radiotherapy or surgery: many early leasions can be treated by etehr method. While advanced cancer is in general incurable by any technique. The type of dental care should to some extent be tailored to take account of the prognosis and must always be planned inr elation to the interest that the patient has in his oral state. Just because the patients are dying does not mean, however, that tey should be allowed to suffer from dental pain, or that tehri appearance be neglected. Indeed, the provision of dental attention, for example the contruction of a denture, may help the patients morale Oral complications of radiotherapy involving the oral cavity or salivary glands a) mucositis , initial reaction is mucosal erythema followed by sloughing and considerable discomfort. Dysphagia and oral soreness becomes maximal 24 weeks after radiotherapy but usually subsides in a further 2-3 weeks. b) Loss of taste (hypoguesia) damage to the taste buds but xerostomia alone can disturbe taste sensation. Taste may start to recover within 2-4 months but, if more than 6000 Cgy have been given, loss of taste is then usually permanent. c) Xerostomia and Infection irridation depresses salivary secretion and the saliva has a higher viscosity bt lower ph. Will return after several months. Xerostomia predisposes to inflammatory periodontal disease, caries, oral candidosis and acute ascending sialadenitis. d) Radiation caries and hypersentsitivity softer and more cariogenic diet because of dryness of mouth and loss of taste.irridation causes hypersensitivity making oral hygiene measures more difficult. e) Osteoradioecrosis and osteomyelitis death of bone of jaw, extraction after radiotherapy can lead to osteomylitis f) Trismus progressive endarteritis of affected tissues, with reduction in their blood supply, follows radiotherapy. The result may be replacemnt fibrosis of the masticatory muscles. Fibrosis becomes apparaent 3-6 months after radiotherapy and can cause permanent limitation of opening

g) Dental defects irradiation of developing teeth can cause hypoplasia and retarded eruption. Dental treatment : complications of dental therapy after radiotherapy are such that planned treatment should be carroed out before irradiation. Oral hygiene should be meticulous, preventive dental care instituted and restorative procedures carried meticulous, preventive dental care instituted and restorative procedures carried out at this stage. The time interval permitted between extractions and radiotherapy is invariably a compromised because of the need to start treatment as soon as possible. No bone should be left exposed in the mouth when radiotherapy begins since, once the blood supply is damaged by radiotherapy, wound healing is jeopardized. Any advise as interval of at least 2 weeks between extracting the teeth and strating radiotherapy but this is not always essential. During therapy Mucositi may be relieved by using warm nomalr saline mouthwashes and lignocaine viscous 2 percent. Smoking and alcohol should be discouraged. A 0.2 % chlorhexidine mouth wash to be used tomaintain oral hygiene. Saliva substitue may provide symptomatic reliefe. Trismus may be reduced by jaw-opening exercises with tongue spatulas or wedges used 3 times a day. Antfubgal drug such as nystatin suspension as mouthwash can be sued four times daily as may be required. After therapy Oral hygiene and preventive dental care should be continued and mucosistis managed as outlined on top. Dentale xtarctions after therapy can cause osteomylitis but if unavaoidable, trauma should be kept to a minimum, raising the periosteum as litte as possible and ensuring the sharp bones are removed. Careful suturing is needed and continued for 4 weeks at least. if in the line fo radiation the extractin is sometimes posposed and cntroled by antimicrobials. Radiation cariesand hypersensitivity can be controlled bydaily topical fluoride applicationsubstitu sugar with alcohol base sugar and mouthwashes of sodium bicarbonate may help dissolve the stringy saliva that forms Dentures be made after 4-6 weeks of therapy to prevent ucosities formation. Xerostomia by artificial saliva 19. squamous cell carcinoma - carcinoma of the tongue is common in alcoholic and smokers male 20. Hypertension the anxiety associated with dental treatment typically causes a rise in blood pressure and may rarely precipitate cardiac arrest or cerebrovascular accident. Preoperative assurance and sedation are therefore important. Patients are best treated in the morning and given short appointments only. Aspirating syringes may be used to give the local anesthetic, since cathecolamines given intravenously may increase hypertension and precipitate arrythmias. Patient with hypertension may also have management problems caused by the underlying disease or

complications such as cardiac or renal failure. Systemic corticosteriods may raise the blood pressure and antihypertensive treatment may have to be adjusted accordingly. 21. Angina before dental treatment, patient must be reassured and possibly sedated with oral diazepam. If angina has followed attention in a particular patient, he should be given his usual medication ( glyceryl trinitrate) before treatment in started. This drug should be readly available in case of emergency. If the patient experiences chest pain, the treatment shouldbe stopped and be given glyceryl trinitrite 0.5 sublingually and oxygen and be kept sitting upright. The pain should be relieved in 2-3 minutes if not the patient is having a myocardial infarction and should be brought to the hospital. For anything but minor treatment under local anesthesia, the physician should be consulted and consideration should be given to any other complicating factors such as beta-blockers terapy, hypertension and cardiac failure. 22. Myocardial infarction consult physician before undertaking operative treatment. General anesthesia is contraindicatd if a recent attack happened. Treatment under GA should be postposed up to 3 months or better yet after a year. Only simple emergency dental treatment under LA should be given within 3-6 months. LA should be given in low doses and without undue anxiety and avoid intravenous injection.

Você também pode gostar